Inbound 3653954915351255072

Download as pdf or txt
Download as pdf or txt
You are on page 1of 153

Engr.

, LPT
FUNDAMENTALS OF DIFFERENTIAL CALCULUS (MODULES/WORKBOOK)
© 2022 Published by University of Eastern Philippines – Main
Catarman, Northern Samar, 6400, Philippines
[email protected]

AUTHOR:
KISSA P. BANAWIS
ALL RIGHTS RESERVED.

No part of the material protected by this copyright notice may be reproduced or


utilized in any form or by means, electronic or mechanical, including photocopying,
recording, or by any information storage and retrieval system, without written
permission form the copyright owner.

ISBN: 978-971-672-125-6
Preface
Fundamentals of Differential Calculus is the instructional
material I constructed to facilitate easy teaching and learning
process.

The need to come up with this instructional module was


due to pandemic brought by CoVID-19, which requires flexible
learning modality. This module in some way aids the absence
of face-to-face classes. Moreover, this can still be used even
after pandemic.

This learning material is intended to all students studying


Differential Calculus and the educators who are teaching the
subject. This module contains simplified discussion of some of
the important topics and learning tasks. These tasks are geared
towards the understanding and mastery of concepts in this
subject.

It is my great hope that the students and fellow educators


would find this module significant in learning and teaching
Differential Calculus.

Engr. KISSA P. BANAWIS


Contents
Module 1: FUNCTIONS AND ITS APPLICATION .............................................................................. 1
Overview ..................................................................................................................................... 1
Learning Outcomes ..................................................................................................................... 1
Lesson 1.1: Functions .................................................................................................................. 1
1.1.1 Domain and Range ......................................................................................................... 1
1.1.2 Classification of Functions .............................................................................................. 3
1.1.3 Piecewise Defined Function ........................................................................................... 6
1.1.4 Symmetry ....................................................................................................................... 8
1.1.5 Increasing and Decreasing Functions ............................................................................. 9
Student Activity 1.1 ................................................................................................................... 10
Lesson 1.2: Application of Functions ......................................................................................... 11
Student Activity 1.2 ................................................................................................................... 13
Summary ................................................................................................................................... 14
References ................................................................................................................................ 15
Module 2: LIMITS AND CONTINUITY ............................................................................................ 16
Overview ................................................................................................................................... 16
Learning Outcomes ................................................................................................................... 16
Lesson 1: Functions ................................................................................................................... 16
2.1.1 Limit of a Function ....................................................................................................... 16
2.1.2 One-Sided Limits .......................................................................................................... 19
2.1.3 Infinite Limits ............................................................................................................... 21
2.1.4 Theorem on Limits ....................................................................................................... 23
Student Activity 2.1 ................................................................................................................... 26
Lesson 2: Continuity .................................................................................................................. 27
2.2.1 Continuity of a Function at a Point ............................................................................... 27
2.2.2 Continuity of a Function on an Open Interval .............................................................. 28
2.2.3 Theorems of Continuity ............................................................................................... 29
2.2.4 Removable Discontinuity ............................................................................................. 29
2.2.5 Jump Discontinuity ....................................................................................................... 30
Student Activity 2.2 ................................................................................................................... 31
Summary ................................................................................................................................... 31
References ................................................................................................................................ 32
Module 3: THE DERIVATIVE.......................................................................................................... 33
Overview ................................................................................................................................... 33
Learning Outcomes ................................................................................................................... 33
Lesson 3.1: The Derivative ........................................................................................................ 33
3.1.1 The Tangent Problem ................................................................................................... 33
3.1.2 Definition ..................................................................................................................... 35
3.1.3 Determination of the Derivative .................................................................................. 35
Student Activity 3.1 ................................................................................................................... 37
3.1.4 Derivative Interpreted as Slope.................................................................................... 37
3.1.5 Rate of Change ............................................................................................................. 41
Student Activity 3.2 ................................................................................................................... 42
Summary ................................................................................................................................... 42
References ................................................................................................................................ 43
Module 4: ALGEBRAIC FUNCTIONS .............................................................................................. 44
Overview ................................................................................................................................... 44
Learning Outcomes ................................................................................................................... 44
Lesson 4.1: General Theorems of Differentiation ..................................................................... 44
4.1.1 Theorem 1: Derivative of a Constant ........................................................................... 44
4.1.2 Theorem 2: Power Rule ................................................................................................ 45
4.1.3 Theorem 3: Constant Multiple Rule ............................................................................. 46
4.1.4 Theorem 4: Derivative of Sum of Functions ................................................................. 47
4.1.5 Theorem 5: Derivative of Difference of Functions ....................................................... 47
4.1.6 Theorem 6: Product Rules ............................................................................................ 49
4.1.7 Theorem 7: Quotient Rules .......................................................................................... 51
4.1.8 Theorem 8: The General Power Formula ..................................................................... 53
Student Activity 4.1 ................................................................................................................... 55
Lesson 4.2: The Chain Rule ........................................................................................................ 55
Lesson 4.3: Higher Derivatives .................................................................................................. 57
Lesson 4.4: Implicit Differentiation ........................................................................................... 60
Student Activity 4.2 ................................................................................................................... 64
Summary ................................................................................................................................... 64
References ................................................................................................................................ 65
Module 5: TRIGONOMETRIC AND INVERSE TIRGONOMETRIC FUNCTIONS ................................ 66
Overview ................................................................................................................................... 66
Learning Outcomes ................................................................................................................... 66
Review: Trigonometric Identities .............................................................................................. 66
Lesson 5.1: Derivative of Trigonometric Functions ................................................................... 67
Student Activity 5.1 ................................................................................................................... 76
Lesson 5.2: Derivative of Inverse Trigonometric Functions ....................................................... 76
Student Activity 5.2 ................................................................................................................... 82
Summary ................................................................................................................................... 82
References ................................................................................................................................ 83
Module 6: EXPONENTIAL AND LOGARITHMIC FUNCTIONS ......................................................... 84
Overview ................................................................................................................................... 84
Learning Outcomes ................................................................................................................... 84
Lesson 6.1: Exponential Functions ............................................................................................ 84
Lesson 6.2: Derivatives of Exponential Functions ..................................................................... 85
Student Activity 6.1 ................................................................................................................... 87
Lesson 6.3: Logarithmic Functions ............................................................................................ 88
Lesson 6.4: Derivative of Logarithmic Functions ....................................................................... 90
Student Activity 6.2 ................................................................................................................... 93
Summary ................................................................................................................................... 93
References ................................................................................................................................ 93
Module 7: POLYNOMIAL CURVES ................................................................................................ 94
Overview ................................................................................................................................... 94
Learning Outcomes ................................................................................................................... 94
Lesson 7.1: Tangents and Normals to Plane Curves .................................................................. 94
7.1.1 Tangent Line................................................................................................................. 94
7.1.2 Normal Line .................................................................................................................. 95
Lesson 7.2: Critical Points, Maxima and Minima ....................................................................... 99
7.2.1 Increasing and Decreasing ........................................................................................... 99
7.2.2 Maximum and Minimum Points ................................................................................. 101
7.2.3 Tests ........................................................................................................................... 101
Lesson 7.3: Concavity .............................................................................................................. 109
7.3.1 Points of Inflection ..................................................................................................... 110
Lesson 7.4: Sketching of Polynomial Curves............................................................................ 111
Student Activity 7.1 ................................................................................................................. 118
Summary ................................................................................................................................. 118
References .............................................................................................................................. 119
Module 8: APPLICATION OF DERIVATIVES ................................................................................. 120
Overview ................................................................................................................................. 120
Learning Outcomes ................................................................................................................. 120
Lesson 8.1: Applications of Maxima and Minima .................................................................... 120
Student Activity 8.1 ................................................................................................................. 137
Lesson 8.1: Time Rates ............................................................................................................ 138
Student Activity 8.2 ................................................................................................................. 146
Summary ................................................................................................................................. 146
References .............................................................................................................................. 146
OVERVIEW

MODULE Differential Calculus, a branch of Calculus, is an


introductory course covering the core concepts of limit,
continuity and differentiability of functions involving one or
more variables. This also includes the application of
differential calculations in solving problems on optimization,
rates of change, related rates, tangents and normal, and
approximations; partial differentiation and transcendental
curve tracing.

This module will introduce you to functions and limits which are the basic
concepts in differential calculus.

LEARNING OUTCOMES

At the end of this module, you should be able to:


1. Find the value of a function;
2. Solve for the domain and range of a function;
3. Evaluate piecewise defined functions;
4. Apply the concept of functions in solving problems.

LESSON 1.1: FUNCTIONS

1.1.1 Domain and Range

A function f is a rule that assigns to each element x in a set D exactly one


element, called f(x), in a set E.

x f(x)

y f(a)

f
D E
Functions arise whenever one quantity depends on another. Example is the
area of a circle, A = πr2, which depends on the radius r of the circle. In the equation,
A is the dependent variable and r is the independent variable. For every value of
r, there is a corresponding value of A.

We usually consider functions for which the sets D and E are sets of real
numbers. The set D is called the domain of the function. The number f(x) is the
value of f at x and is read “f of x.” The range of f is the set of all possible values
of f(x) as x varies throughout the domain. A symbol that represents an arbitrary
2

number in the domain of a function of f is called an independent variable. A


symbol that represents a number in the range of f is called a dependent variable.

We stress that a function can be represented in different ways: by an


equation, in a table, by a graph, or in words. If a single function can be represented
in all four ways, it’s often useful to go from one representation to another to gain
additional insight into the function.

ILLUSTRATIVE EXAMPLES

1
1. Find the domain of each function: a. f(x) = √𝑥 + 2, b. f(x) = 𝑥 2 − 𝑥

a. f(x) = √𝑥 + 2
Since a root of a negative value leads to undefined value of the function, then
x + 2 should be greater or equal to 0 or x + 2 ≥ 0.
x+2≥0 use your knowledge in evaluating inequalities
x+2-2≥0–2 subtracting 2 to both sides of the equation
x≥–2 this means that the minimum value of x should be –2.
Therefore, the domain is the interval [-2, ∞), this means that the value of x
should be from –2 and above.

1
b. g(x) = 𝑥 2 − 𝑥
For a rational fraction to be defined, its denominator should not be equal to
zero.
1 1
g(x) = 𝑥 2 − 𝑥 = 𝑥 (𝑥 − 1) factor the denominator

Values of x which will give zero denominators to the function are 0 and 1.
Therefore, the domain for the function g is all real numbers except for 0 and 1
or {x | x ≠ 0, x ≠ 1}.

3. Find the domain and range of each function: a. f(x) = 2x – 1, b. g(x) = x2

a. f(x) = 2x – 1
Any real number can be substituted to the
variable x and will result to real number, as well.
Therefore, the domain is ℝ and the range is also ℝ. y = 2x - 1
3

b. g(x) = x2
As observed in the graph of the function
on the right, any real number can be
substituted to the function g. Therefore, the
domain of g is ℝ. y = x2
Upon substituting both positive and
negative values to x, the result is always
positive and has a minimum value of 0.
Therefore, the range is from 0 to ∞ value or [0,
∞).

1.1.2 Classification of Functions

There has been already a lot functions that you encountered since you
started studying through the ladder of mathematics, from Algebra to Analytic
Geometry. For better understanding, one should know the classification of these
functions.

Fundamentally, functions are classified into two (2) main categories, the
algebraic functions and the transcendental functions. The algebraic functions
consist of the rational integral functions (polynomials), rational fractions (quotients
of polynomials), and irrational functions. The simplest algebraic functions are
those formed from rational functions through the extraction of roots. The
elementary transcendental functions are the trigonometric functions, inverse
trigonometric functions, exponential functions, and the logarithmic functions.

A function may be classified also according to the number of values the


dependent variable will have for a particular value of the independent variable. The
value of a function is determined by substituting a value for the independent
variable and solving the value of the dependent variable.

When the relation y = f(x) is such that there is only one value of “y” for
each acceptable “x”, then the f(x) is said to be a one-valued (single-valued)
function of x.

When the relation y = f(x) is such that there are two or more values of “y”
for each acceptable “x”, then the f(x) is said to be a multi-valued function of x.

Almost all functions are single-valued. A typical two-valued function is the


square root function or in symbol: y = √𝒙, and the two possible values of y are: y
= + √𝒙 , y = – √𝒙
4

ILLUSTRATIVE EXAMPLES

𝒙−𝟏
1. Given f(x) = 𝒙𝟐 +𝟐, find (a) f (0); (b) f (−1); (c) f (2a); (d) f (1/x); (e) f (x + h).

0−1 𝟏
a. f(0) = 02 +2 = − 𝟐
(−1)−1 𝟐
b. f(-1) = (−1)2 +2 = − 𝟑
2𝑎−1 2𝑎−1 𝟐𝒂−𝟏
c. f(2a) = (2𝑎)2 +2 = 4𝑎2 +2 = 𝟐(𝟐𝒂𝟐 +𝟏)
1 1−𝑥 1−𝑥
𝑥
−1
𝑥 𝑥 1−𝑥 𝑥2 𝒙 (𝟏 − 𝒙)
d. f(1/x) = 1 2
= 1 = 1 + 2𝑥2
=( ) (1 + 2𝑥 2 ) =
( ) +2 +2 𝑥 𝟏 + 𝟐𝒙𝟐
𝑥 𝑥2 𝑥2

(𝑥+ℎ) − 1 𝒙+𝒉−𝟏
e. f (x + h) = (𝑥+ℎ)2 +2 = 𝒙𝟐 +𝟐𝒉𝒙+ 𝒉𝟐 +𝟐

2. If f(x) = x3 – x + 3, find f(0), f(2), f(-4), f(3/2) and f(-2y).


a. f(0) = 03 – 0 + 3 = 3
b. f(2) = (2)3 – 2 + 3 = 8 – 2 + 3 = 9
c. f(-4) = (-4)3 – (-4) + 3 = -64 + 4 +3 = - 57
d. f(3/2) = (3/2)3 – 3/2 + 3 = 27/8 – 3/2 + 3 = (27 – 12 + 24)/8
= 39/8
e. f(-2y) = (-2y)3 – (-2y) + 3 = -8y3 + 2y + 3

3. If f(x) = x2 (x – 3), find f(-¼), f(1¼), f(√2), f(3 – y), f(y2 – y).

1 −1−12 𝟏𝟑
a. f(-¼) = (-¼)2 (-¼ – 3) = 16 ( ) = − 𝟔𝟒
4

5 2 5 25 5−12 25 −7
b. f(1¼) = (1¼)2 (1¼ – 3) = (4) (4 − 3) = 16 ( ) = 16 ( 4 )
4
−𝟏𝟕𝟓
= 𝟔𝟒

c. f(√2) = (√2)2 (√2 – 3) = 2 (√𝟐 – 3)


d. f(3 - y) = (3 - y)2 (3 - y – 3) = (3 - y)2 (–y) = -y (3 - y)2
e. f(y2 - y) = (y2 - y)2 (y2 - y – 3) = [y(y – 1)]2 (y2 - y – 3)
= y2 (y – 1)2 (y2 - y – 3)

28
4. If f (x) = 3x, show that: a. f (x - 2) + f (x + 1) = 9
f(x), and
𝑓(𝑥+5)
b. 𝑓(𝑥−2) = f(7)
28
a. f (x - 2) + f (x + 1) = 9
f(x)
28
3(x - 2) + 3(x + 1) = f(x) Substitute (x – 2) and (x + 1) to the function
9
3x
28
3x 3-2 + 3x 3 = f(x) Exponent property ax+y = ax ay
9
28
3x (3-2 + 3) = f(x) Factor out the common factor 3x
9
5

1 28
3x (32 + 3) = f(x) Exponent property a-y = 1/ay
9
1+27 28
3x ( )= f(x) LCD of 1/32 and 3 is 9, then add the fractions
9 9
28 28
3x ( 9 ) = f(x) 3x is f(x)
9
28 28 28
( 9 ) f(x) = f(x) Therefore, f (x - 2) + f (x + 1) = 9
f(x)
9

𝑓(𝑥+5)
b. = f(7)
𝑓(𝑥−2)
3𝑥+5
= f(7) Substitute (x + 5) and (x – 2) to the function
3𝑥−2
3x
3𝑥 35
= f(7) Exponent property ax+y = ax ay
3𝑥 3−2
35
= f(7) 3x/3x = 1
3−2
5 2
3 3 = f(7) Exponent property 1/a-y = ay
35+2 = f(7) Exponent property ax+y = ax ay
37 = f(7) f(7) = 37
𝑓(𝑥+5)
f(7) = f(7) Therefore,
𝑓(𝑥−2)
= f(7)

5. If g(x) = cos 2x, find g(π/4), g(-x), g(π – x), and g(x – π/2)

a. g(π/4) = cos 2(π/4)


= cos (π/2) π/2 is an angle expressed in radians and is
equivalent to 90°
=0 cos π/2 or cos 90° is equal to zero.
b. g(-x) = cos 2(-x)
= cos (-2x)
= cos 2x using even identity cos (–x) = cos x

c. g(π – x) = cos 2(π – x)


= cos (2π – 2x)
= cos 2π cos x + sin 2π sin x
using identity
cos (x – y) = cos x cos y + sin x sin y
= cos x cos 2π = 1 and sin 2π = 0

𝑠𝑒𝑐 2 𝑥 tan 𝑦
6. If f(x) = tan x, show that f(x+y) – f(x) =
1−tan 𝑥 tan 𝑦

𝑠𝑒𝑐 2 𝑥 tan 𝑦
tan (x+y) – tan x =
1−tan 𝑥 tan 𝑦
tan 𝑥+tan 𝑦 𝑠𝑒𝑐 2 𝑥 tan 𝑦 tan 𝑥+tan 𝑦
– tan x = tan (x+y) =
1−tan 𝑥 tan 𝑦 1−tan 𝑥 tan 𝑦 1−tan 𝑥 tan 𝑦

tan 𝑥 +tan 𝑦−tan 𝑥 (1−tan 𝑥 tan 𝑦) 𝑠𝑒𝑐 2 𝑥 tan 𝑦


= Addition of rational expression
1−tan 𝑥 tan 𝑦 1−tan 𝑥 tan 𝑦
tan 𝑥 + tan 𝑦−tan 𝑥+tan2 𝑥 tan 𝑦 𝑠𝑒𝑐 2 𝑥 tan 𝑦
= Distributive Law
1−tan 𝑥 tan 𝑦 1−tan 𝑥 tan 𝑦
6

tan 𝑦+tan2 𝑥 tan 𝑦 𝑠𝑒𝑐 2 𝑥 tan 𝑦


= tan x – tan x = 0
1−tan 𝑥 tan 𝑦 1−tan 𝑥 tan 𝑦
tan 𝑦(1+tan2 𝑥) 𝑠𝑒𝑐 2 𝑥 tan 𝑦
= Factor out tan y
1−tan 𝑥 tan 𝑦 1−tan 𝑥 tan 𝑦
𝑠𝑒𝑐 2 𝑥 tan 𝑦 𝑠𝑒𝑐 2 𝑥 tan 𝑦
= 1+tan2x = sec2x
1−tan 𝑥 tan 𝑦 1−tan 𝑥 tan 𝑦

1.1.3 Piecewise Defined Functions

A piecewise defined function f(x) is a function that is given by different


formulas for different values of x. For example, in some applications, a function
may be given by one formula when x is positive and by a different formula when x
is negative as in the case of the absolute value function. Sometimes these
functions meet at the endpoints on intervals on which they are defined and are
continuous. Sometimes they have jumps.

ILLUSTRATIVE EXAMPLES
1 − 𝑥 𝑖𝑓 𝑥 ≤ 1
1. A function f is defined by f(x) =
𝑥2 𝑖𝑓 𝑥 > 1
Evaluate f(0), f(1), and f(2) and sketch the graph.

a. f(0)
Since x is less than 1, then we use the equation of the function for x ≤ 1
which is 1 – x.
f(0) = 1 – x = 1 – 0 = 1

b. f(1)
Since x is equal to 1, then we use the equation of the function for x ≤ 1
which is 1 – x.
f(1) = 1 – x = 1 – 1 = 0

c. f(2)
Since x is greater than 1, then we use the equation of the function for x
> 1 which is x2. f(2) = x2 = (2)2 = 4

In drawing the graph, we first plot the points (0, 1) and


(1, 0) which satisfy the first formula of the function, 1 – x.
Extend the graph of this line infinitely to the left of the graph
representing the range of the function for x ≤ 1. Next, we draw
the graph of the second equation x2 which is a parabola
including point (2, 4). But, you should take note that the second
graph represents all points greater than 1.
7

So the final graph should be as shown. The solid dot indicates that the point
(1, 0) is included on the graph; the open dot indicates that the point (1, 1) is
excluded from the graph.

2. Sketch the graph of the absolute value function f(x) = |x|.


𝑥 𝑖𝑓 𝑥 ≥ 0
|x| =
−𝑥 𝑖𝑓 𝑥 < 0
y = |x|
We see that the graph of f coincides with the
line y = x to the right of the y-axis and coincides with
the line y = - x to the left of the y-axis.

3. Find a formula for the function f as graphed.

As you can see in the graph, the function


f is composed of three (3) equations.

For the first equation, let us use our


knowledge in analytic geometry in forming
the equation. Consider the line
connecting the points (0, 0) and (1, 1),
using the equation, y = mx + b, where m = 1 and y-intercept b = 0, its
equation is y = x. Thus, for the part of the graph from points (0,0) to (1, 1)
we have
f(x) = x if 0 ≤ x ≤ 1

For the second equation, m = -1, b = 0, so its point-slope form is


y – y1 = m (x – x1)
y – 0 = -1 (x – 2)
y = -x + 2
y=2–x
Thus, we have f(x) = 2 – x if 1 < x ≤ 2

For the last equation, the horizontal line lies at the x axis, which means that
y = 0 for values greater than 2.
f(x) = 0 if x > 2

Finally, the equation of the function f is


𝑥 𝑖𝑓 0 ≤ 𝑥 ≤ 1
f(x) = 2 − 𝑥 𝑖𝑓 1 < 𝑥 ≤ 2
0 𝑖𝑓 𝑥 > 2
8

1.1.4 Symmetry

If a function satisfies for every number in its


domain, then is called an even function. For
instance, the function f(x) = x2 is even because f(-x) f(x)
f(-x) = (-x)2 = x2 = f(x)

The geometric significance of an even function


is that its graph is symmetric with respect to the
y-axis.

If satisfies for every number in its domain, then is called an odd function. For
example, the function f(x) = x3 is odd because
f(-x) = (-x)3 = -x3 = f(x)

The graph of an odd function is symmetric


about the origin. If we already have the graph
of f for x ≥ 0, we can obtain the entire graph f(x)
by rotating this portion through 180° about
the origin.

ILLUSTRATIVE EXAMPLES

Determine whether each of the following functions is even, odd, or neither even
nor odd.
1. f(x) = x5 + x
To determine if the function is odd or
even, we substitute –x to the function.
f
f(-x) = (-x)5 + (-x)
= -x5 – x
= - (x5 + x) since the sign of the
original equation
changed, then the
function is odd. This is
confirmed by drawing
its graph.
9

2. g(x) = 1 – x4
To determine if the function is odd or even, we g
substitute –x to the function.
g(-x) = 1 – (-x)4
= 1 – x4 since it is equal to the original
equation, then the function is
even. This is confirmed by
drawing its graph.

3. h(x) = 2x – x2
To determine if the function is odd or even, we
substitute –x to the function.
h
h(-x) = 2(-x) - (-x)2
= -2x – x2
= - (2x + x2) since h(-x) ≠ h(x) and h(-x) ≠
- h(x), then the function is neither even nor odd.
As seen in the graph, it is symmetrical neither to
y-axis nor about the origin.

1.1.5 Increasing and Decreasing Functions

A function f is called increasing on an interval I if

f(x1) < f(x2) whenever x1 < x2 in I

It is called decreasing on I if

f(x1) > f(x2) whenever x1 > x2 in I

The graph shown in the figure rises from A


y = f(x)
to B, falls from B to C, and rises again from C to D. ……
The function f is said to be increasing on the interval
[a, b], decreasing on [b, c], and increasing again on
[c, d]. Notice that if x1 and x2 are any two numbers
between a and b with x1 < x2, then f(x1) < f(x2).

STUDENT ACTIVITY 1.1


10

I - Find the value of the function given in each number for the specified values of
“x”:

1. If f(x) = √𝟗 – 𝒙𝟐
a. find f(-3), f(2), f(4/3), f(y + 3), and f(3 sin x).

𝟏 𝟏
2. If f(x) = 1 – 𝟐x + 𝟒x2
a. find f(1), f(-2), f(3), f(½) and f(4x).

3. If f(x) = 2x, show that:


15
a. f(x + 3) – f(x – 1) = f(x), and
2
𝑓(𝑥+3)
b. 𝑓(𝑥−1) = f(4)
4. Let f(x) = x2 − 2x + 3.
a. evaluate f(3), f(−3), f(−x), f(x + 2), f(x −2), f(x + h)
𝑓(𝑥+ℎ)−𝑓(𝑥)
b. solve for f(x + h) − f(x) and ℎ

𝒙−𝟏
5. If f(x) =
𝒙+𝟏
a. find f(0), f(1), f(−2)
1 1 1
b. show that f(𝑥) = −f(x) and f(− 𝑥) = − 𝑓(𝑥)

II - Determine the domains of the functions

1. y = √4 − 𝑥 2
2. y = √𝑥 2 − 16
1
3. y = 𝑥−2
1
4. y = 𝑥 2 −9
𝑥
5. y = 𝑥 2 +4

III - Draw the graph and find the domain and range of the functions

1. f(x) = √4 − 𝑥 2
𝑥 2 −4
2. f(x) = 𝑥−2

3. f(x) = |x – 3|
4
4. f(x)= 𝑥
𝑥 𝑖𝑓 𝑥 ≥ 0
5. f(x) =
2 𝑖𝑓 𝑥 < 0

LESSON 1.2: APPLICATION OF FUNCTIONS


11

Let us now apply what we have learned about functions. We will consider
the relationship of the area of a circle and its radius. The area A of a circle can be
expressed as a function of the radius r, that is, A = πr2. Also, the Circumference C
of a circle can be expressed as a function of its radius r, C = 2πr. There are infinite
number of application of function that can be demonstrated to real world situation.
In this section, we will use functional notation to solve real world problems.

ILLUSTRATIVE EXAMPLES

1. The director of Land Transportation Franchising and Regulatory Board (LTFRB)


charges a taxi fare of ₱ 25.00 for the first 100 meters and ₱ 2.00 for every
succeeding 300 meters or portion thereof. Taxi fare is a function of the distance
traveled. Devise a formula on how he can compute the fare.

Solution:

Let x = distance traveled.


f(x) = taxi fare as a function of the distance x traveled.
₱ 25.00 = minimum fare for the first 100 meters.
₱ 2.00 = fare for every succeeding 300 meters or portion thereof.

Hence, the formula for computing the fare is given by

f(x) = 2x+25

The distance traveled is in the interval of 300 after 100 meters and even if
only a portion of 300 meters after 100 meters has been traveled, 300 meters should
be used.

2. The general manager of computer sales center is paid a monthly salary of ₱


120,000 plus 5% commission on sales. Express his total monthly pay using
functional notation. Assume that he sells x pesos of goods per month. Compute
his income if his sales is ₱ 1.5 million.

Our goal is to:


a. Use functional notation to express the manager’s income per month, I (x).
b. Find the total income for the month.

Solution:

Let The monthly income is ₱ 120, 000 plus 5% commission sales.


₱ 1.5 million = total sales
I (x) = manager’s income which is a function of sales x

a. We can express the manager’s income using functional notation.


I(x) = variable income + fixed income (salary per month)
12

I(x) = 0.05x + 120,000

b. Total income for the month on sales of ₱ 1.5 million.


I(x) = 0.05(1,500,000)+120,000
I(x) = 75,000+120,000
I(x) = ₱ 195,000.00

3. The sum of two positive number is 100. Let one of the number be x and express
the product P of the two numbers as a function of x.

Solution:
Let x =first number
100-x = other number
P = product of the two numbers

We express P as the product of two numbers which is a function of x. that is,


P(x) = x (100-x)
P(x) = 100x – x2

4. A rectangle has a perimeter of 10 meters. Express the area A of the rectangle


as a function of the width x.

Solution:
We express the area of a rectangle as a function of x using the perimeter of
the rectangle (see figure 1.16)
y

x x

y
The Perimeter of the rectangle is
P = 2x + 2y (1)

While the Area of the rectangle is


A = xy (2)

Since P = 10 meters, we solve (1) for y in terms of x, we get

10 = 2x + 2y 2x + 2y = 10
x+y=5 dividing both sides of the
equation by 5
y=5–x (3)
We can express the area of the rectangle in terms of the width x by
substituting (3) into (2).
13

That is,
A = xy
A = x (5-x)

5. The demand function for a certain product is given by


(𝑥 + 5)2
𝑝 = 289 −
10,000
Where p is the price per unit and x is the number of units sold.
a. Find x as a function of p.
b. Use the result of part (a) to find the number of units sold when the price is
₱189.00.

Solution:
a. Solving for x in terms of p in the given function by multiplying both sides of
the equation by 10,00, we get
10,000𝑝 = 2,890,000 − (𝑥 + 5)2
(𝑥 + 5)2 = 2,890,000 − 10,000𝑝
𝑥 + 5 = √2,890,000 − 10,000𝑝
𝑥 = √2,890,000 − 10,000𝑝 − 5

b. Let p = ₱ 189.00, we want to solve for x in part a. that is,


𝑥 = √2,890,000 − 10,000𝑝 − 5
𝑥 = √2,890,000 − 10,000(189) − 5
𝒙 = 𝟗𝟗𝟓 𝒖𝒏𝒊𝒕𝒔

STUDENT ACTIVITY 1.2

1. Galileo Galilei (1564-1642) is also well known for his discoveries about
swinging weight that led to the invention of the pendulum clock. He observed
that the length of a pendulum is a function of the time of swing, as seen in the
following table.
Time of swing Length of the pendulum
t F (t)
1 second 1 unit
2 seconds 4 units
3 seconds 9 units
4 seconds 16 units
5 seconds 25 units
6 seconds 36 units
a. Devise a formula that defines the function
b. Find the length of the pendulum when the time of swings is 10
seconds
c. Find the time of swing when the length of the pendulum is 81 units.
14

2. The area A of a circle is the product of the square of its radius r and π while the
circumference of a circle is the product of π and twice the radius r. express the
area A as function of its circumference.
3. A circular disc of radius x is cut out from a square piece of card board. Express
the area of wasted material a function of x.
4. A farmer bought 20,000 meters of fencing materials to enclose the rectangular
lot to be planted with mango trees. Express the area of the lot as a function of
the length x.
5. The product of two numbers is 50. Call one number as x. Express the sum of
two numbers in terms of x.
6. A rectangle length 10 meters and width 4 meters is inscribed in a circle. How
long is the radius of the circle?
7. Express the area of an equilateral triangle in terms of perimeter. Use A for area
and P for perimeter.
8. If the volume of the right circular cone is 6 cubic centimeters, express the
surface area S of the right circular cone as a function of its radius r.
9. The volume of the sphere is four third π times the cube of its radius. Express
the volume V of the sphere as a function of its radius.
10. A wire 252 cm long will be bent into the shape of right triangle whose sides are
in the ratio of 3:4:5. How long is the hypotenuse of the triangle?

SUMMARY

Functions arise whenever one quantity depends on another. Example is the


area of a circle. The number f(x) is the value of f at x and is read “f of x.” The
range of f is the set of all possible values of f(x) as x varies throughout the domain.

Functions are classified into two (2) main categories, the algebraic
functions and the transcendental functions. It may be classified also according
to the number of values the dependent variable will have for a particular value of
the independent variable, one-valued (single-valued) and multi-valued function
of x.

A piecewise defined function f(x) is a function that is given by different


formulas for different values of x.

If a function satisfies for every number in its domain, then is called an even
function. The geometric significance of an even function is that its graph is
symmetric with respect to the y-axis. On the other hand, if f satisfies for every
number in its domain, then is called an odd function. The graph of an odd function
is symmetric about the origin.

A function f is called increasing on an interval I if f(x1) < f(x2) whenever x1 <


x2 in I. It is called decreasing on I if f(x1) > f(x2) whenever x1 > x2 in I.
15

There are infinite number of applications using functional notation to solve


real world problems. Reading comprehension is very essential in formulating
equations of functions described in a problem.

REFERENCES

Love, Clyde E., Differential and Integral Calculus, The Macmillan


Company
Morado, Lonel E., Module in Differential Calculus
Ryan, Mark, Calculus Essentials for Dummies , Wiley Publishing , Inc.,
2010
Stewart, James, Single Variable Calculus , Brooks/Cole CENGAGE
Learning, 7th Edition, 2012
16

OVERVIEW

MODULE This module will start with the application of functions and
discuss about continuity which are also essential topics that you
should know before dealing with derivatives.

LEARNING OUTCOMES
At the end of this module, you should be able to:
5. train students on the determination of the limit of a
function using laws
6. determine the numbers which a function is continuous
7. find the points of discontinuity in the graph of a certain function.

LESSON 2.1: LIMITS

2.1.1 Limit of a Function

The limit of a function (L) is the value of a function when the value of the
independent variable (x) approaches a value “a”. This is written as:

𝐥𝐢𝐦 𝒇(𝒙) = L
𝒙→𝒂

which is read “the limit of f(x), as ‘x’ approaches ‘a’, is L” if we can make the
values of f(x) arbitrarily close to L (as close to L as we like) by taking x to be
sufficiently close to a (on either side of a) but not equal to a.

Notice the phrase “but x ≠ a” in the definition of limit. This means that in
finding the limit of f(x) as x approaches a, we never consider x = a. In fact, f(x) need
not even be defined when x = a. The only thing that matters is how f is defined near
a.

The following figures show the graphs of three cases where 𝐥𝐢𝐦 𝒇(𝒙) = L is
𝒙→𝒂
true.

f(a) is equal L
17

f(a) is not equal to L f(a) is not defined

There are two ways in solving for the limit. First method is by substituting all
values near a in both sides to get f(a), the more values you assign, the easier for
you to guess what the limit is.

Second method is by direct substitution property stating that if f is a


polynomial or a rational function and a is in the domain of f, then 𝐥𝐢𝐦 f(x) =
𝒙→𝒂
f(a). This is direct substitution of a to the equation (but always bear in mind that x
≠ a, this method is only done to easily get the value of the limit L). In using the
second method, make sure that the equation is completely factored, specially the
rational expressions. You should be careful in using this method because this is
not always applicable to use specially when dealing with rational expressions and
with values of x approaching zero.

ILLUSTRATIVE EXAMPLES
𝒙−𝟏
1. Find the limit of 𝐥𝐢𝐦 𝒙𝟐 − 𝟏
𝑥→1

Method 1: You can tabulate all the points close to 1, both less and more than 1.
As you can see in the table below, values of f(x) for x close to 1, are close to 0.5.
x<1 f(x) x>1 f(x)
0.5 0.666667 1.5 0.400000
0.9 0.526316 1.1 0.476190
0.99 0.502513 1.01 0.497512
0.999 0.500250 1.001 0.499750
0.9999 0.500025 1.0001 0.499975

𝒙−𝟏
Thus, 𝐥𝐢𝐦 𝒙𝟐 − 𝟏 = 0.5
𝑥→1

Method 2: By direct substitution method.


𝑥−1 𝑥−1
lim 𝑥 2 − 1 = 𝑥2 − 1
𝑥→1
𝑥−1
= factor the denominator
(𝑥 − 1)(𝑥+1)
1
= 𝑥+ 1 (x – 1)/(x – 1) = 1
1
= 1+1 substitute 1 to x
1
= 2 = 0.5 the value of limit L
18

The two methods gained the same answer.


√𝒕𝟐 +𝟗 − 𝟑
2. Estimate the value of 𝐥𝐢𝐦
𝑡→0 𝒕𝟐

Method 1: Assign values near zero, as shown in the table.

√𝑡 2 + 9 − 3
t
𝑡2
±1.0 0.16228
±0.5 0.16553
±0.1 0.16662
±0.05 0.16666
±0.01 0.16667

√𝒕𝟐 +𝟗 − 𝟑
Therefore, it is safe to guess that 𝐥𝐢𝐦 = 1/6.
𝑡→0 𝒕𝟐

Method 2: In order for us to use direct substitution method, first, we need to


rationalize the expression.
√𝑡 2 +9 − 3 √𝑡 2 +9 + 3
= lim ( ∙ )
𝑡→0 𝑡2 √𝑡 2 +9 + 3
𝑡 2 +9 − 9
= lim [ ]
𝑡→0 𝑡 2 (√𝑡 2 +9 + 3)
𝑡2
= lim [ ]
𝑡→0 𝑡 2 (√𝑡 2 +9 + 3)
1
= lim [ ]
𝑡→0 (√𝑡 2 +9 + 3)
lim 1
𝑡→0
=
lim (√𝑡 2 +9 + 3)
𝑡→0
1
= (√0+9 + 3)
1 𝟏
= (3 + 3) =
𝟔

𝒔𝒊𝒏 𝒙
3. Guess the value of 𝐥𝐢𝐦
𝑥→0 𝒙

In this problem, the method that should be


used is Method 1, since direct substitution will lead
to an undefined answer. Take note that in dealing
with this kind of problem, angle x is in radians.
Assigning values of x close to zero, approximates
the
𝒔𝒊𝒏 𝒙
𝐥𝐢𝐦 𝒙 = 1.
𝑥→0
This function will be described in a
theorem in the following section.

𝝅
4. Investigate 𝐥𝐢𝐦 𝒔𝒊𝒏
𝑥→0 𝒙
19

Again, direct substitution in this problem leads to undefined value of the limit.
Using Method 1, trying to substitute 1, ½, 1/3, ¼, 1/10, or 1/100 results to zero
which will make you guess that the limit of the function is zero. But this is a wrong
guess. If you try to substitute smaller values of x in the function will give you varying
values between -1 and 1, as shown in the graph.

𝝅
The dashed lines near the y-axis indicate that the values of sin 𝒙 oscillate
between 1 and -1 infinitely often as x approaches 0. Since the values of f(x) do not
approach a fixed number as x approaches 0,
𝝅
𝐥𝐢𝐦 𝒔𝒊𝒏 does not exist.
𝑥→0 𝒙

𝒄𝒐𝒔 𝟓𝒙
5. Find the 𝐥𝐢𝐦 (𝒙𝟑 + 𝟏𝟎 𝟎𝟎𝟎)
𝑥→0

In this problem, it is more practical to use the Method 2.


𝑐𝑜𝑠 5𝑥 𝑐𝑜𝑠 0 𝟏
lim (𝑥 3 + 10 000) = (0 + 10 000) = 𝟏𝟎 𝟎𝟎𝟎
𝑥→0

2.1.2 One-Sided Limits

We write
𝐥𝐢𝐦 𝒇(𝒙) = L
𝒙→𝒂−

and say the left-hand limit of f(x) as x approaches a [or the limit of f(x) as
approaches a from the left] is equal to L if we can make the values of f(x) arbitrarily
close to L by taking x to be sufficiently close to a and x less than a.

Also, we write
𝐥𝐢𝐦 𝒇(𝒙) = L
𝒙→𝒂+

and say the right-hand limit of f(x) as x approaches a [or the limit of f(x) as
approaches a from the right] is equal to L if we can make the values of f(x) arbitrarily
close to L by taking x to be sufficiently close to a and x greater than a.
20

Therefore,
𝐥𝐢𝐦 𝒇(𝒙) = L
𝒙→𝒂
if and only if 𝐥𝐢𝐦− 𝒇(𝒙) = L and 𝐥𝐢𝐦+ 𝒇(𝒙) = L
𝒙→𝒂 𝒙→𝒂

ILLUSTRATIVE EXAMPLES

The graph of a function g is shown below.


Use it to state the values (if they exist) of the
following:
a. lim− 𝑔(𝑥)
𝑥→2
b. lim+ 𝑔(𝑥)
𝑥→2
c. lim 𝑔(𝑥)
𝑥→2
d. lim− 𝑔(𝑥)
𝑥→5
e. lim+ 𝑔(𝑥)
𝑥→5
f. lim 𝑔(𝑥)
𝑥→5

a. lim− 𝑔(𝑥)
𝑥→2

From the graph we see that the values of g(x)


approach 3 as x approaches 2 from the left.
𝐥𝐢𝐦− 𝒈(𝒙) = 3
𝒙→𝟐

b. lim+ 𝑔(𝑥)
𝑥→2

From the graph we see that the values of g(x)


approach 1 as x approaches 2 from the right.
𝐥𝐢𝐦+ 𝒈(𝒙) = 1
𝒙→𝟐

c. lim 𝑔(𝑥)
𝑥→2

Since the left and right limits are different, we conclude that lim 𝑔(𝑥) does not exist.
𝑥→2
21

d. lim− 𝑔(𝑥)
𝑥→5

From the graph we see that the values of


g(x) approach 2 as x approaches 5 from the
left.
𝐥𝐢𝐦− 𝒈(𝒙) = 2
𝒙→𝟓

e. lim+ 𝑔(𝑥)
𝑥→5

From the graph we see that the values of g(x) approach 2 as x approaches 5 from
the right.
𝐥𝐢𝐦+ 𝒈(𝒙) = 2
𝒙→𝟓
g. lim 𝑔(𝑥)
𝑥→5

This time the left and right limits are the same and so we have
𝐥𝐢𝐦 𝒈(𝒙) = 2
𝒙→𝟓

Despite this, notice that g(5) ≠ 2.

2.1.3 Infinite Limits

Let f be a function defined on


both sides of a, except possibly at a
itself. Then
𝐥𝐢𝐦 𝒇(𝒙) = ∞
𝒙→𝒂

means that the values of f(x) can be


made arbitrarily large (as large as we
please) by taking x sufficiently close to
a, but not equal to a. It is read as “the
limit of f(x), as x approaches a, is
infinity.”

This does not mean that we are


regarding ∞ as a number. Nor does it
mean that the limit exists. It simply
expresses the particular way in which
the limit does not exist.

Also, let f be a function defined


on both sides of a, except possibly at a
itself. Then
𝐥𝐢𝐦 𝒇(𝒙) = −∞
𝒙→𝒂

means that the values of f(x) can be


made arbitrarily large negative by taking x sufficiently close to a, but not equal to a.
It is read as “the limit of f(x), as x approaches a, is negative infinity.”
22

The line is called a vertical asymptote of the curve y = f(x) if at least one of
the following statements is true:

𝐥𝐢𝐦 𝒇(𝒙) = ∞ 𝐥𝐢𝐦 𝒇(𝒙) = ∞ 𝐥𝐢𝐦 𝒇(𝒙) = ∞


𝒙→𝒂 𝒙→𝒂+ 𝒙→𝒂−

𝐥𝐢𝐦 𝒇(𝒙) = −∞ 𝐥𝐢𝐦 𝒇(𝒙) = −∞ 𝐥𝐢𝐦 𝒇(𝒙) = −∞


𝒙→𝒂 𝒙→𝒂+ 𝒙→𝒂−

𝐥𝐢𝐦 𝒇(𝒙) = ∞ 𝐥𝐢𝐦 𝒇(𝒙) = ∞


𝒙→𝒂− 𝒙→𝒂+

𝐥𝐢𝐦 𝒇(𝒙) = −∞ 𝐥𝐢𝐦 𝒇(𝒙) = −∞


𝒙→𝒂− 𝒙→𝒂+

ILLUSTRATIVE EXAMPLE
2𝑥 2𝑥
1. Find lim+ and lim−
𝑥→3 𝑥−3 𝑥→3 𝑥−3

If x is close to 3 but larger than 3, then the denominator x – 3 is a small


positive number and 2x is close to 6. So the quotient 2x/(x-3) is a large positive
number. Thus, naturally, we see that
2𝑥
lim+ = ∞
𝑥→3 𝑥−3

If x is close to 3 but smaller than 3, then the denominator x – 3 is a small


negative number and 2x is still a positive number close to 6. So the quotient
2x/(x-3) is a large negative number. Thus,
2𝑥 2𝑥
lim− = −∞ 𝑦=
𝑥−3
𝑥→3 𝑥−3

Our intuition can be confirmed by the graph of


2𝑥
y= below. As you can see, the line x = 3 is the
𝑥−3
vertical asymptote.
x=3
23

2.1.4 Theorems on Limits

The theorems on the operation of limits of a function are:

1. The limit of the values of the variable “x” when the value of “x” approaches
“a” is simply equal to “a”.
lim 𝑥 = a
𝑥→𝑎

2. The limit of a constant “c” as the value of the variable “x” approaches “a”
remains “c”.
lim 𝑐 = c
𝑥→𝑎

3. If “c” is a constant and f(x) is a function, the limit of the product of the
constant and the function is equal to the product of the constant and the limit
of the function.
lim [𝑐 ∙ 𝑓(𝑥)] = c∙ lim 𝑓(𝑥)
𝑥→𝑎 𝑥→𝑎

4. The limit of the sum or difference of two or more functions is equal to the
sum of their limits.
lim [𝑢(𝑥) ± 𝑣(𝑥)] = lim 𝑢(𝑥) ± lim 𝑣(𝑥)
𝑥→𝑎 𝑥→𝑎 𝑥→𝑎

5. The limit of the product of two or more functions is equal to the product of
their limits.
lim [𝑢(𝑥) ∙ 𝑣(𝑥)] = lim 𝑢(𝑥) ∙ lim 𝑣(𝑥)
𝑥→𝑎 𝑥→𝑎 𝑥→𝑎

6. The limit of the quotient of two functions is equal to the quotient of their
limits, provided the limit of the denominator is not zero.
𝑢(𝑥) lim 𝑢(𝑥)
lim [𝑣(𝑥) ] = 𝑥→𝑎
𝑥→𝑎 lim 𝑣(𝑥)
𝑥→𝑎

7. The limit of the “nth” power of a function is equal to the “nth” power of the
limit of the function.
𝑛
lim [𝑓(𝑥)]𝑛 = [lim 𝑓(𝑥)]
𝑥→𝑎 𝑥→𝑎

8. The limit of the “nth” root of a function is equal to the “nth” root of the limit of
the function.
1/𝑛
lim [𝑓(𝑥)]1/𝑛 = [lim 𝑓(𝑥)] = 𝑛√ lim 𝑓(𝑥)
𝑥→𝑎 𝑥→𝑎 𝑥→𝑎

9. If “” is measured in radians,


lim [𝑠𝑖𝑛 𝛼 / 𝛼] = 1
𝑥→𝑎
24

10. The limit of the “nth” power of x is equal to the “nth” power of the limit of x.
lim 𝑥 𝑛 = 𝑎𝑛
𝑥→𝑎

11. The limit of the “nth” root of x is equal to the “nth” root of the limit of x.
𝑛
lim 𝑥 1/𝑛 = 𝑎1/𝑛 = √𝑎
𝑥→𝑎

ILLUSTRATIVE EXAMPLES

1. Find the 𝐥𝐢𝐦 (2x2 + 2x + 1)


𝒙→𝟏
= lim 2𝑥 2 + lim 2𝑥 + lim 1 law 4
𝑥→1 𝑥→1 𝑥→1
= 2(1)2 + 2(1) + 1 laws 1, 2, 3, 10
=5

𝒙𝟐 +𝒙−𝟏𝟐
2. Find the 𝐥𝐢𝐦
𝒙→𝟑 𝟐𝒙𝟐 −𝟕𝒙+𝟑
(𝑥+4)(𝑥−3)
= lim factoring
𝑥→3 (2𝑥−1)(𝑥−3)
(𝑥+4)
= lim cancel common factor
𝑥→3 (2𝑥−1)
lim (𝑥+4)
𝑥→3
= law 6
lim (2𝑥−1)
𝑥→3
3+4
=
2(3)−1
𝟕
=
𝟓

𝟏−𝒄𝒐𝒔 𝒙
3. Find the 𝐥𝐢𝐦
𝒙→𝟎 𝒔𝒊𝒏𝟐 𝒙
1−𝑐𝑜𝑠 𝑥
= lim trigonometric identity
𝑥→0 1−𝑐𝑜𝑠 2 𝑥
1−𝑐𝑜𝑠 𝑥
= lim special product
𝑥→0 (1+cos 𝑥)(1−cos 𝑥)
1
= lim cancel common factor
𝑥→0 (1+cos 𝑥)
lim 1
𝑥→0
= law 6
lim (1+cos 𝑥)
𝑥→0

1 1
= = cos 0 = 1
(1+cos 0) (1+1)
𝟏
=
𝟐
25

𝒔𝒊𝒏𝟐 𝜽
4. Find the 𝐥𝐢𝐦𝝅
𝜽→
𝟒
𝒕𝒂𝒏𝟑 𝜽
lim 𝑠𝑖𝑛2 𝜃
𝜃→𝜋⁄4
= law 6
lim 𝑡𝑎𝑛3 𝜃
𝜃→𝜋⁄4
2
( lim
𝜋
sin 𝜃)
𝜃→ ⁄4
= 3 law 7
( lim
𝜋
tan 𝜃)
𝜃→ ⁄4
(sin 𝜋⁄4)2
= sin 𝜋⁄4 = √2⁄2, tan 𝜋⁄4 = 1
(tan 𝜋⁄4)3
2
(√2⁄2) 𝟏
= =
(1)3 𝟐

𝒙𝟑 +𝒙𝟐 −𝟖𝒙−𝟏𝟐
5. Find the 𝐥𝐢𝐦
𝒙→𝟑 𝒙𝟑 −𝟒𝒙𝟐 −𝟑𝒙+𝟏𝟖
(𝑥−3)(𝑥 2 +4𝑥−4)
= lim factoring
𝑥→3 (𝑥−3)(𝑥 2 −𝑥−6)
(𝑥 2 +4𝑥−4)
= lim cancel common factor
𝑥→3 (𝑥 2 −𝑥−6)
lim (𝑥 2 +4𝑥−4)
𝑥→3
= law 6
lim (𝑥 2 −𝑥−6)
𝑥→3
25
= undefined limit
0

𝒕𝒂𝒏 𝜽
6. Find the 𝐥𝐢𝐦
𝜽→𝟎 𝒔𝒊𝒏 𝟐𝜽

In dealing with trigonometric functions in rational form, and you get zero in
the denominator, first, you need to convert the functions to its equivalent identity
before substituting value where θ is approaching.
𝑠𝑖𝑛 𝜃
cos 𝜃
= lim trigonometric identities
𝜃→0 2 𝑠𝑖𝑛 𝜃 cos 𝜃
𝑠𝑖𝑛 𝜃 1
= lim ( ∙ ) multiplying with reciprocal of the
𝜃→0 cos 𝜃 2 𝑠𝑖𝑛 𝜃 cos 𝜃
denominator
1
= lim ( ) cancel common factor
𝜃→0 2 cos2 𝜃
lim 1
𝜃→0
= law 6
lim (2 cos2 𝜃)
𝜃→0
1
= laws 2, 3
2 lim (cos2 𝜃)
𝜃→0
𝟏
= cos θ = 1
𝟐
26

𝒄𝒐𝒔 𝟐𝜽
7. Find the 𝐥𝐢𝐦
𝝅𝜽→ ⁄𝟐 𝒕𝒂𝒏 𝜽
2 𝑐𝑜𝑠 2 𝜃−1
= lim
𝜋 𝑠𝑖𝑛 𝜃 trigonometric identities
𝜃→ ⁄2
𝑐𝑜𝑠 𝜃
𝑐𝑜𝑠𝜃(2 𝑐𝑜𝑠 2 𝜃−1)
= lim
𝜋
multiplying with reciprocal of the
𝜃→ ⁄2 𝑠𝑖𝑛 𝜃
denominator
(2 𝑐𝑜𝑠 3 𝜃−cos 𝜃)
= lim
𝜋
𝜃→ ⁄2 𝑠𝑖𝑛 𝜃
lim (2 𝑐𝑜𝑠 3 𝜃−cos 𝜃)
𝜃→𝜋⁄2
= law 6
lim 𝑠𝑖𝑛 𝜃
𝜃→𝜋⁄2

(2 𝑐𝑜𝑠 3 𝜋⁄2−cos 𝜋⁄2)


= cos 𝜋⁄2 = 0
𝑠𝑖𝑛 𝜋⁄2
0
= =0
1

NOTE:
𝟎
= 0, limit is equal to zero
𝒏
𝒏
= undefined, which means no limit
𝟎

STUDENT ACTIVITY 2.1

I – Evaluate the limit and justify each step by indicating the appropriate Limit
Law(s).’]..m;

1. lim (3x4 + 2x2 – x + 1)


𝑥→−2
3
2. lim (1 + √𝑥)(2 – 6x2 + x3)
𝑥→8

3. lim (t2 + 1)3(t + 3)5


𝑡→−1

2𝑥 2 +1
4. lim √ 3𝑥−2
𝑥→2

5. lim √𝑢4 + 3𝑢 + 6
𝑢→−2

II – Evaluate the limit, if it exists.

𝑥 2 −6𝑥+5
1. lim 𝑥−5
𝑥→5
𝑥 2 −4𝑥
2. lim
𝑥→4 𝑥 2 −3𝑥−4

(2+ℎ)3 −8
3. lim
ℎ→0 ℎ
27

1 1
4. lim ( 𝑡 − 𝑡 2 +𝑡)
𝑡→0

√𝑥 2 +9 −5
5. lim
𝑥→−4 𝑥+4

𝑠𝑖𝑛2 𝑦
6. lim
𝑦→𝜋 1+cos 𝑦
sin 𝑎 sin 2𝑎
7. lim
𝑎→0 1−cos 𝑎
sin 𝑘𝑥
8. lim
𝑥→0 𝑥
sin 𝑎𝑥
9. lim
𝑥→0 tan 𝑏𝑥
2𝑦−𝜋
10. lim
𝜋𝑦→ ⁄2 cos 𝑦

LESSON 2.2: CONTINUITY

A function is said to be continuous at x = a if there is no interruption in the


graph of f (x) at a . Its graph is unbroken at a , and there is no hole, jump, or gap.

Functions that are continuous at every number in a given interval are


sometimes thought of as function whose graph can be sketched without lifting the
ballpen from the paper; that is, there is no break in the graph.

2.2.1 Continuity of a Function at a Point

A function is said to be continuous at a point x = a if the following three


conditions are satisfied;

I. f (x) is defined, that is, exists at x = a


II. lim f ( x) exists
x→a

III. lim f ( x) = f (a )
x →a

If one or more of these conditions are not satisfied, we say that the function
is discontinuous at x = a . The following graph is an example of a continuous
function.
y = f(x)

f(x)
approaches
f(a)

As x approaches a
28

ILLUSTRATIVE EXAMPLES

1. Discuss the continuity of f ( x) = 2 − x 3 at x = 1 .

Solution We apply the definition of continuity to prove that f ( x) = 2 − x 3 at x = 1 .


We have

i. f (1) = 2 − (1)3 = 1
ii. lim f (2 − x3 ) = 2 − (1)3 = 1
x→1

iii. lim f (2 − x 3 ) = f (1)


x→1

Thus, f ( x) = 2 − x 3 is continuous at x = 1 and its graph is as follows:

f(x) = 2 – x3

2.2.2 Continuity of a Function on an Open Interval

A function is said to be continuous on


an open interval (c, d ) if it is continuous at
each point in that interval. (If f is defined only
on one side of an endpoint of the interval,
we understand continuous at the endpoint
to mean continuous from the right or
continuous from the left.)

ILLUSTRATIVE EXAMPLES

1. Show that the function


𝑓(𝑥) = 1 − √1 − 𝑥 2
f(x) = 1 - √1 − 𝑥2 is continuous on the interval
[-1, 1].

Solution.
lim 𝑓(𝑥) = 1 - √1 − (−1)2 = 1 – 0 = 1
𝑥→−1+

lim 𝑓(𝑥) = 1 - √1 − 12 = 1 – 0 = 1
𝑥→1−
29

so f is continuous from the right at -1 and continuous from the left at 1. Therefore,
according to the definition, f is continuous on interval [-1, 1].

2.2.3 Theorems of Continuity

1. If f and g are continuous at a and c is a constant, then the following functions


are also continuous at a:

i. f+g
ii. f–g
iii. cf
iv. fg
v. f/g if g(a) ≠ 0

2. i. Any polynomial is continuous everywhere; that is, it is continuous on ℝ =


(−∞, ∞).

ii. Any rational function is continuous wherever it is defined; that is, it is


continuous on its domain.

3. The following types of functions are continuous at every number in their


domains:

polynomials rational functions

root functions trigonometric functions

4. If f is continuous at b and lim g(x) = b, then, lim f(g(x)) = f(b). In other words,
𝑥→𝑎 𝑥→𝑎
lim 𝑓(𝑔(𝑥)) = 𝑓 (lim g(x)).
𝑥→𝑎 𝑥→𝑎

5. If g is continuous at a and f is continuous at g(a), then the composite function f


∘ g given by (f ∘ g)(x) = 𝑓(g(x)) is continuous at a.
6. Intermediate Value Theorem: Suppose that f is continuous on the closed interval
[a, b] and let N be any number between f(a) and f(b), where f(a) ≠ f(b). Then there
exists a number c in (a, b) such that f(c) = N.

2.2.4 Removable Discontinuity

A function is said to have a removable discontinuity at x = a , if

i. lim f ( x) exists
x→a

ii. lim f ( x)  f (a)


x →a

Either because f (a) is undefined or the value if f (a) differs from the value of the
limit.
30

ILLUSTRATIVE EXAMPLE

x2 −1
1. Discuss the continuity of f ( x) =
x −1

Solution The function is undefined if the denominator x − 1 = 0 , that is if x = 1 . To


prove that this function is discontinuous at x = 1 , using the definition of continuity
(see figure 4)

x 2 − 1 (1) 2 − 1 1 − 1 0
i. f ( x) = = = = , it is indeterminate
x −1 (1) − 1 0 0
x2 − 1 ( x + 1)( x − 1)
ii. lim = lim = lim ( x + 1) = 1 + 1 = 2
x→1 x − 1 x→1 x −1 x→1

iii. lim f ( x)  f (1) , thus f (x) is discontinuous at x = 1


x→1

2.2.5 Jump Discontinuity

A function is said to have a jump discontinuity at x = a , if lim− f ( x)  lim+ f ( x)


x →a x →a

for a as any positive integer.

ILLUSTRATIVE EXAMPLE

The graph of a function g is shown. Discuss its continuity.

Solution. From the graph we see that the


values of g(x) approach 3 as x approaches 2
from the left. y = g(x)
𝐥𝐢𝐦− 𝒈(𝒙) = 3
𝒙→𝟐

Also, the values of g(x) approach 1 as x


approaches 2 from the right.
𝐥𝐢𝐦+ 𝒈(𝒙) = 1
𝒙→𝟐

Since 𝐥𝐢𝐦− 𝒈(𝒙) ≠ 𝐥𝐢𝐦+ 𝒈(𝒙), we conclude that there is a jump discontinuity
𝒙→𝟐 𝒙→𝟐
at x = 2.
31

STUDENT ACTIVITY 2.2

Graph and discuss the continuity of the following functions:

1. f ( x) = 3 x − 1 , x  1
1
2. f ( x) =
x +1
x3 − 1
3. f ( x) =
x −1
4. f ( x) = x 2 − 5x + 6 , x  1
x 2 − 7 x + 12
5. f ( x) =
x −3
x − 27
3
6. f ( x) = 2
x −9

Find all the numbers for which the following functions are continuous.
3
1. f(x) = 2x4 – √𝑥 + 1
√9−𝑥 2
2. f(x) = 4
𝑥 −16
3. f(x) = √(2 + 𝑥)(3 − 𝑥)
√𝑥
4. f(x) =
𝑥 2 −1

Find the discontinuities of the following functions.

|𝑥 2 −16|
1. f(x) =
𝑥 2 −16
1
2. f(x) =
𝑥 2 −16
𝑥 2 −𝑥−2
3. f(x) =
𝑥 2 −2𝑥
𝑥+2
4. f(x) =
𝑥 3 −8

SUMMARY

The limit of a function (L) is the value of a function when the value of the
independent variable (x) approaches a value “a” and is written as: 𝐥𝐢𝐦 𝒇(𝒙) = L
𝒙→𝒂

There are two ways in solving for the limit. First method, the standard way,
is by substituting all values near a in both sides to get f(a), and second is by direct
substitution property stating that if f is a polynomial or a rational function and a is
in the domain of f, then lim f(x) = f(a).
𝑥→𝑎
32

We write 𝐥𝐢𝐦− 𝒇(𝒙) = L as the limit of f(x) as approaches a from the left. Also,
𝒙→𝒂
we write 𝐥𝐢𝐦+ 𝒇(𝒙) = L or the limit of f(x) as approaches a from the right. Therefore,
𝒙→𝒂
𝐥𝐢𝐦 𝒇(𝒙) = L, if and only if 𝐥𝐢𝐦− 𝒇(𝒙) = L and 𝐥𝐢𝐦+ 𝒇(𝒙) = L.
𝒙→𝒂 𝒙→𝒂 𝒙→𝒂

The theorems on the operation of limits of a function are:

lim 𝑥 = a
𝑥→𝑎
lim 𝑐 = c
𝑥→𝑎
lim [𝑐 ∙ 𝑓(𝑥)] = c∙ lim 𝑓(𝑥)
𝑥→𝑎 𝑥→𝑎
lim [𝑢(𝑥) ± 𝑣(𝑥)] = lim 𝑢(𝑥) ± lim 𝑣(𝑥)
𝑥→𝑎 𝑥→𝑎 𝑥→𝑎
lim [𝑢(𝑥) ∙ 𝑣(𝑥)] = lim 𝑢(𝑥) ∙ lim 𝑣(𝑥)
𝑥→𝑎 𝑥→𝑎 𝑥→𝑎
𝑢(𝑥) lim 𝑢(𝑥)
lim [ ] = 𝑥→𝑎
𝑥→𝑎 𝑣(𝑥) lim 𝑣(𝑥)
𝑥→𝑎
𝑛
lim [𝑓(𝑥)]𝑛 = [lim 𝑓(𝑥)]
𝑥→𝑎 𝑥→𝑎
1/𝑛
lim [𝑓(𝑥)]1/𝑛 = [lim 𝑓(𝑥)] = 𝑛√ lim 𝑓(𝑥)
𝑥→𝑎 𝑥→𝑎 𝑥→𝑎

lim [𝑠𝑖𝑛 𝛼 / 𝛼] = 1
𝑥→𝑎
lim 𝑥 𝑛 = 𝑎𝑛
𝑥→𝑎
𝑛
lim 𝑥 1/𝑛 = 𝑎1/𝑛 = √𝑎
𝑥→𝑎

A function is said to be continuous at x = a if there is no interruption in the


graph of f (x) at a . Its graph is unbroken at a , and there is no hole, jump, or gap.
A function is said to be continuous at a point x = a if the following three
conditions are satisfied: f (x) is defined, that is, exists at x = a ; lim f ( x) exists and
x→a

lim f ( x) = f (a ) .
x →a

A function is said to be continuous on an open interval (c, d ) if it is continuous


at each point in that interval.
A function is said to have a removable discontinuity at x = a , if: lim f ( x)
x→a

exists and lim f ( x)  f (a) .


x →a

A function is said to have a jump discontinuity at x = a , if lim− f ( x)  lim+ f ( x)


x →a x →a

for a as any positive integer.

REFERENCES

Love, Clyde E., Differential and Integral Calculus, The Macmillan


Company

Stewart, James, Single Variable Calculus , Brooks/Cole CENGAGE


Learning, 7th Edition, 2012

Swokowski, Earl W., Calculus with Analytic Geometry, 2nd Edition,


Prindle, Weber & Schmid t, 1979
33
OVERVIEW

MODULE This module will start with the discussion of how


tangent problem had given rise to an important branch of
calculus, the Differential Calculus. In this module,
derivative is defined and related to slope and rate of
change.

LEARNING OUTCOMES

At the end of this module, you should be able to:


1. present solutions in finding the derivative of a function.
2. solve for the slope of the tangent line in a given point of a function.
3. compute for the rate of change of a function.

LESSON 3.1: THE DERIVATIVE

3.1.1 The Tangent Problem

Consider the problem of trying to find an equation of the tangent line to a


curve with equation at a given point P. Since we know that the point P lies on the
tangent line, we can find the equation of t if we know its slope m. The problem is
that we need two points to compute the slope and we know only one point, P, on
t.

y = f(x)

Figure 1

To get around the problem we first find an approximation to m by taking a


nearby point Q on the curve and computing the slope mPQ of the secant line PQ.

Q(x, f(x))
P(a, f(a)) f(x) – f(a)

x–a

<<<<

Figure 2
34

Figure 3

Now imagine that moves along the curve toward as in Figure 3. You can
see that the secant line rotates and approaches the tangent line as its limiting
position. This means that the slope of the secant line becomes closer and closer
to the slope of the tangent line. We write

and we say that m is the limit of mPQ as Q approaches P along the curve. Since x
approaches a as Q approaches P, we could write

The tangent problem has given rise to the branch of calculus called
differential calculus, which was not invented until more than 2000 years after
integral calculus. The main ideas behind differential calculus are due to the French
mathematician Pierre Fermat (1601–1665) and were developed by the English
mathematicians John Wallis (1616–1703), Isaac Barrow (1630–1677), and Isaac
Newton (1642–1727) and the German mathematician Gottfried Leibniz (1646–
1716).
35
3.1.2 Definition
y
B

P’

∆y
T

A P

O x ∆x x

Fundamental Definition. The derivative of y with respect to x is the limit of the ratio
Δ𝑦
when Δ𝑥 approaches zero.
Δ𝑥
𝒅𝒚
The derivative is designated by the symbol :
𝒅𝒙

𝒅𝒚 ∆𝒚 [𝒇(𝒙 + ∆𝒙) − 𝒇(𝒙)]


= 𝐥𝐢𝐦 = 𝐥𝐢𝐦 [ ]
𝒅𝒙 ∆𝒙→𝟎 ∆𝒙 ∆𝒙→𝟎 ∆𝒙

The operation in finding the derivative is called differentiation. There are


three fundamental requirements of the calculus. Each function to be studied must,
for some range of values of the independent variable, be continuous, one-valued,
and differentiable.

3.1.3 Determination of the Derivative

To obtain the derivative of any function, the general process is as follows:

1. Replace x by (x + Δ𝑥), and y by (y + Δ𝑦):


y = f(x)
y + Δ𝑦 = f(x + Δ𝑥)

2. By subtraction, eliminate y between (1) and (2), thus obtaining a formula for Δ𝑦
in terms of x and Δ𝑥:
Δ𝑦 = f(x + Δ𝑥) – f(x)

3. Divide through by Δ𝑥:


Δ𝑦 𝑓(𝑥 + Δ𝑥) − 𝑓(𝑥)
=
Δ𝑥 Δ𝑥

4. Determine the limit as Δ𝑥 approaches zero.


36
ILLUSTRATIVE PROBLEMS

Differentiate the following functions:

1. 𝒚 = 𝟒𝒙𝟐 − 𝟑𝒙 − 𝟐

Step 1 𝑦 + 𝛥𝑦 = 4(𝑥 + 𝛥𝑥)2 − 3(𝑥 + 𝛥𝑥) − 2


𝑦 + 𝛥𝑦 = 4(𝑥 2 + 2𝑥𝛥𝑥 + 𝛥𝑥 2 ) − 3𝑥 − 3𝛥𝑥 − 2
𝑦 + 𝛥𝑦 = 4𝑥 2 + 8𝑥𝛥𝑥 + 4𝛥𝑥 2 − 3𝑥 − 3𝛥𝑥 − 2
Step 2 𝑦 + 𝛥𝑦 − 𝑦 = 4𝑥 2 + 8𝑥𝛥𝑥 + 4𝛥𝑥 2 − 3𝑥 − 3𝛥𝑥 − 2 − (4𝑥 2 − 3𝑥 − 2)
𝛥𝑦 = 4𝑥 2 + 8𝑥𝛥𝑥 + 4𝛥𝑥 2 − 3𝑥 − 3𝛥𝑥 − 2 − 4𝑥 2 + 3𝑥 + 2
𝛥𝑦 = 8𝑥𝛥𝑥 + 4𝛥𝑥 2 − 3𝛥𝑥
𝛥𝑦 8𝑥𝛥𝑥 + 4𝛥𝑥 2 − 3𝛥𝑥
Step 3 =
𝛥𝑥 𝛥𝑥
𝛥𝑦
= 8𝑥 + 4𝛥𝑥 − 3
𝛥𝑥
𝛥𝑦
Step 4 𝐿𝑖𝑚 = 𝐿𝑖𝑚 (8𝑥 + 4𝛥𝑥 − 3)
𝛥𝑥→0 𝛥𝑥 𝛥𝑥→0
𝛥𝑦
𝐿𝑖𝑚 = 8𝑥 − 3
𝛥𝑥→0 𝛥𝑥
𝒅𝒚
Answer 𝒇′(𝒙) = 𝒚′ = = 𝟖𝒙 − 𝟑
𝒅𝒙

2. 𝒚 = 𝟑 + 𝟒𝒙 − 𝒙𝟐

Step 1 𝑦 + 𝛥𝑦 = 3 + 4(𝑥 + 𝛥𝑥) − (𝑥 + 𝛥𝑥)2


𝑦 + 𝛥𝑦 = 3 + 4𝑥 + 4𝛥𝑥 − (𝑥 2 + 2𝑥𝛥𝑥 + 𝛥𝑥 2 )
𝑦 + 𝛥𝑦 = 3 + 4𝑥 + 4𝛥𝑥 − 𝑥 2 − 2𝑥𝛥𝑥 − 𝛥𝑥 2
Step 2 𝑦 + 𝛥𝑦 − 𝑦 = 3 + 4𝑥 + 4𝛥𝑥 − 𝑥 2 − 2𝑥𝛥𝑥 − 𝛥𝑥 2 − (3 + 4𝑥 − 𝑥 2 )
𝛥𝑦 = 3 + 4𝑥 + 4𝛥𝑥 − 𝑥 2 − 2𝑥𝛥𝑥 − 𝛥𝑥 2 − 3 − 4𝑥 + 𝑥 2
𝛥𝑦 = 4𝛥𝑥 − 2𝑥𝛥𝑥 − 𝛥𝑥 2
Step 3 𝛥𝑦 4𝛥𝑥 − 2𝑥𝛥𝑥 − 𝛥𝑥 2
=
𝛥𝑥 𝛥𝑥
𝛥𝑦
= 4 − 2𝑥 − 𝛥𝑥
𝛥𝑥
𝛥𝑦
Step 4 𝐿𝑖𝑚 = 𝐿𝑖𝑚 (4 − 2𝑥 − 𝛥𝑥)
𝛥𝑥→0 𝛥𝑥 𝛥𝑥→0
𝛥𝑦
𝐿𝑖𝑚 = 4 − 2𝑥
𝛥𝑥→0 𝛥𝑥
𝒅𝒚
Answer 𝒇′(𝒙) = 𝒚′ = = 𝟒 − 𝟐𝒙
𝒅𝒙

3. 𝒚 = √𝒙

Step 1 𝑦 = √𝑥
𝑦 + 𝛥𝑦 = √𝑥 + 𝛥𝑥
Step 2 𝑦 + 𝛥𝑦 − 𝑦 = √𝑥 + 𝛥𝑥 − √𝑥
𝛥𝑦 = √𝑥 + 𝛥𝑥 − √𝑥
37

𝛥𝑦 √𝑥 + 𝛥𝑥 − √𝑥
Step 3 =
𝛥𝑥 𝛥𝑥
𝛥𝑦 √𝑥 + 𝛥𝑥 − √𝑥 √𝑥 + 𝛥𝑥 + √𝑥
= •
𝛥𝑥 𝛥𝑥 √𝑥 + 𝛥𝑥 + √𝑥
2 2
𝛥𝑦 (√𝑥 + 𝛥𝑥) − (√𝑥) 𝑥 + 𝛥𝑥 − 𝑥
= =
𝛥𝑥 𝛥𝑥(√𝑥 + 𝛥𝑥 + √𝑥) 𝛥𝑥(√𝑥 + 𝛥𝑥 + √𝑥)
𝛥𝑦 𝛥𝑥 1
= =
𝛥𝑥 𝛥𝑥(√𝑥 + 𝛥𝑥 + √𝑥) √𝑥 + 𝛥𝑥 + √𝑥
𝛥𝑦 1 1
Step 4 𝐿𝑖𝑚 = 𝐿𝑖𝑚 ( )=
𝛥𝑥→0 𝛥𝑥 𝛥𝑥→0 √𝑥 + 𝛥𝑥 + √𝑥 √𝑥 + 0 + √𝑥
𝛥𝑦 1
𝐿𝑖𝑚 =
𝛥𝑥→0 𝛥𝑥 2√𝑥
𝒅𝒚 𝟏
Answer 𝒇′(𝒙) = 𝒚′ = =
𝒅𝒙 𝟐√𝒙

STUDENT ACTIVITY 3.1

Find the derivative of the following functions:

1. y = x2 + 2x + 1
1
2. w = 𝑧−1
3. y = √𝑥 + 2
4. y = √𝑎2 + 𝑥 2
5. u = 𝑡 3⁄2
𝑧+1
6. w = 𝑧 − 1
7. 𝑦 = 𝑥 2 − 5𝑥
8. 𝑦 = 1 + 3𝑥 − 𝑥 2
9. 𝑦 = 𝑥 3 (2𝑥 2 − 1)
10. 𝑦 = (𝑥 − 1)(2𝑥 2 + 3)

3.1.4 Derivative Interpreted as Slope

The derivative of a function is identical with the slope of the graph of the
function. y
B

P’

∆y
T

A P
y = mx + b
y

O x ∆x x
38
Let us verify that the term slope of a curve does not conflict with the
previously used term slope of a line. The equation of a line of slope m can be
written
y = mx + b
y + ∆𝑦 = m(x + ∆𝑥) + b
∆𝑦 = mx + m∆𝑥 + b – mx – b
∆𝑦 = m∆𝑥
𝚫𝒚
=m
𝚫𝒙

ILLUSTRATIVE EXAMPLES

Find the slope of the curves at the given point.

1. y = 2 + x2 at point (3, -2).


y = 2 + x2
y + ∆𝑦 = 2 + (x + ∆𝑥)2
∆𝑦 = 2 + (x2 + 2x∆𝑥 + ∆𝑥 2) - y
∆𝑦 = 2 + (x2 + 2x∆𝑥 + ∆𝑥 2) – (2 + x2)
∆𝑦 = 2x∆𝑥 + ∆𝑥 2
Δ𝑦
= 2x + ∆𝑥
Δ𝑥
𝛥𝑦
𝐿𝑖𝑚 = 𝐿𝑖𝑚 (2𝑥 + 𝛥𝑥)
𝛥𝑥→0 𝛥𝑥 𝛥𝑥→0
𝛥𝑦
𝐿𝑖𝑚 = 2𝑥
𝛥𝑥→0 𝛥𝑥
𝛥𝑦
= 𝑚 = 2𝑥
𝛥𝑥
Therefore, by substitution x = 3, slope at point (3, -2) is
𝑚 = 2(3)
m=6

2. y = 3x2 – 2x at point (2, 8).


y = 3x2 – 2x
y + ∆𝑦 = 3(x + ∆𝑥)2 - 2(x + ∆𝑥)
∆𝑦 = 3(x2 + 2x∆𝑥 + ∆𝑥 2) – 2(x + ∆𝑥) – y
∆𝑦 = 3(x2 + 2x∆𝑥 + ∆𝑥 2) – 2(x + ∆𝑥) – (3x2 – 2x)
∆𝑦 = 3x2 + 6x∆𝑥 + 3∆𝑥 2 – 2x – 2∆𝑥 – 3x2 + 2x
∆𝑦 = 6x∆𝑥 + 3∆𝑥 2 – 2∆𝑥
Δ𝑦
= 6x + 3∆𝑥 – 2
Δ𝑥
𝛥𝑦
𝐿𝑖𝑚 = 𝐿𝑖𝑚 (6𝑥 + 3∆𝑥 – 2)
𝛥𝑥→0 𝛥𝑥 𝛥𝑥→0
𝛥𝑦
𝐿𝑖𝑚 = 6𝑥 − 2
𝛥𝑥→0 𝛥𝑥
𝛥𝑦
= 𝑚 = 6𝑥 − 2
𝛥𝑥
39
Therefore, by substitution x = 2, slope at point (2, 8) is
𝑚 = 6(2) − 2
m = 10

𝟏
3. y = at point (-2, -1).
𝒙+𝟏
1
y =
𝑥+1
1
y + ∆𝑦 =
(𝑥+∆𝑥)+1
1
∆𝑦 = –y
𝑥+∆𝑥+1
1 1
∆𝑦 = –
𝑥+∆𝑥+1 𝑥+1
(𝑥+1) − (𝑥+∆𝑥+1)
∆𝑦 =
(𝑥+∆𝑥+1)(𝑥+1)
−∆𝑥
∆𝑦 =
(𝑥+∆𝑥+1)(𝑥+1)
Δ𝑦 1
=
Δ𝑥 (𝑥+∆𝑥+1)(𝑥+1)
𝛥𝑦 −1
𝐿𝑖𝑚 = 𝐿𝑖𝑚 ( )
𝛥𝑥→0 𝛥𝑥 𝛥𝑥→0 (𝑥 + ∆𝑥 + 1)(𝑥 + 1)
𝛥𝑦 1
𝐿𝑖𝑚 =−
𝛥𝑥→0 𝛥𝑥 (𝑥 + 1)(𝑥 + 1)
𝛥𝑦 1
=𝑚=−
𝛥𝑥 (𝑥 + 1)2

Therefore, by substitution x = -2, slope at point (-2, -1) is


1
𝑚 = − (−2+1)2
m=-1

4. y2 = 4x at point (1, 2).


y2 = 4x
(y + ∆𝑦)2 = 4(x + ∆𝑥)
y2 + 2y∆𝑦 + ∆𝑦2 = 4(x + ∆𝑥)
y2 + 2y∆𝑦 + ∆𝑦2 = 4x + 4∆𝑥
2y∆𝑦 + ∆𝑦2 = 4x + 4∆𝑥 - y2
2√4𝑥 ∆𝑦 + ∆𝑦2 = 4x + 4∆𝑥 - 4x
2√4𝑥 ∆𝑦 + ∆𝑦2 = 4∆𝑥
Δ𝑦 ∆𝑦2
2√4𝑥 + =4
Δ𝑥 ∆𝑥
∆𝑦2
Δ𝑦 4−
∆𝑥
=
Δ𝑥 2√4𝑥
40

∆𝑦 2
𝛥𝑦 4 −
𝐿𝑖𝑚 = 𝐿𝑖𝑚 ( ∆𝑥 )
𝛥𝑥→0 𝛥𝑥 𝛥𝑥→0 2√4𝑥
𝛥𝑦 4
𝐿𝑖𝑚 =
𝛥𝑥→0 𝛥𝑥 2√4𝑥
𝛥𝑦 4
=𝑚=
𝛥𝑥 2√4𝑥
Therefore, by substitution x = 1, slope at point (1, 2) is
4
𝑚=
2√4(1)

m=𝟏

5. y2 = 3x + 1 at point (1, 2).


y2 = 3x + 1
(y + ∆𝑦)2 = 3(x + ∆𝑥) + 1
y2 + 2y∆𝑦 + ∆𝑦2 = 3(x + ∆𝑥) + 1
y2 + 2y∆𝑦 + ∆𝑦2 = 3x + 3∆𝑥 + 1
2y∆𝑦 + ∆𝑦2 = 3x + 3∆𝑥 - y2
2√3𝑥 + 1 ∆𝑦 + ∆𝑦2 = 3x + 3∆𝑥 – (3x + 1)
2√3𝑥 + 1 ∆𝑦 + ∆𝑦2 = 3∆𝑥 – 1
2
Δ𝑦 Δ𝑦 1
2√3𝑥 + 1 Δ𝑥 + =3-
Δ𝑥 Δ𝑥
1 ∆𝑦2
Δ𝑦 3 − ∆𝑥− ∆𝑥
=
Δ𝑥 2√3𝑥+1
2
∆𝑦
𝛥𝑦 3 − 1 −
𝐿𝑖𝑚 = 𝐿𝑖𝑚 ∆𝑥 ∆𝑥
𝛥𝑥→0 𝛥𝑥 𝛥𝑥→0
2√3𝑥 + 1
( )
𝛥𝑦 3
𝐿𝑖𝑚 =
𝛥𝑥→0 𝛥𝑥 2√3𝑥 + 1
𝛥𝑦 3
=𝑚=
𝛥𝑥 2√3𝑥 + 1
Therefore, by substitution x = 1, slope at point (1, 2) is
3
𝑚=
2√3(1)+1
𝟑
m=
𝟒
41
3.1.5 Rate of Change
Δ𝑦
The ratio Δ𝑥 is called the average rate of change over the interval Δ𝑥. If we
let Δ𝑥 approach zero, this ratio in general approaches a limiting value, which is
defined as the rate of change of y corresponding to the given value of x, or the
instantaneous rate of change of y with respect to x:
𝑑𝑦 Δ𝑦
= lim
𝑑𝑥 Δ𝑥 →0 Δ𝑥

Fundamental Definition. The derivative of a function is identical with its rate of


change.

ILLUSTRATIVE EXAMPLES

1. Find how fast (a) the circumference, (b) the area, of a circle increases
when the radius increases.

Solutions:

a. Circumference of a circle is equal to the product of twice the product of pi


𝑑𝐶
(π) and the radius. Let be the rate of change of the circumference as the
𝑑𝑟
radius changes.

C=2πr
C + ∆𝐶 = 2 π (r + ∆𝑟)
∆𝐶 = 2 π (r + ∆𝑟) – C
∆𝐶 = 2 π (r + ∆𝑟) – (2 π r)
∆𝐶 = 2 π r + 2 π ∆𝑟 – 2 π r
Δ𝐶
=2π
Δ𝑟
𝛥𝐶
𝐿𝑖𝑚 = 𝐿𝑖𝑚 (2 π)
𝛥𝑥→0 𝛥𝑟 𝛥𝑥→0
𝛥𝑦
𝐿𝑖𝑚 = 2π
𝛥𝑥→0 𝛥𝑥
𝜟𝒚 𝒅𝒚
= = 𝟐𝛑
𝜟𝒙 𝒅𝒙

b. Area of a circle is equal to the product of pi (π) and the square of the radius.
𝑑𝐴
Let be the rate of change of the area as the radius changes.
𝑑𝑟

A = π r2
A + ∆𝐴 = π (r + ∆𝑟)2
∆𝐴 = π (r2 + 2r∆𝑟 + ∆𝑟2) – A
∆𝐴 = π (r2 + 2r∆𝑟 + ∆𝑟2) – π r2
∆𝐴 = π r2 + 2πr∆𝑟 + 𝜋∆𝑟2 – π r2
Δ𝐴
= 2πr + 𝜋∆𝑟
Δ𝑟
42
𝛥𝐴
𝐿𝑖𝑚 = 𝐿𝑖𝑚 (2πr + π∆𝑟)
𝛥𝑥→0 𝛥𝑟 𝛥𝑥→0
𝛥𝑦
𝐿𝑖𝑚 = 2πr
𝛥𝑥→0 𝛥𝑥
𝜟𝒚 𝒅𝒚
= = 𝟐𝛑𝐫
𝜟𝒙 𝒅𝒙

STUDENT ACTIVITY 3.2

I – Find an equation of the tangent line to the curve at the given point.

1. y = 4x – 3x2 (2, -4)


2. y = x3 – 3x + 1 (2, 3)
3. y = √𝑥 (1, 1)
2𝑥+1
4. y = (1, 1)
𝑥+2
5. y2 = 3x + 1 (1, 2)

II – Solve the following rates of change.


1. Find how fast (a) the volume, (b) the surface area, (c) the diagonal, of a cube
increases when the length of the edge increases.
2. The radius of a sphere, initially zero, increases at the rate of 6 ft. per sec. Find
how fast the volume is increasing after ¼ sec.
3. A right circular cylinder has a fixed height of 6 units. Find the rate of change of
its volume V with respect to the radius r of its base.
4. In item no. 3, find the rate of change of the total surface area A with respect to
r.
5. The dimensions of a box are b, b + 1, b + 4. Find how fast the total surface
area A increases as b increases.

SUMMARY

Δ𝑦
The derivative of y with respect to x is the limit of the ratio when Δ𝑥
Δ𝑥
approaches zero.
𝑑𝑦 ∆𝑦 [𝑓(𝑥 + ∆𝑥) − 𝑓(𝑥)]
= lim = lim [ ]
𝑑𝑥 ∆𝑥→0 ∆𝑥 ∆𝑥→0 ∆𝑥

The operation in finding the derivative is called differentiation. Each


function to be studied must, for some range of values of the independent variable,
be continuous, one-valued, and differentiable.

To obtain the derivative of any function, follow the general procedure:


1. Replace x by (x + Δ𝑥), and y by (y + Δ𝑦).
2. By subtraction, eliminate y, thus obtaining a formula for Δ𝑦 in terms of x and
Δ𝑥.
3. Divide through by Δ𝑥.
43

4. Determine the limit as Δ𝑥 approaches zero.

The derivative of a function is identical with the slope of the graph of the
function. The equation of a line of slope m can be written
Δ𝑦
=m
Δ𝑥

The derivative of a function is identical with its rate of change. The


instantaneous rate of change of y with respect to x is expressed as:
𝑑𝑦 Δ𝑦
= lim
𝑑𝑥 Δ𝑥 →0 Δ𝑥

REFERENCES

Love, Clyde E., Differential and Integral Calculus, The Macmillan Company

Stewart, James, Single Variable Calculus, Brooks/Cole CENGAGE Learning, 7th


Edition, 2012

Swokowski, Earl W., Calculus with Analytic Geometry, 2nd Edition, Prindle, Weber
& Schmidt, 1979
44
OVERVIEW

MODULE As a continuation of the previous chapter, this


module will provide derived equations of finding
the derivatives of the algebraic functions backed
up with theorems and proofs. Chain rule,
derivatives of implicit functions and finding higher
derivatives will also be discussed in this module.

LEARNING OUTCOMES

At the end of this module, you should be able to:


4. familiarize the derived formulas in getting derivatives of different algebraic
functions,
5. solve different problems using these formulas,
6. apply chain rule in solvi
7. find the derivatives of implicit functions, and
8. get the higher derivatives of a function.

LESSON 4.1: GENERAL THEOREMS OF DIFFERENTIATION

4.1.1 Theorem 1: Derivative of a Constant

The derivative of a constant function is zero.

Proof:

let 𝑦 = 𝑐 ∆𝑦
=0
𝑦 + ∆𝑦 = 𝑐 ∆𝑥
𝑦 + ∆𝑦 − 𝑦 = 𝑐 − 𝑦 ∆𝑦
lim = lim (0)
∆𝑦 = 𝑐 − 𝑐 ∆𝑥→0 ∆𝑥 ∆𝑥→0
∆𝑦 = 0
∆𝑦 0 𝑑𝑦
= =0
∆𝑥 ∆𝑥 𝑑𝑥

𝒅𝒚
(c) = 0
𝒅𝒙

ILLUSTRATIVE EXAMPLES

1. Find the derivative of 𝒚 = 𝟖 + 𝝅

Solution:
𝑦 = 8 + 𝜋 is a constant function, hence the derivative is equal to zero. That is,
𝑑
(8 + 𝜋) = 0 + 0 = 𝟎
𝑑𝑥
45
2. Find the derivative of 𝒚 = 1,000,000

Solution:
𝑦 = 1,000,000 is a constant function, hence the derivative is equal to zero. That
is,
𝑑
(1,000,000) = 𝟎
𝑑𝑥

3. Find the derivative of 𝒚 = e + 5

Solution:
𝑦 = 𝑒 + 5 is a constant function, where e is the exponential number equal to
2.7182818285…, hence the derivative is equal to zero. That is,
𝑑
(𝑒 + 5) = 0 + 0 = 𝟎
𝑑𝑥

4.1.2 Theorem 2: Power Rule

The derivative of the nth term power of a variable is the product of 𝑛 and the
(𝑛 − 1)𝑠𝑡 power of the variable. Or, to differentiate 𝑥 to a positive integer power,
take the power and multiply to the next lower integer.

𝒅 𝒏
(𝒙 ) = 𝒏𝒙𝒏−𝟏
𝒅𝒙
Proof:

Let 𝑦 = 𝑥 𝑛
𝑦 + ∆𝑦 = (𝑥 + ∆𝑥)𝑛
𝑦 + ∆𝑦 − 𝑦 = (𝑥 + ∆𝑥)𝑛 − 𝑦
∆𝑦 = (𝑥 + ∆𝑥)𝑛 − 𝑥 𝑛
𝑥+∆𝑥 𝑛
∆𝑦 = 𝑥 𝑛 ( ) − 𝑥𝑛
𝑥

∆𝑥 𝑛
∆𝑦 = 𝑥 𝑛 (1 + ) − 𝑥𝑛
𝑥

∆𝑥 𝑛
∆𝑦 = 𝑥 𝑛 [(1 + ) − 1]
𝑥

By using the expansion of binomial series,


∆𝑥 𝑛(𝑛 − 1) ∆𝑥 2
∆𝑦 = 𝑥 𝑛 [1 + 𝑛 𝑥 + 2!
( ) ]
𝑥
+⋯− 1
∆𝑥 𝑛(𝑛−1) ∆𝑥 2
= 𝑥 𝑛 [𝑛 𝑥 + 2!
(𝑥) ]
+⋯
∆𝑥 𝑛(𝑛−1) ∆𝑥
= 𝑥 𝑛 ( ) [𝑛 + ( ) + ⋯]
𝑥 2! 𝑥
𝑛(𝑛−1) ∆𝑥
= 𝑥 𝑛−1 ∆𝑥 [𝑛 + (𝑥 ) +⋯]
2!
46
𝑛(𝑛−1) ∆𝑥
∆𝑦 𝑥 𝑛−1 ∆𝑥[𝑛+ ( 𝑥 )+⋯ ]
2!
=
∆𝑥 ∆𝑥
∆𝑦 𝑛(𝑛−1) ∆𝑥
= 𝑥 𝑛−1 [𝑛 + ( )+ ⋯]
∆𝑥 2! 𝑥

∆𝑦 𝑛(𝑛 − 1) ∆𝑥
lim = lim {𝑥 𝑛−1 [𝑛 + 2!
( )]}
𝑥
∆𝑥→0 ∆𝑥 ∆𝑥→0
+⋯
𝑑𝑦
= 𝑥 𝑛−1 [𝑛 + 0 + 0 + ⋯ ]
𝑑𝑥
𝒅𝒚 𝒏
(𝒙 ) = 𝒏𝒙𝒏−𝟏
𝒅𝒙

ILLUSTRATIVE EXAMPLES
𝑑𝑦
1. Find 𝑑𝑥 of functions

a. 𝑦 = 𝑥 b. 𝑦 = 𝑥 4
Solutions:
Using the Power Rule, we get
𝑑
a. (𝑥) = (1)𝑥 1=1 = 𝑥 0 = 𝟏
𝑑𝑥
𝑑
b. (𝑥 4 ) = (4)𝑥 4=1 = 𝟒𝒙𝟑
𝑑𝑥

4.1.3 Theorem 3: Constant Multiple Rule


If 'c' is any constant, then
𝒅
[𝒄 ∙ 𝒇(𝒙)] = 𝒄 ∙ 𝒇′ (𝒙)
𝒅𝒙
using the prime notation (‘) for d/dx

Proof:

Let 𝑦 = 𝑐𝑓(𝑥)
𝑦 + ∆𝑦 = 𝑐𝑓(𝑥 + ∆𝑥)
𝑦 + ∆𝑦 − 𝑦 = 𝑐𝑓(𝑥 + ∆𝑥) − 𝑦
𝑦 + ∆𝑦 − 𝑦 = 𝑐𝑓(𝑥 + ∆𝑥) − 𝑐𝑓(𝑥)
∆𝑦 = 𝑐𝑓(𝑥 + ∆𝑥) − 𝑐𝑓(𝑥)
∆𝑦 = 𝑐[𝑓(𝑥 + ∆𝑥) − 𝑓(𝑥)]
∆𝑦 𝑐[𝑓(𝑥 + ∆𝑥) − 𝑓(𝑥)]
=
∆𝑥 ∆𝑥
∆𝑦 𝑐 [𝑓(𝑥 + ∆𝑥) − 𝑓(𝑥)]
lim = lim [ ]
∆𝑥→0 ∆𝑥 ∆𝑥→0 ∆𝑥
∆𝑦 𝑐[𝑓(𝑥 + ∆𝑥) − 𝑓(𝑥)]
lim = lim [ ]
∆𝑥→0 ∆𝑥 ∆𝑥→0 ∆𝑥
47
∆𝑦 [𝑓(𝑥 + ∆𝑥) − 𝑓(𝑥)]
lim = 𝑐 lim [ ]
∆𝑥→0 ∆𝑥 ∆𝑥→0 ∆𝑥
𝒅𝒚
(𝒄 ∙ 𝒇(𝒙)) = 𝒄 ∙ 𝒇′ (𝒙)
𝒅𝒙

ILLUSTRATIVE EXAMPES
𝑑𝑦
1. Find 𝑑𝑥 of functions
2
a. 𝑦 = 3𝑥 6 b. 𝑦 = − 3 𝑥 4

Solution: The Constant is not affected by differentiation.


𝑑 𝑑
a. y = 3x 6  𝑑𝑥 (3𝑥 6 ) = 3 [𝑑𝑥 (𝑥 6 )] = 3(6𝑥 6−1 ) = 𝟏𝟖𝒙𝟓

2 𝑑 2 2 𝑑 2 𝟖
b. y = − x 4  𝑑𝑥 (− 3 𝑥 4 ) = − 3 [𝑑𝑥 (𝑥 4 )] = − 3 (4𝑥 4−1 ) = − 𝟑 𝒙𝟑
3

4.1.4 Theorem 4: Derivative of Sum of Functions


The derivative of sum of functions is the sum of their derivatives.
Proof:
Let 𝑦 = 𝑢(𝑥) + 𝑣(𝑥)
𝑦 + ∆𝑦 = 𝑢(𝑥 + ∆𝑥) + 𝑣(𝑥 + ∆𝑥)
𝑦 + ∆𝑦 − 𝑦 = 𝑢(𝑥 + ∆𝑥) + 𝑣(𝑥 + ∆𝑥) − 𝑦
𝑦 + ∆𝑦 − 𝑦 = 𝑢(𝑥 + ∆𝑥) + 𝑣(𝑥 + ∆𝑥) − 𝑢(𝑥) − 𝑣(𝑥)
∆𝑦 = 𝑢(𝑥 + ∆𝑥) + 𝑣(𝑥 + ∆𝑥) − 𝑢(𝑥) − 𝑣(𝑥)
∆𝑦 = 𝑢(𝑥 + ∆𝑥) − 𝑢(𝑥) + 𝑣(𝑥 + ∆𝑥) − 𝑣(𝑥)
∆𝑦 𝑢(𝑥 + ∆𝑥) − 𝑢(𝑥) + 𝑣(𝑥 + ∆𝑥) − 𝑣(𝑥)
=
∆𝑥 ∆𝑥
∆𝑦 𝑢(𝑥 + ∆𝑥) − 𝑢(𝑥) 𝑣(𝑥 + ∆𝑥) − 𝑣(𝑥)
= +
∆𝑥 ∆𝑥 ∆𝑥
∆𝑦 𝑢(𝑥 + ∆𝑥) − 𝑢(𝑥) 𝑣(𝑥 + ∆𝑥) − 𝑣(𝑥)
lim = lim + lim
∆𝑥→0 ∆𝑥 ∆𝑥→0 ∆𝑥 ∆𝑥→0 ∆𝑥

using the prime notation (‘) for d/dx


𝒅𝒚
[𝒖(𝒙) + 𝒗(𝒙)] = 𝒖′ (𝒙) + 𝒗′ (𝒙)
𝒅𝒙

4.1.5 Theorem 5: Derivative of Difference of Two Functions

The derivative of the difference of two functions is the difference of their


derivatives.
Proof: Let 𝑦 = 𝑢(𝑥) − 𝑣(𝑥)
𝑦 + ∆𝑦 = 𝑢(𝑥 + ∆𝑥) − 𝑣(𝑥 + ∆𝑥)
𝑦 + ∆𝑦 − 𝑦 = 𝑢(𝑥 + ∆𝑥) + 𝑣(𝑥 + ∆𝑥) − 𝑦
𝑦 + ∆𝑦 − 𝑦 = 𝑢(𝑥 + ∆𝑥) + 𝑣(𝑥 + ∆𝑥) − 𝑢(𝑥) − 𝑣(𝑥)
48
∆𝑦 = 𝑢(𝑥 + ∆𝑥) + 𝑣(𝑥 + ∆𝑥) − 𝑢(𝑥) − 𝑣(𝑥)
∆𝑦 = 𝑢(𝑥 + ∆𝑥) − 𝑢(𝑥) − 𝑣(𝑥 + ∆𝑥) − 𝑣(𝑥)
∆𝑦 𝑢(𝑥 + ∆𝑥) − 𝑢(𝑥) + 𝑣(𝑥 + ∆𝑥) − 𝑣(𝑥)
=
∆𝑥 ∆𝑥
∆𝑦 𝑢(𝑥 + ∆𝑥) − 𝑢(𝑥) 𝑣(𝑥 + ∆𝑥) − 𝑣(𝑥)
= −
∆𝑥 ∆𝑥 ∆𝑥
∆𝑦 𝑢(𝑥 + ∆𝑥) − 𝑢(𝑥) 𝑣(𝑥 + ∆𝑥) − 𝑣(𝑥)
lim = lim − lim
∆𝑥→0 ∆𝑥 ∆𝑥→0 ∆𝑥 ∆𝑥→0 ∆𝑥
∆𝑥
using the prime notation (‘) for d/dx
𝒅𝒚
[𝒖(𝒙) − 𝒗(𝒙)] = 𝒖′ (𝒙) − 𝒗′ (𝒙)
𝒅𝒙

ILLUSTRATIVE EXAMPLES

1. Find the indicated derivative


𝑑𝑦
a. 𝑦 = 3𝑥 2 + 2𝑥 − 1, 𝑓𝑖𝑛𝑑 𝑑𝑥
2 𝑑𝑦
b. b.𝑦 = 3 𝑤 3 − 4𝑤 + 86, 𝑓𝑖𝑛𝑑 𝑑𝑤

Solution:
a. 𝑦 = 3𝑥 2 + 2𝑥 − 1
𝑑 𝑑 𝑑 𝑑
(3𝑥 2 + 2𝑥 − 1) = (3𝑥 2 ) + (2𝑥) − (1)
𝑑𝑥 𝑑𝑥 𝑑𝑥 𝑑𝑥
𝑑𝑦
= 3(2𝑥 2−1 ) + 2[(1)𝑥 1−1 ] − 0
𝑑𝑥
𝑑𝑦
= 6𝑥 + 2(𝑥 0 )
𝑑𝑥
𝒅𝒚
= 𝟔𝒙 + 𝟐
𝒅𝒙
2
b. 𝑦 = 3 𝑤 3 − 4𝑤 + 86
𝑑 2 3 𝑑 2 3 𝑑 𝑑
( 𝑤 − 4𝑤 + 86) = ( 𝑤 )− (4𝑤) + (86)
𝑑𝑤 3 𝑑𝑤 3 𝑑𝑤 𝑑𝑤
𝒅𝒚
= 𝟐𝒘𝟐 − 𝟒
𝒅𝒘

2. Differentiate the functions:


a. y = 5 – 4x + 7x3 – x5
b. y = 2x–1/2 + 4x–3/2
1
c. y = 2t√𝑡 +
√𝑡

Solutions:
a. y = 5 – 4x + 7x3 – x5
y’ = 0 – 4x1-1 + 3(7)x3-1 – 5x5-1
y’ = – 4x0 + 21x2 – 5x4
49
y’ = – 4 + 21x2 – 5x4
b. y = 2x–1/2 + 4x–3/2
1 3
y’ = (− )(2) x–1/2 – 1 + (− ) (4) x–3/2 – 1
2 2

y’ = – x–3/2 –6 x–5/2 =– x–3/2 (1 + 6 x–4)


𝟏 𝟏
y’ = – 𝒙𝟑/𝟐 (𝟏 + )
𝒙𝟒
1
c. y = 2t√𝑡 +
√𝑡

y = 2 t5/2 + t –1/2
5 1 1 1
y’ = (2)(2) t5/2 - 1 + (− 2) t –1/2 – 1 = 5 t3/2 − 2 t –3/2 = t3/2 (5 − 2 t –3 )
𝟏
y’ = t3/2 (𝟓 − )
𝟐𝐭 𝟑

4.1.6 Theorem 6: Product Rules


The derivative of a product of the two function is the first function times the
derivative of the second plus the second function times the derivative of the first.
𝒅
[𝒖(𝒙) ∙ 𝒗(𝒙)] = 𝒖(𝒙) ∙ 𝒗′ (𝒙) + 𝒗(𝒙)𝒖′ (𝒙)
𝒅𝒙
Proof:
Let 𝑦 = 𝑢(𝑥) ∙ 𝑣(𝑥)
𝑦 + ∆𝑦 = 𝑢(𝑥 + ∆𝑥) ∙ 𝑣(𝑥 + ∆𝑥)
𝑦 + ∆𝑦 − 𝑦 = 𝑢(𝑥 + ∆𝑥) ∙ 𝑣(𝑥 + ∆𝑥) − 𝑦
𝑦 + ∆𝑦 − 𝑦 = 𝑢(𝑥 + ∆𝑥) ∙ 𝑣(𝑥 + ∆𝑥) − 𝑢(𝑥) ∙ 𝑣(𝑥)
∆𝑦 = 𝑢(𝑥 + ∆𝑥) ∙ 𝑣(𝑥 + ∆𝑥) − 𝑢(𝑥) ∙ 𝑣(𝑥)
Subtracting & Adding 𝑢(𝑥 + ∆𝑥)𝑣(𝑥)
∆𝑦 = 𝑢(𝑥 + ∆𝑥)𝑣(𝑥 + ∆𝑥) − 𝑢(𝑥)𝑣(𝑥) + 𝑢(𝑥 + ∆𝑥)𝑣(𝑥) − 𝑢(𝑥 + ∆𝑥)𝑣(𝑥)
∆𝑦 = 𝑢(𝑥 + ∆𝑥)[𝑣(𝑥 + ∆𝑥) − 𝑣(𝑥)] + 𝑣(𝑥)[𝑢(𝑥 + ∆𝑥) − 𝑢(𝑥)]
∆𝑦 𝑢(𝑥 + ∆𝑥)[𝑣(𝑥 + ∆𝑥) − 𝑣(𝑥)] + 𝑣(𝑥)[𝑢(𝑥 + ∆𝑥) − 𝑢(𝑥)]
=
∆𝑥 ∆𝑥
∆𝑦 [𝑣(𝑥 + ∆𝑥) − 𝑣(𝑥)] [𝑢(𝑥 + ∆𝑥) − 𝑢(𝑥)]
= 𝑢(𝑥 + ∆𝑥) + 𝑣(𝑥)
∆𝑥 ∆𝑥 ∆𝑥
∆𝑦 [𝑣(𝑥 + ∆𝑥) − 𝑣(𝑥)]
lim = lim 𝑢(𝑥 + ∆𝑥) ∙ lim
∆𝑥→0 ∆𝑥 ∆𝑥→0 ∆𝑥→0 ∆𝑥
[𝑢(𝑥 + ∆𝑥) − 𝑢(𝑥)]
+𝑣(𝑥) lim
∆𝑥→0 ∆𝑥
𝑑
[𝑢(𝑥) ∙ 𝑣(𝑥)] = 𝑢(𝑥 + 0) ∙ 𝑣 ′ (𝑥) + 𝑣(𝑥) ∙ 𝑢′ (𝑥)
𝑑𝑥
using the prime notation (‘) for d/dx
𝒅
[𝒖(𝒙) ∙ 𝒗(𝒙)] = 𝒖(𝒙) ∙ 𝒗′ (𝒙) + 𝒗(𝒙) ∙ 𝒖′ (𝒙)
𝒅𝒙
Or simply d(uv) = u dv + v du
50
Derivative of Multiple Product
𝒅
[𝒖(𝒙) ∙ 𝒗(𝒙) ∙ 𝒘(𝒙)] = 𝒖(𝒙)𝒗(𝒙)𝒘′ (𝒙) + 𝒖(𝒙)𝒘(𝒙)𝒗′ (𝒙)
𝒅𝒙
+𝒗(𝒙)𝒘(𝒙)𝒖′ (𝒙)
Or simply d(uvw) = u v dw + u w dv + v w du

ILLUSTRATIVE EXAMPLES

1. Find the derivative of 𝑦 with respect to 𝑥


a. 𝑦 = (𝑥 3 + 2𝑥)(2𝑥 − 1)
b. 𝑦 = (𝑥 3 − 3𝑥 2 )(𝑥 2 + 4𝑥 + 2)
Solution: We differentiate these functions using two methods (the Product Rule
and first finding the product then differentiate)

a. 𝒚 = (𝒙𝟑 + 𝟐𝒙)(𝟐𝒙 − 𝟏)

Using method 1
Let 𝑢(𝑥) = 𝑥 3 + 2𝑥 𝑣(𝑥) = 2𝑥 − 1
2
𝑢′(𝑥) = 3𝑥 + 2 𝑣′(𝑥) = 2
𝑑
[𝑢(𝑥) ∙ 𝑣(𝑥)] = 𝑢(𝑥) ∙ 𝑣 ′ (𝑥) + 𝑣(𝑥) ∙ 𝑢′ (𝑥)
𝑑𝑥
𝑑
[(𝑥 3 + 2𝑥)(2𝑥2 − 1)] = (𝑥 3 + 2𝑥)(2) + (2𝑥 − 1)(3𝑥 2 + 2)
𝑑𝑥
𝑑𝑦
= 2𝑥 3 + 4𝑥 + 6𝑥 3 + 4𝑥 − 3𝑥 2 − 2 = 8𝑥 3 + 8𝑥 − 3𝑥 2 − 2
𝑑𝑥
𝒚′ = 𝟖𝒙𝟑 − 𝟑𝒙𝟐 + 𝟖𝒙 − 𝟐

Using method 2
𝑦 = (𝑥 3 + 2𝑥)(2𝑥 − 1)
𝑦 = 2𝑥 4 − 𝑥 3 + 4𝑥 2 − 2𝑥
𝑑 𝒅𝒚
(2𝑥 4 − 𝑥 3 + 4𝑥 2 − 2𝑥) ⇒ = 𝒚′ = 𝟖𝒙𝟑 − 𝟑𝒙𝟐 + 𝟖𝒙 − 𝟐
𝑑𝑥 𝒅𝒙

b. 𝒚 = (𝒙𝟑 − 𝟑𝒙𝟐 )(𝒙𝟐 + 𝟒𝒙 + 𝟐)

Using method 1
Let 𝑢(𝑥) = (𝑥 3 − 3𝑥 2 ) 𝑣(𝑥) = (𝑥 2 + 4𝑥 + 2)
𝑢′(𝑥) = 3𝑥 2 − 6𝑥 𝑣′(𝑥) = 2𝑥 + 4
𝑑 𝑑𝑦
[(𝑥 3 − 3𝑥 2 )(𝑥 2 + 4𝑥 + 2)] = = y’
𝑑𝑥 𝑑𝑥
𝑦′ = (𝑥 3 − 3𝑥 2 )(2𝑥 + 4) + (𝑥 2 + 4𝑥 + 2)(3𝑥 2 − 6𝑥)
𝑦 ′ = 2𝑥 4 + 4𝑥 3 − 6𝑥 3 − 12𝑥 2 + 3𝑥 4 − 6𝑥 3 + 12𝑥 3 − 24𝑥 2 + 6𝑥 2 − 12𝑥
𝒚′ = 𝟓𝒙𝟒 + 𝟒𝒙𝟑 − 𝟑𝟎𝒙𝟐 − 𝟏𝟐𝒙

Using method 2
𝑦 = (𝑥 3 − 3𝑥 2 )(𝑥 2 + 4𝑥 + 2)
51
𝑦 = 𝑥 5 + 𝑥 4 − 10𝑥 3 − 6𝑥 2
𝑑 5 𝒅𝒚
(𝑥 + 𝑥 4 − 10𝑥 3 − 6𝑥 2 ) ⇒ = 𝒚′ = 𝟓𝒙𝟒 + 𝟒𝒙𝟑 − 𝟑𝟎𝒙𝟐 − 𝟏𝟐𝒙
𝑑𝑥 𝒅𝒙

4.1.7 Theorem 7: Quotient Rules


𝒅 𝒖(𝒙) 𝒗(𝒙)𝒖′ (𝒙) − 𝒖(𝒙)𝒗′ (𝒙)
[ ]=
𝒅𝒙 𝒗(𝒙) [𝒗(𝒙)]𝟐
Proof:
𝑢(𝑥)
Let 𝑦 = [ ]
𝑣(𝑥)

𝑢(𝑥 + ∆𝑥)
𝑦 + ∆𝑦 = [ ]
𝑣(𝑥 + ∆𝑥)
𝑢(𝑥+∆𝑥)
𝑦 + ∆𝑦 − 𝑦 = [ ]−y
𝑣(𝑥+∆𝑥)
𝑢(𝑥+∆𝑥) 𝑢(𝑥)
y+∆𝑦 − 𝑦 = [ ] −[ ]
𝑣(𝑥+∆𝑥) 𝑣(𝑥)
𝑢(𝑥+∆𝑥) 𝑢(𝑥)
∆𝑦 = [𝑣(𝑥+∆𝑥)] − [𝑣(𝑥)]
𝑣(𝑥)𝑢(𝑥+∆𝑥)−𝑢(𝑥)𝑣((𝑥+∆𝑥)
∆𝑦 =
𝑣(𝑥+∆𝑥)∙𝑣(𝑥)

Subtracting & Adding 𝑢(𝑥)𝑣(𝑥)


𝑣(𝑥)𝑢(𝑥 + ∆𝑥) − 𝑢(𝑥)𝑣(𝑥) − 𝑢(𝑥)𝑣(𝑥 + ∆𝑥) + 𝑢(𝑥)𝑣(𝑥)
∆𝑦 =
𝑣(𝑥 + ∆𝑥) ∙ 𝑣(𝑥)
𝑣(𝑥)[𝑢(𝑥+∆𝑥)−𝑢(𝑥)]−𝑢(𝑥)[𝑣(𝑥+∆𝑥)−𝑣(𝑥)]
∆𝑦 =
𝑔(𝑥+∆𝑥)∙𝑔(𝑥)

∆𝑦 𝑣(𝑥)[𝑢(𝑥+∆𝑥)−𝑢(𝑥)]−𝑢(𝑥)[𝑣(𝑥+∆𝑥)−𝑣(𝑥)]
=
∆𝑥 ∆𝑥∙𝑣(𝑥+∆𝑥)∙𝑣(𝑥)
∆𝑦 𝑣(𝑥) 𝑢(𝑥+∆𝑥)−𝑢(𝑥) 𝑢(𝑥) 𝑣(𝑥+∆𝑥)−𝑣(𝑥)
= (𝑣(𝑥+∆𝑥)𝑣(𝑥)) ( ) − (𝑣(𝑥+∆𝑥)𝑣(𝑥)) ( )
∆𝑥 ∆𝑥 ∆𝑥
∆𝑦 𝑣(𝑥) 𝑢(𝑥+∆𝑥)−𝑢(𝑥) 𝑢(𝑥) 𝑣(𝑥+∆𝑥)−𝑣(𝑥)
lim = lim ∙ lim − lim ∙ lim
∆𝑥→0 ∆𝑥 ∆𝑥→0 𝑣(𝑥+∆𝑥)𝑣(𝑥) ∆𝑥→0 ∆𝑥 ∆𝑥→0 𝑣(𝑥+∆𝑥)𝑣(𝑥) ∆𝑥→0 ∆𝑥

𝑑 𝑣(𝑥) 𝑢(𝑥)
= 𝑣(𝑥+0)𝑣(𝑥) ∙ 𝑢′(𝑥) − 𝑣(𝑥+0)𝑣(𝑥) ∙ 𝑣′(𝑥)
𝑑𝑥
𝑑 𝑣(𝑥) ′ (𝑥)
𝑢(𝑥)
= ∙ 𝑢 − ∙ 𝑣′(𝑥)
𝑑𝑥 [𝑣(𝑥)]2 [𝑣(𝑥)]2
𝒅 𝒖(𝒙) 𝒗(𝒙) ∙ 𝒖′ (𝒙) − 𝒖(𝒙) ∙ 𝒗′ (𝒙)
[ ]=
𝒅𝒙 𝒗(𝒙) [𝒗(𝒙)]𝟐

𝒗 𝒅𝒖 − 𝒖 𝒅𝒗
Or simply d(u/v) =
𝒗𝟐
52
When denominator is constant c
𝒅 𝒖(𝒙) 𝒖′ (𝒙)
[ ] =
𝒅𝒙 𝒄 𝒄

When numerator is constant c


𝒅 𝒄 𝒄 ∙ 𝒖′ (𝒙)
[ ]= −
𝒅𝒙 𝒖(𝒙) [𝒖(𝒙)]𝟐

ILLUSTRATIVE EXAMPLES
𝑑𝑦
1. Find .
𝑑𝑥
𝑥 2 +1 (𝑥 2 −1)(𝑥+1)
a. 𝑦 = b. 𝑦 =
𝑥 3 −1 𝑥 2 −2𝑥+1
Solution:
𝒙𝟐 +𝟏
a. Differentiating 𝒚 = , using quotient rule
𝒙𝟑 −𝟏
𝑑 𝑢(𝑥) 𝑣(𝑥)𝑢′(𝑥) − 𝑢(𝑥)𝑣 ′ (𝑥)
[ ]=
𝑑𝑥 𝑣(𝑥) [𝑣(𝑥)]2
𝑢(𝑥) = 𝑥 2 + 1 𝑣(𝑥) = 𝑥 3 − 1
𝑢′(𝑥) = 2𝑥 𝑣′(𝑥) = 3𝑥 2
[𝑣(𝑥)]2 = (𝑥 3 − 1)2
𝑑 𝑥 2 +1 (𝑥 3 −1)(2𝑥)−(𝑥 2 +1)(3𝑥 2 ) 𝑑𝑦
( )= = = 𝑦′
𝑑𝑥 𝑥 3 −1 (𝑥 3 −1)2 𝑑𝑥
2𝑥 3 − 2𝑥 − 3𝑥 4 − 3𝑥 2
𝑦′ =
(𝑥 3 − 1)2
−𝑥 − 2𝑥 − 3𝑥 2
4
𝑦′ =
(𝑥 3 − 1)2
−𝒙(𝒙𝟑 + 𝟑𝒙 + 𝟐)
𝑦′ =
(𝒙𝟑 − 𝟏)𝟐

(𝑥 2 −1)(𝑥+1)
b. We differentiate the function 𝑦 = using the shortest method.
𝑥 2 −2𝑥+1

First, we simplify the expression by factoring the numerator and the


denominator. Then we differentiate the remaining expression.
(𝑥 2 −1)(𝑥+1) (𝑥+1)(𝑥−1)(𝑥+1) (𝑥+1)(𝑥+1) 𝑥 2 +2𝑥+1
𝑦= = = =
𝑥 2 −2𝑥+1 (𝑥−1)(𝑥−1) (𝑥−1) 𝑥−1

𝑑 𝑢(𝑥) 𝑣(𝑥)𝑢′(𝑥) − 𝑢(𝑥)𝑣 (𝑥)
[ ]=
𝑑𝑥 𝑣(𝑥) [𝑣(𝑥)]2
𝑥 2 + 2𝑥 + 1
𝑦=
𝑥−1
2
𝑢(𝑥) = 𝑥 + 2𝑥 + 1
𝑢′(𝑥) = 2𝑥 + 2 𝑣(𝑥) = 𝑥 − 1
53
𝑣′(𝑥) = 1 2
(𝑣(𝑥)) = (𝑥 − 1)2

𝑑 𝑥 2 + 2𝑥 + 1 (𝑥 − 1)(2𝑥 + 2) − (𝑥 2 + 2𝑥 + 1)(1)
( )= = 𝑦′
𝑑𝑥 𝑥−1 (𝑥 − 1)2
2𝑥 2 +2𝑥−2𝑥−2−𝑥 2 −2𝑥−1
𝑦′ = (𝑥−1)2
𝟐
𝒙 − 𝟐𝒙 − 𝟑
𝒚′ =
(𝒙 − 𝟏)𝟐

𝑑𝑦 (𝟏+√𝒙)(𝟐+√𝒙)
2. Find 𝑑𝑥 of y = , at x = 1
𝒙
Solution:
(1+√𝑥)(2+√𝑥)
y=
𝑥
1 1
𝑥[(1+√𝑥)( )+(2+√𝑥)( )]−(1+√𝑥)(2+√𝑥)(1)
2√𝑥 2√𝑥
y’ =
𝑥2
1+√𝑥 2+√𝑥 1+√𝑥 + 2 +√𝑥
𝑥[( )+( )]−(2+3√𝑥+𝑥) 𝑥( )−(2+3√𝑥+𝑥)
2√𝑥 2√𝑥 2√𝑥
y’ = =
𝑥2 𝑥2
3𝑥+2𝑥√𝑥 3𝑥 + 2𝑥√𝑥 − 2√𝑥 (2 + 3√𝑥 + 𝑥)
( )−(2+3√𝑥+𝑥)
2√𝑥 2√𝑥
y’ = =
𝑥2 𝑥2
3𝑥 + 2𝑥√𝑥 − 4√𝑥 − 6𝑥− 2𝑥√𝑥 −3𝑥 − 4√𝑥 √𝑥 (3√𝑥 + 4)
y’ = = =−
2𝑥 5/2 2𝑥 5/2 2𝑥5/2
(𝟑√𝒙 + 𝟒)
y’ = −
𝟐𝒙𝟐
when x = 1
(3√1 + 4) 7
y’ = − 2 = − 2 = − 3.5
2(1)

4.1.8 The General Power Formula

The derivative of the nth term power of any differentiable function u(x) is the
product of 𝑛, the (𝑛 − 1)𝑠𝑡 power of the variable and the derivative of the function
u(x).

𝒅
(𝒖𝒏 ) = 𝒏 𝒖𝒏−𝟏 𝒖′ (𝒙)
𝒅𝒙
54
ILLUSTRATIVE EXAMPLES

Find the first derivative of the following functions

1. y = √𝟐𝒙 − 𝟏

Solution:

We know that √2𝑥 − 1 = (2x – 1)1/2


𝑑 1 1
[(2𝑥 − 1)1/2 ] = 𝑦′ = (2𝑥 − 1)2−1 𝑑(2𝑥 − 1)
𝑑𝑥 2
1 1
𝑦′ = (2𝑥 − 1)− 2 (2)
2
2 𝟏 √𝟐𝒙−𝟏
𝑦′ = = =
2√2𝑥−1 √𝟐𝒙−𝟏 𝟐𝒙−𝟏

2. y = (3x2 + 3)4

Solution:

𝑑
[(3𝑥 2 + 3)4 ] = 𝑦′ = 4 (3𝑥 2 + 3)3 𝑑(3𝑥 2 + 3)
𝑑𝑥
𝑦′ = 4 (3𝑥 2 + 3)3 (6𝑥)
𝒚′ = 24𝒙(𝟑𝒙𝟐 + 𝟑)𝟑

𝟒
3. y = √𝟑𝒙𝟐 + 𝟐𝒙 + 𝟏

Solution:
4
We know that √3𝑥 2 + 2𝑥 + 1 = (3𝑥 2 + 2𝑥 + 1)1/4
𝑑 1 1
[(3𝑥 2 + 2𝑥 + 1)1/4 ] = 𝑦′ = (3𝑥 2 + 2𝑥 + 1)4−1 𝑑(3𝑥 2 + 2𝑥 + 1)
𝑑𝑥 4
1 3
𝑦′ = (3𝑥 2 + 2𝑥 + 1)− 4 (6𝑥 + 2)
4
6𝑥+2 2(3𝑥+1) 3𝑥+1
𝑦′ = 4 = 4 = 4
4 √(3𝑥 2 +2𝑥+1)3 4 √(3𝑥 2 +2𝑥+1)3 2 √(3𝑥 2 +2𝑥+1)3
4
3𝑥+1 √(3𝑥 2 +2𝑥+1)
y' = 4 ∙ 4
2 √(3𝑥 2 +2𝑥+1) 3 √(3𝑥 2 +2𝑥+1)
𝟒
(𝟑𝐱+𝟏) √(𝟑𝐱 𝟐 +𝟐𝐱+𝟏)
y’ =
𝟐 (𝟑𝐱 𝟐 +𝟐𝐱+𝟏)

4. Find the equation of the tangent line to y = x(2 – x)2 at point (1, 1).
Solution:
Solve for y’ to get the slope of the tangent line to y = x(2 – x)2

y’ = x [(2)(2 – x)( –1)] y’ = –2x(2 – x) + (2 – x)2


+ (2 – x)2(1) y’ = (2 – x)[–2x + (2 – x)]
55
y’ = (2 – x)(–2x + 2 – x) y’ = (2 – 1)[2 – 3(1)]
y’ = (2 – x)(2 – 3x) y’ = (1)(–1)
y’ = –1
slope at point (1,1) where x = 1
to solve for the equation of the tangent line, use the point-slope equation of a line
y – y1 = m (x – x1)
y – 1 = (– 1) (x – 1)
y – 1 = –x + 1
x+y–2=0

STUDENT ACTIVITY 4.1

I. Check your answers in the PE 3.2 of the previous module by using the newly
learned formulas for algebraic functions.
1. y = 4x – 3x2 (2, -4) 2𝑥+1
4. y = (1, 1)
2. y = x – 3x + 1 (2, 3)
3 𝑥+2
3. y = √𝑥 (1, 1) 5. y2 = 3x + 1 (1, 2)

𝑑𝑦
II. Find in each of the following.
𝑑𝑥
1. 𝑦 = 1 + 3𝑥 − 𝑥 2 7. 𝑦 =
(2−𝑥)(2𝑥+1)
𝑥
2. 𝑦 = 𝑥 3 (2𝑥 2 − 1) 1 2

3. 𝑦 = (𝑥 − 1)(2𝑥 2 + 3) 8. 𝑦 = 2𝑥 + 3𝑥 3
2

4. 𝑦 = 𝑎𝑥 2 + 𝑏𝑥 + 𝑐 9. 𝑦 = 4𝑥 −2 + 3𝑥 −1 + 7
1
3
5. 𝑦 = √𝑥 2 − 3 √𝑥 − 5
3 10. 𝑦 = 2 (𝑥 2 − 𝑥 −2 )
𝑥 5 −2𝑥 3 −3
6. 𝑦 = 𝑥2

LESSON 4.2: THE CHAIN RULE

If y = f(u) and u = g(x), then y = f [g(x)] is a function of “x”. If “y” is a


differentiable function of “u” and if “u” is likewise a differentiable function of “x”, then
𝑑𝑦
y = f [g(x)] is a differentiable function of “x” and the derivative can be obtained
𝑑𝑥
by any of the following procedures:

𝑑𝑦
1. Express “y” explicitly in terms of “x” and differentiate to obtain .
𝑑𝑥
2. Differentiate each function with respect to its independent variable and use the
Chain Rule formula as follows:
𝒅𝒚 𝒅𝒚 𝒅𝒖
=
𝒅𝒙 𝒅𝒖
∙ 𝒅𝒙
56
𝒅𝒚
3. Differentiate “y” with respect to “u” to obtain . Express “x” explicitly in terms
𝒅𝒖
𝒅𝒙
of “u” and differentiate it with respect to “u” to obtain . Finally, use this
𝒅𝒖
formula:
𝒅𝒚
𝒅𝒚 𝒅𝒖
=
𝒅𝒙 𝒅𝒙
𝒅𝒖
ILLUSTRATIVE EXAMPLES

𝑑𝑦
Use the chain rule in obtaining
𝑑𝑥

1. y = 1 - √𝑢 − 3, u = (2x – 1)2
𝑑𝑦
First, find
𝑑𝑢
𝑑𝑦 1
y = 1 - √𝑢 − 3 ; =−
𝑑𝑢 2√𝑢−3
𝑑𝑢
Next, get , using the general power formula
𝑑𝑥
𝑑𝑢
u = (2x – 1)2 ; = 2 (2x – 1) (2) = 4 (2x – 1)
𝑑𝑥
𝑑𝑦
Use Chain Rule to find
𝑑𝑥
𝑑𝑦 𝑑𝑦 𝑑𝑢 1
= ∙ 𝑑𝑥 = (− 2√𝑢−3 ) ∙ [4 (2x – 1)]
𝑑𝑥 𝑑𝑢
𝑑𝑦 4(2𝑥−1) 2(2𝑥−1)
= − = −
𝑑𝑥 2√𝑢−3 √𝑢−3
Finally, substitute the value of u = (2x – 1)2
𝑑𝑦 2(2𝑥−1) √(2𝑥−1)2 −3
=− ∙ (rationalization)
𝑑𝑥 √(2𝑥−1)2 −3 √(2𝑥−1)2 −3
2 2
2(2𝑥−1)√(2𝑥−1) −3 2(2𝑥−1)√4𝑥 −4𝑥−2
=− 2 =− 2
(2𝑥−1) −3 4𝑥 −4𝑥−2
2
2(2𝑥−1)√4𝑥 −4𝑥−2
=− 2
2(2𝑥 −2𝑥−1)
𝒅𝒚 𝟐
(𝟐𝒙−𝟏)√𝟒𝒙 −𝟒𝒙−𝟐
=− 𝟐
𝒅𝒙 (𝟐𝒙 −𝟐𝒙−𝟏)

2. y = √𝑢 − 1, u = v (3 – 2v), v = x2

𝑑𝑦 𝑑𝑦 1
First, find y = √𝑢 − 1 ; =−
𝑑𝑢 𝑑𝑢 2√𝑢−1
𝑑𝑢
Next, get u = v (3 – 2v)
𝑑𝑣
57
𝑑𝑢
= v (-2) + (3 – 2v) = -2v + 3 – 2v = 3 – 4v
𝑑𝑣
𝑑𝑣 𝑑𝑣
Then, get v = x2 ; = 2x
𝑑𝑥 𝑑𝑥

𝑑𝑦
Use Chain Rule to find
𝑑𝑥
𝑑𝑦 𝑑𝑦 𝑑𝑢 𝑑𝑣 1
= ∙ ∙ = (− ) ∙ (3 – 4v) ∙ (2x)
𝑑𝑥 𝑑𝑢 𝑑𝑣 𝑑𝑥 2√𝑢−1
𝑑𝑦 2𝑥(3−4𝑣) 𝑥(3−4𝑣)
= − = −
𝑑𝑥 2√𝑢−1 √𝑢−1

Finally, substitute the value of u and v to express the function in terms of x.


𝑑𝑦 𝑥(3−4𝑥2) 𝑥(3−4𝑥2) 𝑥(3−4𝑥2)
= − = − = −
𝑑𝑥 √𝑣(3−2𝑣)−1 √𝑥2 (3−2𝑥2 )−1 √3𝑥2 −2𝑥4 −1

√3𝑥2 −2𝑥4 −1
𝑥(3−4𝑥2 )
= − ∙ (rationalization)
√3𝑥2 −2𝑥4 −1 √3𝑥2 −2𝑥4 −1

𝑥(3−4𝑥2)√3𝑥2 −2𝑥4 −1 𝑥(3−4𝑥2)√3𝑥2 −2𝑥4 −1


=− =−
3𝑥2 −2𝑥4−1 −(2𝑥4−3𝑥2+1)
𝒅𝒚 𝒙(𝟑−𝟒𝒙𝟐 )√𝟑𝒙𝟐 −𝟐𝒙𝟒 −𝟏
=
𝒅𝒙 𝟐𝒙𝟒 −𝟑𝒙𝟐 +𝟏

LESSON 4.3: HIGHER DERIVATIVES

Given a function, y = f(x), that is differentiable with respect to “x”, its


𝒅𝒚
derivative is called the first derivative of the function, denoted as .
𝒅𝒙
If the first derivative is differentiable, its derivative is called the second
derivative of the (original) function and is denoted by one of the following symbols:
𝒅𝟐 𝒚
, y”, f”(x)
𝒅𝒙𝟐
If the second derivative is differentiable, its derivative is called the third
derivative of the (original function) and is denoted by one of the following symbols:
𝒅𝟑 𝒚
, y’’’, f’’’(x)
𝒅𝒙𝟑
If a function has been subjected to “n” times of differentiation, its derivative
is called the nth derivative of the (original) function and is denoted by one of the
following symbols:
𝒅𝒏 𝒚 n n
, y , f (x)
𝒅𝒙𝒏
58
𝒅𝒚 𝒅𝟐 𝒚 𝒅𝒏 𝒚
The symbols , 𝒅𝒙𝟐 , 𝒅𝒙𝒏 are called Leibniz notations.
𝒅𝒙

ILLUSTRATIVE EXAMPLES

1. Find the fourth derivative [y(iv)] of y = 5x4 – 4x3 + 3x2 – 2x + 1

Solution:

y = 5x4 – 4x3 + 3x2 – 2x + 1


y’ = 4(5)x4-1 – 3(4)x3-1 + 2(3)x2-1 – 2x1-1 = 20x3 – 12x2 + 6x – 2
y’’ = 3(20)x3-1 – 2(12)x2-1 + 6x1-1 = 60x2 – 24x + 6
y’’’ = 2(60)x2-1 – 24x1-1 = 120x – 24
y(iv) = 120x1-1 = 120

𝟐
2. Find the third derivative [y’’’] of y =
√𝒙
Solution:

2
y=
√𝑥
2 (21√𝑥) 1
√𝑥 1 1
y’ = − 2 = − 𝑥 = − 𝑥√𝑥 = −
(√𝑥) √𝑥3

3√𝑥
− (32√𝑥) 2 3√𝑥 3
y’’ = − 2 = 𝑥3 = =
2𝑥 3 2𝑥 5/2
(√𝑥3 )

3(2)(52)(𝑥5/2−1) (15𝑥3/2 ) 15𝑥3/2 𝟏𝟓


y’’’ = − 2 =− 5 =− 5 = − 𝟕/𝟐
5/2 4𝑥 4𝑥 𝟒𝒙
(2𝑥 )

𝟐𝒙+𝟑
3. Find the second derivative [y’’] of y =
𝟐𝒙−𝟑

Solution:

2𝑥+3
y=
2𝑥−3
(2𝑥−3)(2)−(2𝑥+3)(2) (4𝑥−6)−(4𝑥+6) −12
y’ = = =
(2𝑥−3)2 (2𝑥−3)2 (2𝑥−3)2

−12(2)(2𝑥−3)(2) 48(2𝑥−3) 𝟒𝟖
y’’ = − 4 = 4 =
(2𝑥−3) (2𝑥−3) (𝟐𝒙−𝟑)𝟑

4. Find the second derivative [y’’] of y = (1 + 2x)2 (2 – 3x)3

Solution:
59
y = (1 + 2x)2 (2 – 3x)3
y’ = (1 + 2x)2 [3(2 – 3x)2(–3)] + (2 – 3x)3 [(2)(1 + 2x)(2)]
y’ = – 9 (1 + 2x)2 (2 – 3x)2 + 4 (2 – 3x)3 (1 + 2x)
y’ = [(1 + 2x) (2 – 3x)2] [– 9 (1 + 2x) + 4 (2 – 3x)]
y’ = [(1 + 2x) (2 – 3x)2] [– 9 – 18x + 8 – 12x)]
y’ = – (1 + 2x) (2 – 3x)2 (1 + 30x)
y’’ = – [(1 + 2x) (2 – 3x)2 (30) + (1 + 2x) (1 + 30x) (2) (2 – 3x) (– 3)
+ (2 – 3x)2 (1 + 30x) (2)]
y’’ = – [30(1+2x)(2–3x)2 – 6(1+2x)(1+30x)(2–3x) + 2(2–3x)2(1+30x)]
y’’ = –(2–3x) [30(1+2x)(2–3x) – 6(1+2x)(1+30x) + 2(2–3x)(1+30x)]
y’’ = –(2–3x) [30(2+4x–3x–6x2) – 6(1+2x+30x+60x2) + 2(2–3x+60x–90x2)]
y’’ = –(2–3x) [30(2+x–6x2) – 6(1+32x+60x2) + 2(2+57x–90x2)]
y’’ = –2(2–3x) [15(2+x–6x2) – 3(1+32x+60x2) + (2+57x–90x2)]
y’’ = –2(2–3x) (30 + 15x – 90x2 – 3 – 96x –180x2 + 2 + 57x – 90x2)
y’’ = –2 (2–3x) (29 – 24x – 360x2)
y’’ = 2 (2–3x) (360x2 + 24x – 29)

𝒙
5. Find the second derivative [y’’] of y =
√𝒙+𝟏
Solution:
𝑥
y=
√𝑥+1
1 𝑥 2(𝑥+1) − 𝑥
√𝑥+1 (1) − 𝑥 2 𝑥+1 √𝑥+1 − 2 𝑥+1 2√𝑥+1
√ √
y’ = 2 = =
(√𝑥+1) 𝑥+1 𝑥+1

2𝑥+2 − 𝑥 𝑥+2
= =
2(𝑥+1)√𝑥+1 2(𝑥+1)3/2

2(𝑥+1)3/2 (1) −(𝑥+2)[(2)(3/2)(𝑥+1)1/2 ]


y’’ = 2
[2(𝑥+1)3/2 ]
2(𝑥+1)3/2 −3 (𝑥+2)(𝑥+1)1/2 (𝑥+1)1/2 [2(𝑥+1) −3 (𝑥+2)]
y’’ = =
4(𝑥+1)3 4(𝑥+1)3
(𝑥+1)1/2 (2𝑥 + 2 − 3𝑥 −6) (𝑥+1)1/2 (− 𝑥 −4) (− 𝑥 −4)
y’’ = = =
4(𝑥+1)3 4(𝑥+1) 3 4(𝑥+1)5/2
𝒙+𝟒
y’’ = − 𝟓/𝟐
𝟒(𝒙+𝟏)

6. Find the second derivative of y = x4 – 5 + x–2 + 4x–4 when x = 1


y = x4 – 5 + x–2 + 4x–4
y’ = 4x3 – 2x–2 – 1 – 16x–4 – 1 = 4x3 – 2x–3 – 16x–5
60
y’ = 2(2x3 – x–3 – 8x–5)
y’’ = 2 [2(3)x3–1 – (-3)x–3–1 – 8(-5)x–5–1]
y’’ = 2 [6x2 + 3x–4 + 40x–6]
when x = 1; y’’ = 2 [6(1)2 + 3(1)–4 + 40(1)–6] = 2 (6 + 3 + 40) = 2(49)
y’’ = 98

LESSON 4.4: IMPLICIT DIFFERENTIATION

There are two ways to define functions, implicitly and explicitly. Most of
the equations we have dealt with have been explicit equations, such as y = 2x-3,
so that we can write y = f(x) where f(x) = 2x-3. But the equation 2x-y = 3 describes
the same function. This second equation is an implicit definition of y as a function
of x. As there is no real distinction between the appearance of x or y in the second
form, this equation is also an implicit definition of x as a function of y.

An implicit function is an equation of the form “f(x, y) = 0” in which y cannot


be expressed solely as a function of x or in which x cannot be expressed solely as
a function of y.

For this type of equation, the derivative y’ may be obtained by one of the
following procedures:
1. Differentiate both sides of the equation with respect to x.
𝑑𝑦
2. Collect all the terms with 𝑑𝑥 or y’ on one side of the equation.
𝑑𝑦
3. Factor out or y’.
𝑑𝑥
𝑑𝑦
4. Solve for or y’.
𝑑𝑥

In the implicit differentiation of higher orders, the following procedures


should be followed:

1. Differentiate implicitly the derivative of one lower order and replace the y’
by the relation previously found.

2. Differentiate implicitly the given equation as many times as necessary to


produce the required derivative and eliminate all derivatives of lower order.
This procedure is recommended only when a derivative of higher order at a
given point is required.
61
ILLUSTRATIVE EXAMPLES

Find the first and second derivative (dy/dx and d2y/dx2) of the given implicit
functions:

1. x + xy + y = 5
Solution:
x + xy + y = 5 (𝑥 + 1)(− 𝑦𝑥 +
+1
1
)− 𝑦− 1
1 + [xy’ + y(1)] + y’ = 0 y’’ = − 2
(𝑥 + 1)
xy’ + y’ = – 1 – y −(𝑦 + 1)− 𝑦− 1
(x + 1)y’ = – 1 – y y’’ = − 2
(𝑥 + 1)
–1–𝑦
y’ = −𝑦−1−𝑦−1
𝑥+1 y’’ = − 2
𝒚+𝟏 (𝑥 + 1)
y’ = − 𝒙 + 𝟏 −2𝑦−2
y’’ = − 2
(𝑥 + 1)𝑦′−(𝑦 + 1)(1) (𝑥 + 1)
y’’ = − 2
(𝑥 + 1) 𝟐(𝒚 + 𝟏)
y’’ =
(𝑥 + 1)𝑦′ − 𝑦− 1 (𝒙 + 𝟏)𝟐
y’’ = − 2
(𝑥 + 1)

2. x2 – xy + y2 = 3
Solution:
x2 – xy + y2 = 3
2x – [xy’ + y(1)] + 2yy’ = 0
2x – xy’ – y + 2yy’ = 0
2yy’ – xy’ = y – 2x
(2y – x) y’ = y – 2x
𝒚 – 𝟐𝒙
y’ =
𝟐𝒚 − 𝒙
(2𝑦−𝑥)(𝑦 ′ −2)− (𝑦−2𝑥)(2𝑦 ′ −1)
y’’ =
(2𝑦− 𝑥)2
(2𝑦𝑦 ′ −𝑥𝑦 ′ −4𝑦+2𝑥)− (2𝑦𝑦 ′ −2𝑥𝑦 ′ −𝑦+2𝑥)
y’’ =
(2𝑦− 𝑥)2
𝑦 – 2𝑥 𝑥(𝑦−2𝑥)−3𝑦(2𝑦−𝑥)
𝑥𝑦 ′ −3𝑦 𝑥( )−3𝑦 [ ]
2𝑦 − 𝑥 2𝑦 − 𝑥
y’’ = = =
(2𝑦− 𝑥)2 (2𝑦− 𝑥)2 (2𝑦− 𝑥)2
𝑥𝑦−2𝑥 2 −6𝑦 2 +3𝑥𝑦 4𝑥𝑦−2𝑥 2 −6𝑦 2
y’’ = =
(2𝑦− 𝑥)3 (2𝑦− 𝑥)3
𝟐(𝟐𝒙𝒚 −𝒙𝟐 −𝟑𝒚𝟐 )
y’’ =
(𝟐𝒚− 𝒙)𝟑

3. (x + y)2 = 2y
Solution:
(x + y)2 = 2y
2(x + y)(1+y’) = 2y’
62
2(x + y) + 2(x + y) y’ = 2y’
2(x + y) y’ – 2y’ = – 2(x + y)
[2(x + y) – 2] y’ = – 2(x + y)
– 2(𝑥 + 𝑦) – 2(𝑥 + 𝑦)
y’ = =
2(𝑥+𝑦)−2 2(𝑥+𝑦 −1)
𝒙+𝒚
y’ = − 𝒙+𝒚 −𝟏
(𝑥+𝑦 −1)(1 + 𝑦 ′ )−(𝑥+𝑦)(1+𝑦 ′ ) (1 + 𝑦 ′ ) (𝑥+𝑦 −1−𝑥−𝑦)
y’’ = – [ ] = – [ ]
(𝑥+𝑦 −1)2 (𝑥+𝑦 −1)2
𝑥+𝑦 𝑥+𝑦 𝑥+𝑦 −1−𝑥− 𝑦
(1 + 𝑦 ′ ) (−1) 1 + (− ) 1−
𝑥+𝑦 −1 𝑥+𝑦 −1 𝑥+𝑦 −1
y’’ = – [ ]= = =
(𝑥+𝑦 −1)2 (𝑥+𝑦 −1)2 (𝑥+𝑦 −1)2 (𝑥+𝑦 −1)2
𝟏
y’’ = − 𝟑
(𝒙+𝒚 −𝟏)

4. Find y’ of x3 – y3 = x1/2
Solution:
x3 – y3 = x1/2 6𝑥 2 √𝑥 − 1
1 3y2y’ =
2√ 𝑥
3x2 – 3y2y’ =
2√ 𝑥 6𝑥 2 √𝑥 − 1
1 y’ =
3y2y’ = 3x2 – 6𝑦2 √𝑥
2√ 𝑥
𝟔𝒙𝟓/𝟐 − 𝟏
y’ =
𝟔𝒚𝟐 √𝒙

5. Find y’ and y’’ of x2 – 4y2 = 4


Solution:
x2 – 4y2 = 4
2x – 8yy’ = 0 𝑥2 4𝑦2 − 𝑥2
𝑦−
8yy’ = 2x 4𝑦 4𝑦
y’’ = =
2𝑥 4𝑦 2 4𝑦2
y' =
8𝑦 4𝑦2 − 𝑥 2 𝑥2 − 4𝑦2
𝒙 y’’ = =−
y’ = 16𝑦3 16𝑦3
𝟒𝒚
Since: x2 – 4y2 = 4
4𝑦(1)−𝑥(4𝑦′ ) 4𝑦−4𝑥𝑦 ′ 4
y’’ = = y’’ = −
(4𝑦)2 16𝑦2 16𝑦3
𝑥
𝑦 − 𝑥(4𝑦) 𝟏
4(𝑦−𝑥𝑦′ ) y’’ = − 𝟑
y’’ = = 𝟒𝒚
16𝑦2 4𝑦2
6. Show that xy = 1, then y” ∙ x” = 4
Solution:
To get y” 𝑥 𝑦′− 𝑦
y’’ = −
xy = 1 𝑥2
𝑦
xy’ + y(1) = 0 𝑥 (− ) − 𝑦
𝑥
xy’ = - y y’’ = − 𝑥2
𝑦
y’ = − 𝑥 −𝑦 − 𝑦 −2𝑦
y’’ = − = −
𝑥2 𝑥2
63
𝟐𝒚 𝑦 𝑥′− 𝑥
y’’ = x’’ = −
𝒙𝟐 𝑦2
𝑥
𝑦 (− ) − 𝑥
𝑦
To get x” x’’ = − 𝑦2
xy = 1 −𝑥 − 𝑥 −2𝑥
x(1) + y(x’) = 0 x’’ = − = −
𝑦2 𝑦2
x’y = - x 𝟐𝒙
𝑥
x’ = − 𝑦 x’’ =
𝒚𝟐

Now, let us show that y” ∙ x” = 4


2𝑦 2𝑥
∙ =4
𝑥2 𝑦2 Since xy = 1
4𝑥𝑦 4
=4 =4
𝑥2𝑦2 1
4 Then, 4 = 4
=4
𝑥𝑦
7. Find y’ of x3y2 – xy = x+ y
Solution:
x3y2 – xy = x + y
x3 (2)yy’ + y2 (3)x2 – (xy’ + y) = 1 + y’
2 x3 y y’ + 3 x2 y2 – xy’ – y = 1 + y’
2 x3 y y’ – xy’ – y’ = 1 – 3 x2 y2 + y
(2 x3 y – x – 1) y’ = 1 – 3 x2 y2 + y
𝟏 – 𝟑 𝒙𝟐 𝒚𝟐 + 𝒚
y’ =
𝟐𝒙𝟑 𝒚 − 𝒙 − 𝟏

𝒅𝟐 𝒚 𝟏
8. Given √𝒙 + √𝒚 = 2, show that =
𝒅𝒙𝟐 𝒙𝟑/𝟐
Solution:
1 𝑦′ 𝑥 𝑦′ − 𝑦 𝑦
𝑥 (−√√𝑥 ) − 𝑦
+ =0 2√𝑥𝑦
2√ 𝑥 2√ 𝑦 y’’ = − =−
𝑥 2𝑥3/2√𝑦
𝑦′ 1
= −2 𝑥 √𝑥√𝑦 + 𝑦 √𝑦 (√𝑥 + √𝑦)
2√ 𝑦 √ y’’ = =
2𝑥 3/2 √𝑦 2𝑥 3/2 √𝑦
2√𝑦 √𝒚
y’ = − 2 𝑥 = − 𝒙
√ √
𝑦′ 1 Since √𝑥 + √𝑦 = 2
√𝑥 ( )− 𝑦 ( )
2√𝑦 √ 2√𝑥 2
y’’ = −
(√𝑥)
2 y’’ = 3/2
2𝑥
√𝑥 𝑦′ 𝑦
( − (2√√𝑥)
) 𝟏
y’’ = −
2√𝑦 y’’ = 𝟑/𝟐
𝑥 𝒙
64
STUDENT ACTIVITY 4.2
𝑑𝑦
I. Find 𝑑𝑥 of the following functions using chain rule.
1. y = 1 + √3𝑢 − 1, u = (1 – 2x)2
2. y = √1 − 𝑢, u = (4 – 2v), v = x3

II. Find the required derivative of the following functions:

1. y(iv) of y = 6x5 + 4x3 – 5x2 + 3x – 1


1
2. y’’ of y =
√𝑥−2
1 − 4𝑥
3. y’’ of y =
1 + 4𝑥
4. y’’’ of y = (2x2 – 3) (3 + 4x2)2
√𝑥+1
5. y’’ of y =
𝑥2

III. Find the first and second IV. Find the slope of the curve of the
derivative (dy/dx and d2y/dx2) of the given implicit function at the indicated
given implicit functions: point:

1. (x + y)2 = y 1. x2 – y2 – x = 1 at (2, 1)
2. x2 – 4xy + y2 = 5 2. (3x – y)2 = 6x + 2y + 23 when
3. x2 + y2 = y1/2 x=1

SUMMARY

Algebraic Functions
𝑑𝑦
(c) = 0
𝑑𝑥
𝑑𝑦 𝑛
(𝑥 ) = 𝑛𝑥 𝑛−1
𝑑𝑥
𝑑𝑦
(𝑐 ∙ 𝑓(𝑥)) = 𝑐 ∙ 𝑓 ′ (𝑥)
𝑑𝑥
𝑑𝑦
[𝑢(𝑥) + 𝑣(𝑥)] = 𝑢′ (𝑥) + 𝑣 ′ (𝑥)
𝑑𝑥
𝑑𝑦
[𝑢(𝑥) − 𝑣(𝑥)] = 𝑢′ (𝑥) − 𝑣 ′ (𝑥)
𝑑𝑥
𝑑
[𝑢(𝑥) ∙ 𝑣(𝑥)] = 𝑢(𝑥) ∙ 𝑣 ′ (𝑥) + 𝑣(𝑥) ∙ 𝑢′ (𝑥)
𝑑𝑥
𝑑
[𝑢(𝑥) ∙ 𝑣(𝑥) ∙ 𝑤(𝑥)] = 𝑢(𝑥)𝑣(𝑥)𝑤 ′ (𝑥) + 𝑢(𝑥)𝑤(𝑥)𝑣 ′ (𝑥) + 𝑣(𝑥)𝑤(𝑥)𝑢′ (𝑥)
𝑑𝑥

𝑑 𝑢(𝑥) 𝑣(𝑥) ∙ 𝑢′ (𝑥) − 𝑢(𝑥) ∙ 𝑣 ′ (𝑥)


[ ]=
𝑑𝑥 𝑣(𝑥) [𝑣(𝑥)]2
𝑑 𝑢(𝑥) 𝑢′ (𝑥)
[ ]=
𝑑𝑥 𝑐 𝑐
65
𝑑 𝑐 𝑐∙ 𝑢′ (𝑥)
[ ]= −
𝑑𝑥 𝑢(𝑥) [𝑢(𝑥)]2
𝑑 𝑛
(𝑢 ) = 𝑛 𝑢𝑛−1 𝑢′ (𝑥)
𝑑𝑥

Chain Rule
𝑑𝑦
𝑑𝑦 𝑑𝑦 𝑑𝑢 𝑑𝑦
𝑑𝑢
= ∙ or =
𝑑𝑥 𝑑𝑢 𝑑𝑥 𝑑𝑥 𝑑𝑥
𝑑𝑢

Higher Derivatives
𝑑2𝑦 𝑑𝑛𝑦
, 𝑑𝑥 𝑛
𝑑𝑥 2

REFERENCES

Love, Clyde E., Differential and Integral Calculus, The Macmillan Company

Morado, Lonel I., Module in Differential Calculus

Stewart, James, Single Variable Calculus, Brooks/Cole CENGAGE Learning, 7th


Edition, 2012

Swokowski, Earl W., Calculus with Analytic Geometry, 2nd Edition, Prindle, Weber
& Schmidt, 1979

Varela, Benjamin D., Workbook in Differential Calculus, 2009


66

OVERVIEW

MODULE In this module, we will have first a review on


some of the important trigonometric identities.
The main topics that we will discuss are finding
the derivatives of the trigonometric and inverse
trigonometric functions.

LEARNING OUTCOMES

At the end of this module, you should be able to:


1. to differentiate functions involving trigonometric functions;
2. to solve problems involving differentiation of trigonometric functions; and
3. to apply trigonometric identities to simplify the functions
4. to derive the formula for the derivatives of the inverse trigonometric
functions;
5. to differentiate functions involving inverse trigonometric functions;
6. to apply the derivative formulas to solve for the derivatives of inverse
trigonometric functions; and
7. to solve problems involving derivatives of inverse trigonometric functions.

REVIEW: TRIGONOMETRIC IDENTITITES

Reciprocal Identities
1 1
sin x = csc 𝑥 csc x = sin 𝑥
1 1
cos x = sec 𝑥 sec x = cos 𝑥
sin 𝑥 1 cos 𝑥 1
tan x = cos 𝑥 = cot 𝑥 cot x = =
sin 𝑥 tan 𝑥

Double Angle Formulas


sin 2x = 2 sin x cos x 2 tan 𝑥
tan 2x = 1−𝑡𝑎𝑛2 𝑥
cos 2x = cos2 x – sin2 x 𝑐𝑜𝑡 2 𝑥−1
cot 2x =
= 1 – 2 sin2 x 2 cot 𝑥

= 2 cos2 x – 1

Squared Identities
sin2 x + cos2 x = 1 1−cos 2𝑥
sin2 x = 2
sin2 x = 1 – cos2 x
cos2 x = 1 – sin2 x
67

1+cos 2𝑥 sec2 x = 1+ tan2 x


cos2 x = 2
tan2 x = sec2 x – 1
cot2 x = csc2 x – 1
csc2 x = 1 + cot2 x

Sum and Difference of Two Angles


sin (x ± y) = sin x cos y ± cos x sin y
cos (x ± y) = cos x cos y ∓ sin x sin y
tan 𝑥 ± tan 𝑦
tan (x ± y) = 1 ∓ tan 𝑥 tan 𝑦

LESSON 5.1: DERIVATIVE OF TRIGONOMETRIC FUNCTION

For the differentiation formulas of the trigonometric functions, all you need
to know is the differentiation formulas of sin u and cos u. Using these formulas and
the differentiation formulas of the algebraic functions, the differentiation formulas
of the remaining functions, that is, tan u, cot u, sec u and csc u may be obtained.

Derivative of sin u where u = f(x)


𝒅 𝒅𝒖
sin u = cos u
𝒅𝒙 𝒅𝒙
Derivative of cos u where u = f(x)
𝒅 𝒅𝒖
cos u = - sin u
𝒅𝒙 𝒅𝒙
𝐷𝑒𝑟𝑖𝑣𝑎𝑡𝑖𝑣𝑒 𝑜𝑓 𝑡𝑎𝑛 𝑢 𝑤ℎ𝑒𝑟𝑒 𝑢 = 𝑓(𝑥)
𝑑 𝑑 𝑠𝑖𝑛𝑥
(𝑡𝑎𝑛𝑥) = ( )
𝑑𝑥 𝑑𝑥 𝑐𝑜𝑠𝑥
𝑢𝑠𝑖𝑛𝑔 𝑑𝑒𝑟𝑖𝑣𝑎𝑡𝑖𝑣𝑒 𝑜𝑓 𝑞𝑢𝑜𝑡𝑖𝑒𝑛𝑡,
𝑑 𝑑
𝑑 (𝑐𝑜𝑠𝑥) (𝑠𝑖𝑛𝑥) − (𝑠𝑖𝑛𝑥) (𝑐𝑜𝑠𝑥)
(𝑡𝑎𝑛𝑥) = 𝑑𝑥 𝑑𝑥
𝑑𝑥 (𝑐𝑜𝑠𝑥)2
(𝑐𝑜𝑠𝑥)(𝑐𝑜𝑠𝑥) − (𝑠𝑖𝑛𝑥)(−𝑠𝑖𝑛𝑥)
=
𝑐𝑜𝑠 2 𝑥
(𝑐𝑜𝑠𝑥)(𝑐𝑜𝑠𝑥) − (𝑠𝑖𝑛𝑥)(−𝑠𝑖𝑛𝑥)
=
𝑐𝑜𝑠 2 𝑥
2 2
𝑐𝑜𝑠 𝑥 + 𝑠𝑖𝑛 𝑥 1
= =
𝑐𝑜𝑠 2 𝑥 𝑐𝑜𝑠 2 𝑥
𝑑
(𝑡𝑎𝑛𝑥) = 𝑠𝑒𝑐 2 𝑥
𝑑𝑥
𝑑 𝑑𝑢
𝑇ℎ𝑒𝑟𝑒𝑓𝑜𝑟𝑒 (𝑡𝑎𝑛 𝑢) = 𝑠𝑒𝑐 2 𝑢
𝑑𝑥 𝑑𝑥
68

Derivative of cot u where u = 𝑓(𝑥)


𝑑 𝑑 1
(cotx) = ( )
dx dx tanx
using derivative of quotient,
𝑑
𝑑 0 − (1) (tanx) 0 − (1)sec 2 𝑥
(tanx) = dx =
dx (tanx)2 (tanx)2
1
−sec 2 𝑥 − cos2 𝑥 −1
= = = = −csc 2 𝑥
tan2 𝑥 sin2 𝑥 sin2 𝑥
cos2 𝑥
𝑑
(cotx) = -csc 2 𝑥
dx
𝑑 du
Therefore (cot u) = -csc 2 u
dx dx

𝐷𝑒𝑟𝑖𝑣𝑎𝑡𝑖𝑣𝑒 𝑜𝑓 𝑠𝑒𝑐 𝑢 𝑤ℎ𝑒𝑟𝑒 𝑢 = 𝑓(𝑥)


𝑑 𝑑 1
(𝑠𝑒𝑐𝑥) = ( )
𝑑𝑥 𝑑𝑥 𝑐𝑜𝑠𝑥
𝑢𝑠𝑖𝑛𝑔 𝑑𝑒𝑟𝑖𝑣𝑎𝑡𝑖𝑣𝑒 𝑜𝑓 𝑞𝑢𝑜𝑡𝑖𝑒𝑛𝑡,
𝑑
𝑑 0 − (1) (𝑐𝑜𝑠𝑥) 0 − (1)(−𝑠𝑖𝑛𝑥)
(𝑡𝑎𝑛𝑥) = 𝑑𝑥 =
𝑑𝑥 (𝑐𝑜𝑠𝑥) 2 (𝑐𝑜𝑠𝑥)2
+𝑠𝑖𝑛𝑥 𝑠𝑖𝑛𝑥 1
= 2
= ⋅ = 𝑡𝑎𝑛 𝑥 𝑠𝑒𝑐 𝑥
𝑐𝑜𝑠 𝑥 𝑐𝑜𝑠𝑥 𝑐𝑜𝑠𝑥
𝑑
(𝑠𝑒𝑐𝑥) = 𝑡𝑎𝑛 𝑥 𝑠𝑒𝑐 𝑥
𝑑𝑥
𝑑 𝑑𝑢 𝑑𝑢
𝑇ℎ𝑒𝑟𝑒𝑓𝑜𝑟𝑒 (𝑠𝑒𝑐 𝑢) = 𝑡𝑎𝑛 𝑢 𝑠𝑒𝑐 𝑢 = 𝑠𝑒𝑐 𝑢 𝑡𝑎𝑛 𝑢
𝑑𝑥 𝑑𝑥 𝑑𝑥

𝐷𝑒𝑟𝑖𝑣𝑎𝑡𝑖𝑣𝑒 𝑜𝑓 𝑐𝑠𝑐 𝑢 𝑤ℎ𝑒𝑟𝑒 𝑢 = 𝑓(𝑥)


𝑑 𝑑 1
(𝑐𝑠𝑐𝑥) = ( )
𝑑𝑥 𝑑𝑥 𝑠𝑖𝑛𝑥
𝑢𝑠𝑖𝑛𝑔 𝑑𝑒𝑟𝑖𝑣𝑎𝑡𝑖𝑣𝑒 𝑜𝑓 𝑞𝑢𝑜𝑡𝑖𝑒𝑛𝑡,
𝑑
𝑑 0 − (1) (𝑠𝑖𝑛𝑥) 0 − (1)(𝑐𝑜𝑠𝑥)
(𝑐𝑠𝑐𝑥) = 𝑑𝑥 =
𝑑𝑥 (𝑠𝑖𝑛 𝑥) 2 (𝑠𝑖𝑛 𝑥)2
−𝑐𝑜𝑠𝑥 −𝑐𝑜𝑠𝑥 1
= = ⋅ = − 𝑐𝑜𝑡 𝑥 𝑐𝑠𝑐 𝑥
𝑠𝑖𝑛2 𝑥 𝑠𝑖𝑛𝑥 𝑠𝑖𝑛𝑥
𝑑
(𝑐𝑠𝑐𝑥) = −𝑐𝑜𝑡 𝑥 𝑐𝑠𝑐 𝑥
𝑑𝑥
𝑑 𝑑𝑢 𝑑𝑢
𝑇ℎ𝑒𝑟𝑒𝑓𝑜𝑟𝑒 (𝑐𝑠𝑐 𝑢) = −𝑐𝑜𝑡 𝑢 𝑐𝑠𝑐 𝑢 = −𝑐𝑠𝑐 𝑢 𝑐𝑜𝑡 𝑢
𝑑𝑥 𝑑𝑥 𝑑𝑥

Summary:

If u is a differentiable function of x, then the following are differentiation


formulas of the trigonometric functions

𝒅 𝒅𝒖
sin u = cos u
𝒅𝒙 𝒅𝒙
69

𝒅 𝒅𝒖
cos u = - sin u
𝒅𝒙 𝒅𝒙
𝒅 𝒅𝒖
tan u = sec2 u
𝒅𝒙 𝒅𝒙
𝒅 𝒅𝒖
cot u = - csc2 u
𝒅𝒙 𝒅𝒙
𝒅 𝒅𝒖
sec u = sec u tan u
𝒅𝒙 𝒅𝒙
𝒅 𝒅𝒖
csc u = - csc u cot u
𝒅𝒙 𝒅𝒙

ILLUSTRATIVE EXAMPLES

Find the first derivative of the given function.

1. y = sin 3x
du
u = 3x; =3
dx
d
sin 3x = cos 3x (3) = 3 cos 3x
dx

2. w = tan 2θ
du
u = 2θ; =2

𝐝
tan 2θ = sec2 2θ (2) = 2 sec2 2θ
𝐝𝛉

𝐲
3. z = sec
𝟐
y du 1
u= ; =
2 dy 2

d y y y 1 𝟏 𝐲 𝐲
sec = sec tan (2) = sec tan
dy 2 2 2 𝟐 𝟐 𝟐

4. y = csc 7x
du
u = 7x; =7
dx
d
csc 7x = - csc 7x cot 7x (7) = -7 csc 7x cot 7x
dx

5. w = 2 csc (1 – 3x)
du
u = 1 – 3x; =–3
dx
d
2 csc (1 – 3x) = 2 [- csc (1 – 3x) cot (1 – 3x) (– 3)]
dx
= 2 {-3 [- csc (1 – 3x) cot (1 – 3x)]}
= 2 [3 csc (1 – 3x) cot (1 – 3x)]
= 6 csc (1 – 3x) cot (1 – 3x)
70

6. z = cos3 2x
For this problem, we use combination of derivative formulas for power and
trigonometric functions.
du
u = cos 2x; = – sin 2x (2) = - 2 sin 2x
dx
d du
Now, using the power rule, un = n un-1
dx dx
d
cos3 2x = 3 cos2 2x (- 2 sin 2x) = - 6 cos2 2x sin 2x
dx
= - 6 sin 2x cos2 2x
𝐱
7. y = x2 sin
𝟐

For this problem, we use combination of derivative formulas for product,


power and trigonometric functions.
x
Let u = x2 and v = sin
2
du dv x 1 1 x
where
dx
= 2x and
dx
= cos ( ) = 2 cos 2
2 2
d dv du
Using the product rule, uv = u +v
dx dx dx
d x 1 x x x2 x x
dx
x2 sin
2
= x2 (2 cos 2) + sin 2 (2x)= 2
cos
2
+ 2x sin
2

x
We can simplify it further by factoring out
2
𝐱 𝐱 𝐱
= (𝒙 𝐜𝐨𝐬 𝟐 + 𝟒 𝐬𝐢𝐧 𝟐)
𝟐

8. w = sin β cos2 β

For this problem, we use combination of derivative formulas for product,


power and trigonometric functions.

Let u = sin β and v = cos2 β


du dv
where = cos β and = 2 cos β (-sin β) = -2 sin β cos β
dβ dβ

d dv du
Using the product rule, uv = u dx + v
dx dx
d
sin β cos2 β = sin β (-2 sin β cos β) + cos2 β (cos β)

= - 2 sin2 β cos β + cos3 β

We can simplify it further by factoring out - cos β


= - cos β (2 sin2 β – cos2 β)

9. y = cos4 t – sin4 t

For this problem, we use combination of derivative formulas for subtraction,


power and trigonometric functions.
71

Let u = cos t and v = sin t


du dv
where = - sin t and = cos t
dt dt

d du dv
Using the derivative for subtraction, (u-v) = dx - dx and power formula
dx
d du
un = n un-1
dx dx
d
(cos4 t – sin4 t) = 4 cos3 t (- sin t) – 4 sin3 t (cos t)
dt
= - 4 sin t cos3 t – 4 sin3 t cos t

We can simplify it further by factoring out – 4 sin t cos t


= - 4 sin t cos t (cos2 t + sin2 t)

We know that sin2 t + cos2 t = 1, therefore


= - 4 sin t cos t

10. y = sec4 θ – tan4 θ

For this problem, we use combination of derivative formulas for subtraction,


power and trigonometric functions.

Let u = sec θ and v = tan θ


du dv
where = sec θ tan θ and = sec2 θ
dθ dθ
d du dv
Using the derivative for subtraction, (u-v) = dx - dx and power formula
dx
d du
un = n un-1
dx dx
d
(sec4 θ – tan4 θ)

= 4 sec3 θ (sec θ tan θ) – 4 tan3 θ (sec2 θ)
= 4 sec4 θ tan θ – 4 tan3 θ sec2 θ

We can simplify it further by factoring out 4 sec2 θ tan θ


= 4 sec2 θ tan θ (sec2 θ – tan2 θ)

We know that sec2 θ - tan2 θ = 1, therefore


= 4 sec2 θ tan θ

11. y = sin (cos x)

Solution 1:
du
let u = cos x, where = - sin x
dx
Then,
dy
y = sin u, where = cos u
du
Using chain rule,
dy du dy
= ∙ = - sin x cos u
dx dx du
72

Substituting u = cos x
𝐝𝐲
= - sin x cos (cos x)
𝐝𝐱

Solution 2:
d du
Using sin u = cos u
dx dx
d
sin (cos x) = cos (cos x) (- sin x) = - sin x cos (cos x)
dx

12. r = (2 tan3 2θ – 1)1/2

For this problem, we use combination of derivative formulas for power and
trigonometric functions.
d du
First, let us use the power rule, un = n un-1 for the function inside the
dx dx
parenthesis.
du
u = 2 tan3 2θ - 1; = 2 (3)(tan2 2θ)(sec2 2θ)(2)

= 12 tan2 2θ sec2 2θ
Now, using the power rule to the whole function,
d 1
(2 tan3 2θ – 1)1/2 = (2 tan3 2θ – 1)-1/2 (12 tan2 2θ sec2 2θ)
dθ 2
= 6 tan2 2θ sec2 2θ (2 tan3 2θ – 1)-1/2
6 tan2 2θ sec2 2θ
=
√2 tan3 2θ – 1
Rationalizing,
6 tan2 2θ sec2 2θ √2 tan3 2θ – 1
= ∙
√2 tan3 2θ – 1 √2 tan3 2θ – 1
2 2
6 tan 2θ sec 2θ √𝟐 𝐭𝐚𝐧𝟑 𝟐𝛉 – 𝟏
=
𝟐 𝐭𝐚𝐧𝟑 𝟐𝛉 – 𝟏

𝐭𝐚𝐧 𝟐𝒙
13. y = 𝟏−𝐜𝐨𝐭 𝟐𝒙

For this problem, we use combination of derivative formulas for division and
trigonometric functions.
u = tan 2x, where du/dx = sec2 2x (2) = 2 sec2 2x
v = 1 – cot 2x, where dv/dx = - (-csc2 2x) (2) = 2 csc2 2x
du dv
d u v 𝑑𝑥 − u 𝑑𝑥
First, let us use the derivative for division,
dx v
( )= v2
d tan 2𝑥 (1−cot 2𝑥) (2 sec2 2x) −(tan 2𝑥) (2csc2 2x)
(
dx 1−cot 2𝑥
)= (1−cot 2𝑥)2
2 sec 2x−2 cot 2𝑥 sec 2x − 2 tan 2𝑥 csc2 2x
2 2
=
(1−cot 2𝑥)2
1 1
Using Inverse Identities sec2 2x = cos22x and csc2 2x = sin22x, also tan x =
sin 𝑥 cos 𝑥
cos 𝑥
and cot x = sin 𝑥
73

1 cos 2x 1 sin 2x 1
( )− ( ) − cos ( )
cos2 2x sin 2x cos2 2x 2x sin2 2x
=2
cos 2x 2
(1− )
sin 2x
2
sin 2x − sin 2x cos 2x − sin 2x cos 2x sin2 2x − 2 sin 2x cos 2x
( ) ( )
sin2 2x cos2 2x sin2 2x cos2 2x
=2 =2
sin 2x−cos 2x 2 ( sin 2x−cos 2x)2
( )
sin 2x sin2 2x
sin2 2x − 2 sin 2x cos 2x
( )
cos2 2x sin2 2x − 2 sin 2x cos 2x
=2 ( sin 2x−cos 2x)2
= 2 cos2 2x ( sin 2x−cos 2x)2

We know that sin 4x = 2 sin 2x cos 2x


sin2 2x − sin 4x
=2
cos2 2x ( sin2 2x−2 sin 2𝑥 cos 2𝑥 + cos2 2x)
sin2 2x − sin 4x
=2
cos2 2x ( sin2 2x + cos2 2x −2 sin 2𝑥 cos 2𝑥)

Since sin2 x + cos2 x = 1


sin2 2x − sin 4x
=2
cos2 2x ( 1 −sin 4𝑥)

To make the angles similar, we make use of other double angle identities,
1−cos 4𝑥 1+cos 4𝑥
sin2 2x = and cos2 2x =
2 2
1−cos 4𝑥
− sin 4x
= 2 1+cos24𝑥
( 1 −sin 4𝑥)
2
1−cos 4𝑥−2 sin 4𝑥
= 2 (1+cos 4𝑥)(21 −sin 4𝑥)
2
𝟏− 𝐜𝐨𝐬 𝟒𝒙 − 𝟐 𝐬𝐢𝐧 𝟒𝒙
= 𝟐 (𝟏+ 𝐜𝐨𝐬 𝟒𝒙)(𝟏 − 𝐬𝐢𝐧 𝟒𝒙)

If you have simpler solution than mine, you are free to send it to me and have
them checked. There can be more than one way of solving problems in Calculus.
Just choose the shortest and most practical solution, AND CORRECT, of course.

𝟏−𝐜𝐨𝐬 𝒙 𝟑
14. y = ( )
𝟏+𝐜𝐨𝐬 𝒙

Solution 1:
For this problem, we use combination of derivative formulas for power,
division and trigonometric functions.
Solving for the inner portion of the function,
u = 1 – cos x, where du/dx = - (- sin x) = sin x
v = 1 + cos x, where dv/dx = - sin x

d du
Next, let us use the power rule for the whole function, un = n un-1 for
dx dx
the function inside the parenthesis.

d 1−cos 𝑥 3 1−cos 𝑥 2 (1+cos 𝑥) (sin 𝑥) −(1−cos 𝑥) (− sin 𝑥)


(1+cos 𝑥) = 3 (1+cos 𝑥)
dx (1+cos 𝑥)2
74

(1−cos 𝑥)2
= 3 (1+cos 𝑥)4 [sin 𝑥 (1 + cos 𝑥) + sin 𝑥 (1 − cos 𝑥)]
(1−cos 𝑥)2
= 3 (1+cos 𝑥)4 [(1 + cos 𝑥) + (1 − cos 𝑥)]
sin 𝑥 (1−cos 𝑥)2
=3 (2)
(1+cos 𝑥)4
𝐬𝐢𝐧 𝒙 (𝟏−𝐜𝐨𝐬 𝒙)𝟐
=6 (𝟏+𝐜𝐨𝐬 𝒙)𝟒

Solution 2:
We can also try to simplify the given equation of a function first by using the
equivalent identities.
𝑥 1−cos 𝑥
Let us use half angle identity of tangent, tan = √1+cos 𝑥
2
1−cos 𝑥 𝑥
or 1+cos 𝑥 = tan2 2
d 1−cos 𝑥 3 d 𝑥 3 𝑥 2 𝑥 𝑥 1
(1+cos 𝑥) = (tan2 2) = 3 (tan2 2) (2 tan 2) (sec 2 2) (2)
dx dx
𝒙 𝒙
= 3 (𝐭𝐚𝐧𝟓 𝟐) (𝐬𝐞𝐜 𝟐 𝟐)
---------------------------
Now, let us prove that the answers we got from the 1st and 2nd solutions are equivalent.
sin 𝑥 (1−cos 𝑥)2 𝑥 𝑥
6 (1+cos 𝑥)4
= 3 (tan5 ) (sec 2 )
2 2
𝑥 1−cos 𝑥 1−cos 𝑥 𝑥
Again, Let us use half angle identity of tangent, tan = √1+cos 𝑥 or 1+cos 𝑥 = tan2 2
2
sin 𝑥 (1−cos 𝑥)2 𝑥 𝑥 𝑥
6 (1+cos 𝑥)4
= 3 (tan4 2) (tan 2) (1 + tan2 2)
sin 𝑥 (1−cos 𝑥)2 1−cos 𝑥 2 1−cos 𝑥 1−cos 𝑥
6 (1+cos 𝑥)4
= 3 (1+cos 𝑥) (√1+cos 𝑥) (1 + 1+cos 𝑥)
sin 𝑥 (1−cos 𝑥)2 (1−cos 𝑥)2 (1−cos 𝑥)1/2 1+cos 𝑥+ 1−cos 𝑥
6 (1+cos 𝑥)4
= 3 (1+cos 𝑥)2 (1+cos 𝑥)1/2 ( )
1+cos 𝑥
sin 𝑥 (1−cos 𝑥)2 (1−cos 𝑥)5/2 2
6 (1+cos 𝑥)4
=3 (1+cos 𝑥)5/2
(1+cos 𝑥)
sin 𝑥 (1−cos 𝑥)2 (1−cos 𝑥)5/2 √(1−cos 𝑥)5 (1−cos 𝑥)2 √(1−cos 𝑥)
6 (1+cos 𝑥)4
=6 (1+cos 𝑥)7/2
=6 = 6 (1+cos
√(1+cos 𝑥)7 𝑥)3 √(1+cos 𝑥)
sin 𝑥 (1−cos 𝑥)2 (1−cos 𝑥)2 1−cos 𝑥
6 (1+cos 𝑥)4
=6 (1+cos 𝑥)3

1+cos 𝑥
sin 𝑥 (1−cos 𝑥)2 (1−cos 𝑥)2 𝑥
6 (1+cos 𝑥)4
=6 (1+cos 𝑥)3
tan 2
𝑥 sin 𝑥 1−cos 𝑥
Other values equivalent to tan 2 = 1+cos 𝑥 = sin 𝑥
sin 𝑥 (1−cos 𝑥)2 (1−cos 𝑥)2 sin 𝑥
6 (1+cos 𝑥)4
=6 (1+cos 𝑥)3 1+cos 𝑥
𝐬𝐢𝐧 𝒙 (𝟏−𝐜𝐨𝐬 𝒙)𝟐 𝐬𝐢𝐧 𝒙 (𝟏−𝐜𝐨𝐬 𝒙)𝟐
6 (𝟏+𝐜𝐨𝐬 𝒙)𝟒
=6 (𝟏+𝐜𝐨𝐬 𝒙)𝟒
Therefore, the answers to the two different solutions are the same.

15. y = 3 sin 4x3 cos 4x3

Solution 1:

We will solve the derivative of this function by using the product rule,
d dv du
uv = u +v
dx dx dx

du
Let u = sin 4x3 ; = cos 4x3 (12x2) = 12x2 cos 4x3
dx
dv
v = cos 4x3 ; = - sin 4x3 (12x2) = -12x2 sin 4x3
dx
75

y’ = 3 [sin 4x3 (–12x2 sin 4x3) + cos 4x3 (12x2 cos 4x3)]
y’ = 3 (–12x2) (sin2 4x3 – cos2 4x3)
y’ = –36x2 (sin2 4x3 – cos2 4x3)

Use double angle identity for cos 8x3 = cos2 4x3 – sin2 4x3
y’ = 36x2 (cos2 4x3 – sin2 4x3)
y’ = 36x2 cos2 8x3

Solution 2:
In the alternative solution, we will first transform the function by using double
angle trigonometric identity for
sin 8x3 = 2 sin 4x3 cos 4x3
sin 8𝑥 3
= sin 4x3 cos 4x3
2
Transformed function is
3 sin 8𝑥 3
y = 3 sin 4x3 cos 4x3 =
2
We can now solve for the derivative of this function
3 sin 8𝑥 3
y=
2
3
y’ = cos 8x3 (24 x2) = 36 x2 cos 8x3
2

As you can see, the two solutions arrived at the same answer. It is for you
to decide which solution you will use.

16. y = (sec 2x)cos 2x

For this kind of problem, we will use logarithmic differentiation. Let us apply
natural logarithm to both sides of the equation.

ln y = ln (sec 2x)cos 2x

Recalling one of the properties of natural logarithm, ln xn = n ln x,


ln y = (cos 2x) ln (sec 2x)

Next, let us apply implicit differentiation.


𝑦′ sec 2𝑥 tan 2𝑥 (2)
= cos 2x + ln(sec 2𝑥) (− sin 2𝑥)(2)
𝑦 sec 2𝑥
𝑦′
= 2 cos 2x tan 2x – 2 sin 2x ln (sec 2x)
𝑦
y' = y [2 cos 2x tan 2x – 2 sin 2x ln (sec 2x)]

Substitute value of y = (sec 2x)cos 2x


sin 2𝑥
y' = 2(sec 2x)cos 2x [cos 2x cos 2𝑥 – sin 2x ln (sec 2x)]
y' = 2(sec 2x)cos 2x [sin 2x – sin 2x ln (sec 2x)]
y' = 2(sec 2x)cos 2x (sin 2x) [1 – ln (sec 2x)]
76

STUDENT ACTIVITY 5.1

Find the derivative of the following trigonometric functions:


𝑥2 sin x
1. y = cos 9. y =
2 1 – cos x
2. y = tan (1 – x) 1−tan x
10. y =
3. y = cot (2x + 3)2 sec x
sin x + cos 𝑥
4. y = sec x csc 2x 11. y =
sin x − cos 𝑥
5. y = cos 2x – sin x
12. y = tan (2x + 3)2
6. y = x – tan2 x
13. y = cot2 2x – 1
7. y = cot2 2x – 1
14. y = sin 2x – tan 2x
8. y = sin 2x – tan 2x 𝑥
15. y = x2 sec3 (3)

LESSON 5.2: DERIVATIVE OF INVERSE TRIGONOMETRIC FUNCTIONS

Let us first have a review on the definition and properties of inverse trigonometric
functions.
If y is a function of x determined by the relation
sin y = x → y is called the inverse sine function of x
denoted by
y = arcsin x or y = sin–1 x.

This also means y is the angle whose sine is x.

In general, the following are the definitions for inverse trigonometric functions:
𝜋 𝜋
y = sin–1 x ⟹ if sin y = x where − 2 ≤ 𝑦 ≤ 2
y = cos–1 x ⟹ if cos y = x where 0 ≤ 𝑦 ≤ 𝜋
𝜋 𝜋
y = tan–1 x ⟹ if tan y = x where – 2 < 𝑦 < 2
𝜋
y = cot–1 x ⟹ if cot y = x where 0 < 𝑦 < 2
𝜋
y = sec–1 x ⟹ if sec y = x where 0 ≤ 𝑦 ≤ if x ≥ 1
2
𝜋
or < 𝑦 ≤ 𝜋 if x ≥ -1
2
𝜋
y = csc–1 x ⟹ if csc y = x where 0 < 𝑦 ≤ if x ≥ 1
2
𝜋
or − ≤ 𝑦 < 0 if x ≤ -1
2

Derivative of Inverse Trigonometric Function

1. Derivative of y = Arcsin x or sin-1 x


In finding the derivative of y = sin-1 x, we use the definition of inverse trigonometric
function
𝜋 𝜋
sin y = x where − 2 ≤ 𝑦 ≤ 2

Differentiating with respect to y:


sin y = x
𝑑𝑥
cos y =
𝑑𝑦
𝑑𝑦
Cross multiplying to obtain
𝑑𝑥
77

𝑑𝑦 1
=
𝑑𝑥 cos 𝑦

From the identity: sin2y + cos2y = 1; cos2y = 1 – sin2y


cos y = √1 – sin2 𝑦
Since it sin y = x, then
cos y = √1 − 𝑥 2
Substitute,
𝑑𝑦 1
=
𝑑𝑥 √1−𝑥 2
But y = sin-1 x
𝑑(𝑠𝑖𝑛−1 𝑥) 1
=
𝑑𝑥 √1−𝑥 2
Therefore,
𝒅(𝒔𝒊𝒏−𝟏 𝒖) 𝟏 𝒅𝒖
=
𝒅𝒙 √𝟏 − 𝒖𝟐 𝒅𝒙
In similar manner, we can derive the formulas for other inverse trigonometric
functions.

Differentiation formulas for inverse trigonomet ric functions :


d
(
sin −1 u =
1
) du
dx 1 − u 2 dx
d
(cos −1 u = − ) 1 du
dx 1 − u 2 dx
d
dx
(
tan −1 u = )
1 du
1 + u 2 dx
d
dx
(
cot −1 u = − )1 du
1 + u 2 dx
d
(sec −1 u = ) 1 du
dx u u − 1 dx
2

d
(csc −1 u = − ) 1 du
dx u u − 1 dx
2

ILLUSTRATIVE EXAMPLES

Find the first derivative of the following inverse trigonometric functions:

1. y = Arcsin 4x

u = 4x; du/dx = 4

𝑑(𝐴𝑟𝑐 sin 4𝑥) 1 4


y’ = = (4) =
𝑑𝑥 √1 – (4𝑥)2 √1 − 16𝑥 2

Rationalizing,
78

4 √1 − 16𝑥 2
y’ = ∙
√1 − 16𝑥 2 √1 − 16𝑥 2
𝟒√𝟏 − 𝟏𝟔𝒙𝟐
y’ =
𝟏 − 𝟏𝟔𝒙𝟐

𝒙
2. y = Arctan 𝒂

𝑥 1
u = ; du/dx = , where 𝑎 is constant.
𝑎 𝑎

𝑥
𝑑(𝐴𝑟𝑐 tan ) 1 1 1 1
𝑎
y’ = = (𝑎) = =
𝑑𝑥 𝑥 2 𝑥 2 𝑥2
1+ ( ) 𝑎[1+ ( ) ] 𝑎[1+ 2 ]
𝑎 𝑎 𝑎
1 1
y’ = 𝑎2 + 𝑥2
= 𝑎2 + 𝑥2
𝑎( ) ( )
𝑎2 𝑎

𝒂
y’ =
𝒂𝟐 + 𝒙𝟐

3. x = Arcsin (1 – 2v)

u = 1 – 2v; du/dv = - 2

𝑑[𝐴𝑟𝑐 sin(1−2𝑣)] 1 −2
x’ = = (−2) =
𝑑𝑣 √1 – (1−2𝑣)2 √1 − (1−4𝑣+4𝑣 2 )
−2 −2 −2 −1
x’ = = = =
√4𝑣−4𝑣 2 √4𝑣(1−𝑣) 2√𝑣(1−𝑣) √𝑣(1−𝑣)

Rationalizing,
−1 √𝑣(1−𝑣)
x’ = ∙
√𝑣(1−𝑣) √𝑣(1−𝑣)

√𝒗(𝟏−𝒗)
x’ = −
𝒗(𝟏−𝒗)

4. x = Arctan (t2)

u = t2 ; du/dt = 2t

𝑑(𝐴𝑟𝑐 tan 𝑡 2 ) 1 2𝑡
x’ = = 2 2
(2𝑡) =
𝑑𝑡 1+ (𝑡 ) 1+ 𝑡 4
𝟐𝒕
x’ =
𝟏+ 𝒕𝟒

5. y = cos– 1 (3x)
79

u = 3x; du/dx = 3

𝑑[cos−1 (3𝑥)] 1 3
y’ = =− (3) = −
𝑑𝑥 √1 – (3𝑥)2 √1 − 9𝑥2

Rationalizing,
3 √1−9𝑥2
y’ = −
√1−9𝑥2
∙ √1−9𝑥2
𝟑√𝟏−𝟗𝒙𝟐
y’ = −
𝟏−𝟗𝒙𝟐

6. y = sec–1 2x2

u = 2x2; du/dx = 4x

𝑑(sec2 2𝑥 2 ) 1 4𝑥
y’ = = (4𝑥) =
𝑑𝑥 2𝑥 2 √(2𝑥 2 )2 − 1 2𝑥 2 √4𝑥 4 − 1
2
y’ =
𝑥 √4𝑥 4 − 1
Rationalizing,
2 √4𝑥 4 − 1
y’ = ∙
𝑥 √4𝑥 4 − 1 √4𝑥 4 − 1
𝟐√𝟒𝒙𝟒 − 𝟏
y’ =
𝒙(𝟒𝒙𝟒 − 𝟏)

7. y = 2 cos– 1 √𝒙
1
u = √𝑥; du/dx = 2
√𝑥

𝑑[cos−1 √𝑥] 1 1 1
y’ = 2 = −2 (2 𝑥 ) = −
𝑑𝑥 √1 – (√𝑥)2 √ √𝑥 √1−𝑥

1
=−
√𝑥(1−𝑥)

Rationalizing,
√𝑥(1−𝑥)
1
y’ = − ∙
√𝑥(1−𝑥) √𝑥(1−𝑥)

√𝒙 (𝟏−𝒙)
y’ = −
𝒙 (𝟏−𝒙)
80

𝟐
8. y = cot –1 ( )
𝒙

2 −2(1) −2
u= ; du/dx = =
𝑥 𝑥2 𝑥2

2
𝑑(𝐴𝑟𝑐 cot ) 1 2 2
𝑥 −2
y’ = =− ( 2) = 4 = 𝑥2 + 4
𝑑𝑥 2 2 𝑥 𝑥2 (1+ 2 ) 𝑥2 ( 2 )
1+ (𝑥) 𝑥 𝑥
𝟐
y’ =
𝒙𝟐 +𝟒

9. y = sec–1 5t + csc–1 5t

We solve for the derivative of this function by applying both addition of derivatives
and the derivative of inverse trigonometric functions.

u = 5t; du/dt = 5

𝑑(sec−1 5𝑡+ csc−1 5𝑡) 1 −1


y’ = = (5) + (5)
𝑑𝑡 5𝑡√(5𝑡)2 − 1 5𝑡√(5𝑡)2 − 1
1 1
= −
𝑡√25𝑡 2 − 1 𝑡√25𝑡 2 − 1

y’ = 𝟎

10. f(x) = x2 tan –1 (3x)

We solve for the derivative of this function by applying the product rule, power rule
and the derivative of inverse trigonometric function.

Let u = x2 ; du/dx = 2x
1 3
v = tan –1 (3x) ; dv/dx = (3 ) =
1+ (3𝑥)2 1+ 9𝑥
2

3
f’(x) = x2 ( ) + tan –1(3x) (2x)
1+ 9𝑥 2
3 𝑥2
f’(x) = ( ) + 2x tan –1(3x)
1+ 9𝑥 2

𝟑 𝒙𝟐 + 𝟐𝒙 (𝟏+ 𝟗𝒙𝟐 )𝒕𝒂𝒏−𝟏 (𝟑𝒙)


f’(x) =
𝟏+ 𝟗𝒙𝟐

𝟓
11. y = sec–1 (𝐜𝐬𝐜 )
𝒙

5 𝑑𝑢 5 5 −5 5 5 5
u = csc ; = - csc cot ( 𝑥2 ) = csc cot
𝑥 𝑑𝑥 𝑥 𝑥 𝑥2 𝑥 𝑥
81

5
𝑑[sec−1 (csc )] 1 5 5 5
𝑥
y’ = = 2
(𝑥 2 csc cot 𝑥)
𝑑𝑥 5 5 𝑥
csc √(csc ) − 1
𝑥 𝑥

5 2 5 2
We know that (csc 𝑥) − 1 = (cot 𝑥)
5 5
5 cot 5 cot
𝑥 𝑥
= 2
= 5
2
𝑥 cot
𝑥2 √(cot5) 𝑥
𝑥
𝟓
y' =
𝒙𝟐

12. Find y’ if tan –1 (x2y) = x + xy2

This is an example of a function in implicit form. We will just follow the procedure
in finding y’ for implicit equations.

tan –1 (x2y) = x + xy2

1
[x2y’ + y(2x)] = 1 + [x (2yy’) + y2 (1)]
1+ (𝑥 2 𝑦)2
𝑥2 2𝑥𝑦
y’ + = 1 + 2xyy’ + y2
1+𝑥 4 𝑦2 1+𝑥 4 𝑦2
𝑥2 2𝑥𝑦
y’ – 2xyy’ = 1 + y2 − 1+𝑥4 𝑦2
1+𝑥 4 𝑦2
𝑥2 (1+𝑥 4 𝑦 2 )(1+𝑦2 )− 2𝑥𝑦
(1+𝑥 4 𝑦2 – 2xy) y’ = 1+𝑥 4 𝑦2
𝑥2 −2𝑥𝑦 (1+𝑥4 𝑦2 ) (1+𝑥 𝑦2 )(1+𝑦2 )− 2𝑥𝑦
4
[ 1+𝑥4 𝑦2
]y’ =
1+𝑥 4 𝑦2

[𝑥 2 − 2𝑥𝑦 (1 + 𝑥 4 𝑦 2 ) ] y’ = (1 + 𝑥 4 𝑦 2 )(1 + 𝑦 2 ) − 2𝑥𝑦


(𝟏 + 𝒙𝟒 𝒚𝟐 )(𝟏+𝒚𝟐 ) − 𝟐𝒙𝒚
y’ =
𝒙𝟐 − 𝟐𝒙𝒚 (𝟏 + 𝒙𝟒 𝒚𝟐 )
1 + 𝑥 4 𝑦 2 + 𝑦 2 + 𝑥 4 𝑦 4 − 2𝑥𝑦
y’ =
𝑥 2 − 2𝑥𝑦− 2𝑥 5 𝑦 3
𝒙𝟒 𝒚𝟒 + 𝒙𝟒 𝒚𝟐 + 𝒚𝟐 − 𝟐𝒙𝒚 + 𝟏
y’ =
𝒙𝟐 − 𝟐𝒙𝒚− 𝟐𝒙𝟓 𝒚𝟑

𝒙
13. g(x) = x sin –1 + √𝟏𝟔 − 𝒙𝟐 , find g’(2)
𝟒

For the first term, we will apply the product rule.


1 1 𝑥 −2𝑥
g’(x) = [x ( ) + (sin−1 ) (1)] +
√1−(𝑥)
2 4 4 2 √16−𝑥 2
4
82

𝑥
+ (sin−1 4𝑥) −
𝑥
g’(x) =
4 √1 −
𝑥2 √16−𝑥 2
16
𝑥
+ (sin−1 4𝑥) −
𝑥
g’(x) =
4√
16 − 𝑥2 √16−𝑥 2
16
𝑥
+ (sin−1 4𝑥) −
𝑥
g’(x) = 4
√16−𝑥2 √16−𝑥 2
4
𝑥
+ (sin−1 4𝑥) −
𝑥
g’(x) =
√16−𝑥2 √16−𝑥 2
2
g’(2) = sin−1 4
g’(2) = 𝟑𝟎°

STUDENT ACTIVITY 5.2

I – Show the derivation of the derivatives of following inverse trigonometric


functions:
1. cos - 1 u
2. tan - 1 u
3. cot - 1 u
4. sec - 1 u
5. csc - 1 u

II – Find the derivatives of following functions and simplify.

1. y = tan –1 (x2)
2. y = (tan –1 x)2
3. y = cos –1 (sin –1 t)
4. y = sin –1 (2x + 1)
5. g(x) = √𝑥 2 − 1 sec–1 x
6. y = x sin –1 x + √1 − 𝑥 2
7. h(x) = arcsin √sin 𝑦
1−𝑥
8. y = arctan √1 + 𝑥
9. y = arctan (cos x)
10. y = tan –1 (x −√1 + 𝑥 2 )

SUMMARY

Derivatives of Trigonometric Functions


𝑑 𝑑𝑢 𝑑 𝑑𝑢
sin u = cos u cot u = - csc2 u
𝑑𝑥 𝑑𝑥 𝑑𝑥 𝑑𝑥
𝑑 𝑑𝑢 𝑑 𝑑𝑢
cos u = - sin u sec u = sec u tan u
𝑑𝑥 𝑑𝑥 𝑑𝑥 𝑑𝑥
𝑑 𝑑𝑢 𝑑 𝑑𝑢
tan u = sec2 u csc u = - csc u cot u
𝑑𝑥 𝑑𝑥 𝑑𝑥 𝑑𝑥
83
Derivatives of Inverse Trigonometric Functions
𝑑 1 𝑑𝑢
(sin – 1 u) =
𝑑𝑥 √1−𝑢2 𝑑𝑥
𝑑 1 𝑑𝑢
𝑑 1 𝑑𝑢 (sec – 1 u) =
(cos – 1 u) = − 𝑑𝑥 𝑢 √𝑢2 −1 𝑑𝑥
𝑑𝑥 √1 − 𝑢2 𝑑𝑥
𝑑 1 𝑑𝑢
𝑑 1 𝑑𝑢 (csc – 1 u) = −
𝑑𝑥 𝑢 √𝑢2 −1 𝑑𝑥
(tan – 1 u) =
𝑑𝑥 1 + 𝑢2 𝑑𝑥
𝑑 1 𝑑𝑢
(cot – 1 u) = − 1 + 𝑢2 𝑑𝑥
𝑑𝑥

REFERENCES

Love, Clyde E., Differential and Integral Calculus, The Macmillan Company

Stewart, James, Single Variable Calculus, Brooks/Cole CENGAGE Learning, 7th


Edition, 2012

Swokowski, Earl W., Calculus with Analytic Geometry, 2nd Edition, Prindle, Weber
& Schmidt, 1979

Varela, Benjamin D., Workbook in Differential Calculus, 2009


84
OVERVIEW

MODULE This module will review you on some of the important


properties of exponential and logarithmic functions. The
main focus of this module is finding derivatives of
exponential and logarithmic functions.

LEARNING OUTCOMES

At the end of this module, you should be able


to:
8. apply properties of exponential functions;
9. to differentiate functions involving exponential
functions;
10. apply properties on logarithmic functions; and
11. to differentiate functions involving logarithmic functions.

LESSON 6.1: EXPONENTIAL FUNCTIONS

Definition

If b is any number such that 0 < b < 1 or b > 1 then an exponential function
is a function in the form,
f (x) = bx
where b is called the base and x can be any real number.

Properties of bx

1. The exponential function f (x) is one-to-one. Thus, the following equivalent


conditions are satisfied for real numbers x1 and x2:
(a) If x ≠ y, then bx ≠ by
(b) If x = y, then bx = by

The Number e

If n is a positive integer, then


1 𝑛
(1 + 𝑛) → e ≈ 2.718281828… as n → ∞

The Natural Exponential Function

This natural exponential function is defined by


f (x) = ex
for every real number x.
85
Laws on Exponents

1. a𝑚 ⋅ 𝑎𝑛 = 𝑎𝑚+𝑛
𝑎𝑚−𝑛 , 𝑖𝑓 𝑚 > 𝑛
𝑚
𝑎 1 , 𝑖𝑓 𝑚 = 𝑛
2. 𝑛 = {
𝑎 1
, 𝑖𝑓 𝑚 < 𝑛
𝑎𝑛−𝑚
3. (𝑎𝑚 )𝑛 = 𝑎𝑚𝑛
4. (𝑎𝑏)𝑛 = 𝑎𝑛 𝑏 𝑛
𝑎 𝑛 𝑎𝑛
5. ( ) = 𝑛
𝑏 𝑏
6. 𝑎0 = 1 , provided a ≠ 0
𝑚
7. 𝑎𝑚⁄𝑛 = (𝑎1⁄𝑛 ) = (𝑎𝑚 )1⁄𝑛
8. alog𝑎 𝑥 = 𝑥
9. if a𝑥 = 𝑎 𝑦 then x = y

LESSON 6.2: DERIVATIVES OF EXPONENTIAL FUNCTIONS

The derivative of the exponential function for any given base and any
differentiable function of u
𝐹𝑜𝑟 𝑎𝑛𝑦 𝑔𝑖𝑣𝑒𝑛 𝑏𝑎𝑠𝑒 𝑎:
𝑑 𝑢 𝑑𝑢
(𝑎 ) = 𝑎𝑢 (𝑙𝑛 𝑎) ; where u = 𝑓(𝑥)
𝑑𝑥 𝑑𝑥

𝐹𝑜𝑟 𝑏𝑎𝑠𝑒 = 𝑒:
𝑑 𝑢 𝑑𝑢
(𝑒 ) = 𝑒 𝑢 ; where u = 𝑓(𝑥)
𝑑𝑥 𝑑𝑥

ILLUSTRATIVE EXAMPLES

𝟐
1. f(x) = 𝒆𝟑𝒙
2 𝑑
f '(x) = 𝑒 3𝑥 3𝑥 2
𝑑𝑥
2
= 𝑒 3𝑥 (6𝑥)
𝟐
f '(x) = 6x 𝒆𝟑𝒙

2. g(x) = 𝒆√𝟏−𝟐𝒙
𝑑
𝑑 (1−2𝑥)
g’(x) = 𝑒 √1−2𝑥 [𝑑𝑥 (√1 − 2𝑥)] = 𝑒 √1−2𝑥 [𝑑𝑥
2 √1−2𝑥
]
−2 √1−2𝑥
= 𝑒 √1−2𝑥 [2 ]∙[ ]
√1−2𝑥 √1−2𝑥
−√𝟏−𝟐𝒙 𝒆√𝟏−𝟐𝒙
g’(x) =
𝟏−𝟐𝒙
86
3. h(x) = 4x2 e1/x
𝑑 𝑑 𝑑 1
= 4x2 (𝑑𝑥 𝑒 1/𝑥 ) + 4e1/x (𝑑𝑥 𝑥 2 ) = 4x2 [𝑒 1/𝑥 (𝑥)] + 4e1/x (2𝑥)
𝑑𝑥
−1
= 4x2 (𝑒 1/𝑥 ) ( 2 ) + 8xe1/x = - 4e1/x + 8xe1/x
𝑥
h'(x) = - 4xe1/x (1 – 2x)

𝟐 −𝟒𝒙+𝟓
4. y = 𝟕𝟑𝒙
2 −4𝑥+5 𝑑
y' = 73𝑥 ln 7 [𝑑𝑥 (3𝑥 2 − 4𝑥 + 5)]
2 −4𝑥+5
= 73𝑥 ln 7 (6𝑥 − 4)
𝟐 −𝟒𝒙+𝟓
y' = 2 (ln 7) (𝟑𝒙 − 𝟐) 𝟕𝟑𝒙

𝟐
5. h(x) = ln 𝟒𝟑𝒙
1 𝑑 2 1 2 𝑑
h'(x) = 2 ∙ [𝑑𝑥 (43𝑥 )] = 2 ∙ 43𝑥 ln 4 [𝑑𝑥 (3𝑥 2 )] = ln 4 (6x)
4 3𝑥 4 3𝑥
h'(x) = (6 ln 4) x

6. G(x) = log(ex + 1) (e2x + 3)


1 𝑑 1 𝑑
G’(x) = log e [ (𝑒 𝑥 + 1)] + log e [ (𝑒 2𝑥 + 3)]
𝑒 𝑥 +1 𝑑𝑥 𝑒 2𝑥 +3 𝑑𝑥
log 𝑒 𝑑 log 𝑒 2𝑥 𝑑
= 𝑒 𝑥+1 𝑒 𝑥 [𝑑𝑥 (𝑥)] + 𝑒 2𝑥 +3 𝑒 [𝑑𝑥 (2𝑥)]
log 𝑒 log 𝑒
= 𝑒 𝑥+1 𝑒 𝑥 (1) + 𝑒 2𝑥 +3 𝑒 2𝑥 (2)
1 2𝑒 𝑥
= 𝑒 𝑥 log 𝑒 [𝑒 𝑥 +1 + ]
𝑒 2𝑥 +3
(𝑒 2𝑥 +3)+ 2𝑒 𝑥 (𝑒 𝑥 +1)
= 𝑒 𝑥 log 𝑒 [ (𝑒 𝑥 +1)(𝑒 2𝑥 +3)
]
𝑒 2𝑥 +3+ 2𝑒 2𝑥 +2𝑒 𝑥
= 𝑒 𝑥 log 𝑒 [ (𝑒 𝑥 +1)(𝑒 2𝑥 +3)
]
𝒆𝒙 𝐥𝐨𝐠 𝒆(𝟑𝒆𝟐𝒙 +𝟐𝒆𝒙 +𝟑)
G’(x) =
(𝒆𝒙 +𝟏)(𝒆𝟐𝒙 +𝟑)

𝟒 𝟐
7. f(x) = 𝟐𝟑𝒙 ∙ 𝟓𝒙
4 𝑑 2 2 𝑑 4
f '(x) = 23𝑥 [𝑑𝑥 (5𝑥 )] + 5𝑥 [𝑑𝑥 (23𝑥 )]
4 2 𝑑 2 4 𝑑
= 23𝑥 [5𝑥 ln 5 (𝑥 2 )] + 5𝑥 [23𝑥 ln 2 (3𝑥 4 )]
𝑑𝑥 𝑑𝑥
4 2 2 4
= 23𝑥 ∙ 5𝑥 ∙ ln 5 ∙ 2𝑥 + 5𝑥 ∙ 23𝑥 ∙ ln 2 ∙ 12𝑥 3
𝟒 𝟐
f '(x) = 𝟐𝒙 (𝟐𝟑𝒙 ∙ 𝟓𝒙 )[𝐥𝐧 𝟓 + (𝟔 𝐥𝐧 𝟐) ∙ 𝒙𝟐 ]
Or
𝟒 +𝟏 𝟐
f '(x) = 𝒙 (𝟐𝟑𝒙 ∙ 𝟓𝒙 )[𝐥𝐧 𝟓 + 𝐥𝐧 𝟐𝟔 ∙ 𝒙𝟐 ]
4 2
from, 2𝑥 (23𝑥 ∙ 5𝑥 ),
4 4
focusing on 2 ∙ (23𝑥 ) = 21 ∙ (23𝑥 )
applying law of exponent an ∙ am = an+m, then
4 4 +1
= (21+3𝑥 ) = (23𝑥 )
87
8. f(x) = 3xπ
At first, you may think that this is a problem which we could apply
exponential function differentiation. But taking a closer look at the exponent π,
which is a constant, we can just simply use power formula in getting the derivative
of the given function.
f'(x) = 3π xπ – 1

𝟐
9. y = 𝟑−𝒙
2 𝑑
y' = 3−𝑥 ln 3 [𝑑𝑥 (−𝑥 2 )]
2
y' = 3−𝑥 ln 3 (-2x)
𝟐
y' = - 2x (ln 3) 𝟑−𝒙

𝒙
10. y = 𝒆𝒆
𝑑
𝑥
y' = 𝑒 𝑒 𝑒𝑥
𝑑𝑥
𝑥 𝑑
y' = 𝑒 𝑒 𝑒 𝑥 𝑑𝑥 𝑥
𝑒𝑥 𝑥
y' = 𝑒 𝑒 (1)
𝒙
y' = 𝒆𝒆 +𝒙

11. Find dy/dx of this implicit equation: exy – x + y2 = 1


exy – x + y2 = 1
𝑑
exy 𝑑𝑥 (xy) – 1 + 2yy’ = 0
exy (xy’ + y) – 1 + 2yy’ = 0
(2y + exy x) y’ + exy y – 1 = 0
(2y + exy x) y’ = 1 – exy y
𝟏 – 𝒚𝒆𝒙𝒚
y’ =
𝟐𝒚 + 𝒙𝒆𝒙𝒚

STUDENT ACTIVITY 6.1

Find the first derivative of the following exponential functions.

1. f(x) = e5
2. f(r) = er + re
3. y = e–2t cost 4t
𝑒𝑥
4. y = 1−𝑒 𝑥
5. f(u) = e1/u
6. f(t) = sin (et) + esin t
7. y = x2 e–1/x
8. y = 𝑒 𝑘 tan √𝑥
9. y = √1 + 2𝑒 3𝑥
2
10. f(t) = sin2(𝑒 𝑠𝑖𝑛 𝑡 )
88
LESSON 3: LOGARITHMIC FUNCTIONS

Definition
If b is any number such that b > 0 and b ≠ 1 and x > 0 then,
y = logb x is equivalent to by = x
We usually read this as “log base b of x”.
In this definition logb y = x is called the logarithm form and by = x is called
the exponential form.

Looking closely, here is a diagram which will help you become an expert in
changing each form into the other.
exponent

y = logb x by = x

base
Property of logax

Property of logax Reason


(1) loga 1 = 0 a0 = 1
(2) loga a = 1 a1 = a
(3) loga ax = x ax = ax
(4) 𝒂𝒍𝒐𝒈𝒂𝒙 = x if y = loga x, then x = ay,
or x = 𝒂𝒍𝒐𝒈𝒂𝒙

More Properties of Logarithms

For these properties we will assume that u > 0 and w > 0.


Property Logarithms
5 logb (uw) = logb u + logb w
𝒖
6 logb (𝒘) = logb u – logb w
7 logb (uc) = c logb u
8 logb u = logb w then u = w

COMMON LOGARITHM

Definition
Logarithms with base 10 are called common logarithms. The symbol log
x is used as an abbreviation of log10 x, just as √ is used as an abbreviation for
2
√ .
log x = log10 x for every x>0
Note that the properties mentioned in the previous topic are also applicable
for common logarithms.
89
NATURAL LOGARITHM

Definition
The natural exponential function is given by f(x) = ex. The logarithmic
function with base e is called the natural logarithmic function. The ln x (read
“ell-en of x”) is an abbreviation for loge x, and we refer to it as the natural logarithm
of x. Thus, the natural logarithmic function and the natural exponential function
are inverse functions of each other.
ln x = log e x for every x>0

Examples: Equivalent Forms

Logarithmic form Exponential form


log x = 2 102 = x
log z = y + 3 10 y + 3 = z
ln x = 2 e2 = x
ln z = y + 3 e y+3 = z

Common and Natural Logarithms

Logarithms with base a Common logarithms Natural logarithms


loga 1 = 0 log 1 = 0 ln 1 = 0
loga = 1 log 10 = 1 ln e = 1
loga ax = x log 10x = x ln ex = x
𝑎𝑙𝑜𝑔𝑎 𝑥 = x 10log 𝑥 = x 𝑒 ln 𝑥 = x

Properties of Natural Logarithms

Law Natural Logarithms


1 ln (uw) = ln u + ln w
𝒖
2 ln (𝒘) = ln u – ln w
3 ln (uc) = c ln u

WARNINGS!
lna (u + w) ≠ lna u + lna w
lna (u – w) ≠ lna u – lna w

Change of Base Formula

For any positive number , we have


𝐥𝐧 𝒙
loga x =
𝐥𝐧 𝒂
90
LESSON 6.4: DERIVATIVE OF LOGARITHMIC FUNCTIONS

𝒅 1 𝟏
(loga x) = logae =
𝒅𝒙 x 𝐱 𝐥𝐧 𝐚

𝒅 𝑑𝑢/𝑑𝑥 𝟏 𝒅𝒖/𝒅𝒙
(loga u) = logae = 𝐥𝐧 𝐚
𝒅𝒙 u 𝐮

𝒅 𝑑𝑢/𝑑𝑥 𝑑𝑢/𝑑𝑥 𝟏 𝒅𝒖/𝒅𝒙


(log10 u) = M = log10e = 𝐥𝐧 𝟏𝟎
𝒅𝒙 u u 𝐮

𝒅 𝟏
(ln x) =
𝒅𝒙 𝒙

𝒅 𝒅𝒖/𝒅𝒙
(ln u) =
𝒅𝒙 𝒖

LOGARITHMIC DIFFERENTIATION

The calculation of derivatives of complicated functions involving products,


quotients, or powers can often be simplified by taking logarithms. The method used
in the following example is called logarithmic differentiation.

Steps in Logarithmic Differentiation


1. Take natural logarithms of both sides of an equation and use the properties of
logarithms to simplify.
2. Differentiate implicitly with respect to x.
3. Solve the resulting equation for y’.

𝑥 (1−𝑥 2 )2
Let us solve this function y = using logarithmic differentiation. Let us follow
(1+𝑥 2 )1/2
above steps.
𝑥 (1−𝑥 2 )2
ln y = ln Step 1
(1+𝑥 2 )1/2
ln y = ln [x(1 – x2)2] – ln (1+x2)1/2
ln y = ln x + ln (1 – x2)2 – ln (1+x2)1/2
ln y = ln x + 2 ln (1 – x2) – ½ ln (1+x2)

1 1 2 1
y’ = + (–2x) – (2x) Step 2
𝑦 𝑥 1−𝑥 2 2(1+𝑥 2 )
1 1 4𝑥 𝑥
y’ = – –
𝑦 𝑥 1−𝑥 2 (1+𝑥 2 )
1 4𝑥 𝑥
y’ = y [ – – ] Step 3
𝑥 1−𝑥 2 (1+𝑥 2 )

𝟐
𝒙 (𝟏−𝒙𝟐 ) 𝟏 𝟒𝒙 𝒙
y’ = [
(𝟏+𝒙𝟐 )𝟏/𝟐 𝒙
– – ] Substitute value of y
𝟏−𝒙𝟐 (𝟏+𝒙𝟐 )
91
ILLUSTRATIVE EXAMPLES

Find the first derivative of the following functions:

1. y = ln (x4 + 7x)
1 𝑑
y' = (x4 + 7x)
𝑥 4 +7𝑥 𝑑𝑥
1
y’ = (4x3 + 7)
𝑥 4 +7𝑥
𝟒𝒙𝟑 +𝟕
y’ =
𝒙(𝒙𝟑 +𝟕)

2. y = ln (cos 2x)
1 𝑑 1
y' = (cos 2x) = (- sin 2x)(2)
cos 2𝑥 𝑑𝑥 cos 2𝑥
2 sin 2𝑥
y' = − cos 2𝑥
y' = −2 tan 2x

3. y = cos (ln 2x)


𝑑 1
y' = - sin (ln 2x) (ln 2x) = - sin (ln 2x) (2)
𝑑𝑥 2𝑥
1
y' = - sin (ln 2x)
𝑥
𝐬𝐢𝐧(𝐥𝐧 𝟐𝒙)
y' = -
𝒙

5. y = x3/x
ln y = ln x3/x
3
ln y = ln x
𝑥
1 3 1 −3
y’ = ( ) + ln x ( 𝑥 2 )
𝑦 𝑥 𝑥
1 3 3 ln 𝑥
y’ = –
𝑦 𝑥2 𝑥2
1 3(1−ln 𝑥)
y’ =
𝑦 𝑥2
3𝑦(1−ln 𝑥)
y’ =
𝑥2
𝟑𝒙𝟑/𝒙 (𝟏−𝐥𝐧 𝒙)
y’ =
𝒙𝟐

6. y = log10 (3x2 – 5)
𝑑(3𝑥2 −5)
1 𝑑𝑥
y' =
ln 10 3x2 −5
𝟏 𝟔𝒙
y' =
𝐥𝐧 𝟏𝟎
(𝟑𝐱 𝟐 −𝟓)
92

𝑥 3/4 √𝑥 2 +1
7. y =
(3𝑥+2)5
𝑥 3/4 √𝑥 2 +1
ln y = ln
(3𝑥+2)5
ln y = ln (x3/4 √𝑥 2 + 1) – ln (3x + 2)5
ln y = ln x3/4 + ln (𝑥 2 + 1)1/2 – ln (3x + 2)5
ln y = ln x3/4 + ½ ln (x2 + 1) – 5 ln (3x + 2)
3 −1/4
1 𝑥 2𝑥 5 (3)
4
y’ = + –
𝑦 𝑥 3/4 2(𝑥 2 +1) (3𝑥+2)
1 3 𝑥 15
y’ = + –
𝑦 4𝑥 (𝑥 2 +1) (3𝑥+2)
3 𝑥 15
y’ = y [ + – ]
4𝑥 (𝑥 2 +1) (3𝑥+2)

𝒙𝟑/𝟒 √𝒙𝟐 +𝟏 𝟑 𝒙 𝟏𝟓
y’ =
(𝟑𝒙+𝟐)𝟓
[𝟒𝒙 + – ]
𝒙𝟐 +𝟏 𝟑𝒙+𝟐

8. y = ln |cos (ln x)|


1 𝑑 1 1
y' = [cos (ln x)] = cos(ln 𝑥) [- sin (ln x)] 𝑥 (1)
cos(ln 𝑥) 𝑑𝑥
1 sin(ln 𝑥) 1
y' = − 𝑥 [cos(ln 𝑥)] = − 𝑥 tan (ln x)
𝐭𝐚𝐧(𝐥𝐧 𝒙)
y' = − 𝒙

9. Find y’ and y” of y = ln (𝑥 + √1 + 𝑥 2 )
1 𝑑
y' = (𝑥 + √1 + 𝑥 2 )
𝑥+√1+𝑥 2 𝑑𝑥
1 2𝑥
y' = (1 + )
𝑥+√1+𝑥 2 2√1+𝑥 2
1 2√1+𝑥 2 + 2𝑥
y' = ( )
𝑥+√1+𝑥 2 2√1+𝑥 2
1 2(√1+𝑥 2 + 𝑥)
y' = [ ]
𝑥+√1+𝑥 2 2√1+𝑥 2
𝟏
y' =
√𝟏+𝒙𝟐
2𝑥

2√1+𝑥2
y’’ = [ 2 ]
(√1+𝑥 2 )

𝑥
y’’ = − [ 3]
(√1+𝑥2 )
𝒙
y’’ = − 𝟑/𝟐
(𝟏+𝒙𝟐 )
93
STUDENT ACTIVITY 6.2

Find the first derivative of the following logarithmic functions. Simplify if possible.

1. y = x ln x – x
2. y = ln (sin2x)
3. y = log5 (x ex)
4. y = ln (𝑥 √𝑥 2 − 1)
5. y = x5 + 5 x
6. y = tan [ln(ax + b)]
7. y = 2x log10 √𝑥
ln 𝑥
8. y = 𝑥2
9. y = ln ln ln x
10. y = √2 + ln 𝑥

SUMMARY
𝑑 𝑑𝑢
(au) = au (ln a) ; where u = f(x)
𝑑𝑥 𝑑𝑥
𝑑 𝑑𝑢
(eu) = eu 𝑑𝑥 ; where u = f(x)
𝑑𝑥
𝑑 1
(loga x) = x ln a
𝑑𝑥
𝑑 1 𝑑𝑢/𝑑𝑥
(loga u) = ln a
𝑑𝑥 u
𝑑 1 𝑑𝑢/𝑑𝑥
(log10 u) = ln 10
𝑑𝑥 u
𝑑 1
(ln x) = 𝑥
𝑑𝑥
𝑑 𝑑𝑢/𝑑𝑥
(ln u) =
𝑑𝑥 𝑢

REFERENCES

Love, Clyde E., Differential and Integral Calculus, The Macmillan Company

Stewart, James, Single Variable Calculus, Brooks/Cole CENGAGE Learning, 7th


Edition, 2012

Swokowski, Earl W., Calculus with Analytic Geometry, 2nd Edition, Prindle, Weber
& Schmidt, 1979

Varela, Benjamin D., Workbook in Differential Calculus, 2009


94
OVERVIEW

MODULE In this module we will discuss all about polynomial


curves. The topics are tangents and normals, critical
points, maxima and minima, concavity, points of inflection
and sketching polynomial curves.

LEARNING OUTCOMES

At the end of this module, you should be able to:


12. develop the equation of tangents and normals to plane curves;
13. locate the critical points and maxima and minima of a curve;
14. determine the concavity and points of inflection of the curve; and
15. sketch the polynomial curve.

LESSON 7.1: TANGENTS AND NORMALS TO PLANE CURVES

P1 (x1, y1)

Tangent
Line

Normal
Line
Polynomial
Curve

Figure A

Generally, the equation of a line of slope m through the point (x1, y1) is

y – y1= m (x – x1)

7.1.1 Tangent Line

The equation of tangent line at any point of a plane curve is

y – y1= mt (x – x1)

where mt is the slope of the curve at that point and is also the dy/dx = f ’(x) of the
curve.
95
7.1.2 Normal Line

The normal to a curve at the point (x1, y1) is defined to be the line through
that point and perpendicular to the tangent line there.

The equation of normal line at any point of a plane curve is


y – y1= mn (x – x1)
where mn = - 1/mt
y – y1= (- 1/mt) (x – x1)

ILLUSTRATIVE EXAMPLES

1. Find equations of the tangent and normal lines to y = f(x) = x3 – 2x2 + 4 at


(2, 4).

Solution:

a. Tangent Line

First, find the first derivative (y’ or f ’(x)) of the equation of the polynomial
curve.
y = f(x) = x3 – 2x2 + 4
y’ = f ’(x) = 3x2 – 4x
At point (2, 4), x = 2
mt = f ’(2) = 3(2)2 – 4(2) =4

Therefore, the equation of the tangent line is


y − 4 =4(x −2)

To find the slope–intercept for of this equation


y −4 =4(x −2)
y = 4x − 8 + 4
y = 4x − 4

b. Normal Line
1
We know that mn = - 1/mt = −
4
Therefore, the equation of the normal line can be easily written as
𝟏
y – 4 = − 𝟒 (x – 2)

Its slope–intercept equation is


1
y – 4 = − 4 (x – 2)
1 2
y = −4 x +4 + 4
1 2 + 16
y = −4 x + 4
𝟏 𝟗
y=− x+
𝟒 𝟐
96
2. Find equations of the tangent and normal lines to x2 + 3xy + y2 = 5 at
(1, 1).

Solution:

a. Tangent Line

By implicit differentiation,
x2 + 3xy + y2 = 5
2x + 3(xy’ + y) + 2yy’ = 0
2x + 3xy’ + 3y + 2yy’ = 0.
(3x + 2y) y’ = - 2x - 3y
−(2𝑥+3𝑦)
y’= mt =
3𝑥+2𝑦
At point (1, 1)
−[2(1) + 3(1)] −5
mt = = = −1
3(1) + 2(1) 5

Therefore, the equation of the tangent line is


y − 1 = −(x − 1)
Its slope–intercept equation is
y − 1 = −(x − 1)
y = −x + 1 + 1
y = −x + 2

b. Normal Line
−1
We know that mn = - 1/mt = −1 = 1
Therefore, the equation of the normal line can be easily written as
y −1 = x – 1
y=x–1+1
y = x.
𝟐
3. Find the equations of the tangent and normal lines with slope m = − 𝟗 to
the ellipse 4x2 + 9y2 = 40.

Solution:

a. point of tangency (x1, y1)

By implicit differentiation,
4x2 + 9y2 = 40
8x + 18yy’ = 0
18yy’ = - 8x
−8x
y’ =
18y
−4x
y’= mt =
9y

So, at a point of tangency (x1, y1),


4𝑥 2
mt = − 9𝑦 =− 9
97
4𝑥 2
− 9𝑦1 =− 9
1
-36x1 = -18y1
y1 = 2x1 (1)

Substitute the point (x1, y1) to the equation of the ellipse,


4x2 + 9y2 = 40
4x12 + 9y12 = 40 (2)

Substitute eq. (1) to eq. (2)


4x12 + 9(2x1)2 = 40
4x12 + 9(4x12) = 40
4x12 + 36x12 = 40
40x12 = 40
x1 2 = 1
x1 = ±1

Therefore, the required points are (1, 2) and (−1, −2).


𝟐
At (1, 2), an equation of the tangent line is y − 2 = − 𝟗 (x −1) and the
𝟗
equation of the normal line is y −2 = (x −1)
𝟐

𝟐
At (−1, −2), an equation of the tangent line is y +2 =− 𝟗 (x +1) and the
𝟗
equation of the normal line is y +2 = 𝟐 (x +1)

𝒙−𝟏
4. At what points do the curve y = , (x ≠ -1) have slope of 2?
𝒙+𝟏

Solution:
𝑥−1
y=
𝑥+1
(𝑥+1)(1) − (𝑥−1)(1)
y’ =
(𝑥+1)2
𝑥+1 − 𝑥+1
y’ =
𝑥 2 +2𝑥+1
2
y’ =
𝑥 2 +2𝑥+1

Substitute value of m = y’ = 2
2
2=
𝑥 2 +2𝑥+1
2 (𝑥 2 + 2𝑥 + 1) = 2
2x2 + 4x + 2 – 2 = 0
2x2 + 4x = 0
2x(x + 2) = 0

Finding the values of x


2x = 0; x=0
x + 2 = 0; x = -2

Points where slope is equal to 2


0−1
When x = 0, y = = -1 Therefore, P1 (0, -1)
0+1
98
−2−1 −3
When x = -2, y = = =3 Therefore, P2 (-2, 3)
−2+1 −1

−𝟖
5. What is the equation of the tangent line and normal line to y = at (4, -
√𝒙
4)?

Solution:

First, let us check if P(4, -4) is on the curve by substituting the coordinates to the
equation.
−8 −8 −8
y= ⇒ -4= ⇒ -4= ⇒ -4=-4
√𝑥 √4 2
Therefore, P(4, -4) is on the curve and it is the point of tangency.

−8
y=
√𝑥
1
−(−8)( )
2√𝑥
y’ = 2
(√𝑥)
4
√𝑥
y’ =
𝑥
4
y’ =
𝑥 √𝑥

At P(4, -4)
4 4 1
mt = y’ = = =
4√ 4 8 2

Therefore, equation of the tangent line at P(4, -4) with mt = ½ is


y - y1 = m (x – x1)
y + 4 = ½ (x – 4)
2y + 8 = x – 4
x – 2y – 8 – 4 = 0
x – 2y – 12 = 0 tangent line equation

Getting the equation of normal line, we know that mn = - 1/mt = -1/(1/2) = - 2


y - y1 = m (x – x1)
y + 4 = -2 (x – 4)
y + 4 = -2x + 8
2x + y – 8 + 4 = 0
2x + y – 4 = 0 normal line equation

6. Find the equation of the tangent line to the curve y = 2x2 + 3 that is parallel
to the line 8x – y + 3 = 0?

Solution:

Let us solve for the slope of the line since mt = mline


8x – y + 3 = 0
y = 8x + 3
y’ = 8 = mline

Now, we can solve for the slope of the tangent line to the curve y
99
y = 2x2 + 3
y’ = 4x

Substituting value of y’ = mline = mt


8 = 4x1
x1 = 2

When x1 = 2
y1 = 2x12 + 3
y1 = 2(2)2 + 3
y1 = 8 + 3 = 11

Therefore, point of tangency is P(2, 11)

Now, let us find the equation of the Tangent Line, knowing that P(2,11) and mt = 8
y – y1 = m (x – x1)
y – 11 = 8 (x – 2)
y – 11 = 8x – 16
8x – y + 5 = 0 tangent line equation

LESSON 7.2: CRITICAL POINTS, MAXIMA AND MINIMA

7.2.1 Increasing and Decreasing Functions

The term increasing, decreasing, and constant are used to describe the
behavior of a function as we travel left to right along its graph. An example is shown
below.

Figure B

The following definition, which is illustrated in the following figure, expresses these
intuitive ideas precisely.

Let f be defined on an interval, and let x1 and x2 denote points in that interval.

(a) f is increasing on the interval if f (x1) < f (x2) whenever x1 < x2.
(b) f is decreasing on the interval if f (x1) > f (x2) whenever x1 < x2.
(c) f is constant on the interval if f (x1) = f (x2) for all points x1 and x2.
100

f(x2) f(x1)
f(x1) f(x2)
f(x1) f(x2)

f(x1) < f(x2) if x1 < x2 f(x1) > f(x2) if x1 < x2 f(x1) = f(x2) for all x1 and x2

Figure C

Figure D

Theorem 1. Let f be a function that is continuous on a closed interval [a, b] and


differentiable on the open interval (a, b).
(a) If f’(x) > 0 for every value of x in (a, b), then f is increasing on [a, b].
(b) If f’(x) < 0 for every value of x in (a, b), then f is decreasing on [a, b].
(c) If f’(x) = 0 for every value of x in (a, b), then f is constant on [a, b].

7.2.2 Maximum and Minimum Points


At a point where the value of the function f(x) is algebraically greater than
the neighboring points, the function is said to have a maximum value and this
point is called a maximum point (point F on Figure A).
At a point where the value of the function f(x) is algebraically lower than
the neighboring points, the function is said to have a minimum value and this point
is called a minimum point (point D on Figure A).
The tangents to the maximum and minimum points are parallel to the x-axis,
which means that:
y' = 0 = mt
The points at which y’ = 0 are first called critical points and the
corresponding values of “x” are the critical values or numbers of “x” prior to the
test for maximum and minimum points.
101

Figure E

Definition 1. Let c be a number in the domain D of a function f. Then f(c)is the


■ absolute maximum value of f on D if f(c) ≥ f(x) for all x in D.
■ absolute minimum value of f on D if f(c) ≤ f(x) for all x in D.

An absolute maximum or minimum is sometimes called a global maximum


or minimum. The maximum and minimum values of are called extreme values of
f.

Definition 2. The number f(c) is a


■ local maximum value of f if f(c) ≥ f(x) when x is near c.
■ local minimum value of f if f(c) ≤ f(x) when x is near c.
In the Figure E,
Absolute Minimum = f(a)
Absolute Maximum = f(d)
Local Minimum = f(c) for interval (a, c) and f(e) for interval (c, e)
Local Maximum = f(b) for interval (a, c) and f(d) for interval (c, e)

7.2.3 Tests

To find the maxima and minima of the function, the following steps may be
followed:
▪ Obtain the first derivative, y’ or dy/dx, of the function.
▪ Equate the first derivative to zero and solve for the critical values of “x”.
▪ Determine whether a particular critical value of “x” is a maximum or minimum
point employing one of the following tests:
102
First Derivative Test

This is done by substituting values of “x” which are greater and lesser than
the given critical value of “x”, and then observing the change in the value of y’.
The decision rule is:
At a point where y’=0,
(a) if y’ changes from positive to negative (as x increases), y is a maximum;
(b) if y’ changes from negative to positive, y is a minimum;
(c) if y’ does not change sign, y is neither a maximum nor a minimum.

Figure F

Second Derivative Test

The Second Derivative (y’’) Test is the rate of change of the first derivative
(y’). It follows that when y’’ is positive, y’ is increasing; as “x” increases, the tangent
line turns in a counterclockwise movement and the curve is concave upward.
When y’’ is negative, y’ decreases; the curve is concave downward.
Based on the above stated facts, this test is expressed as follows: At a point
where y’ = 0
103
(a) if y’’ < 0, the value of the function ‘y’ is a maximum;
(b) if y’’ > 0, ‘y’ is a minimum; and
(c) if y’’ = 0, the test fails

ILLUSTRATIVE EXAMPLES

A. Find the maximum and/or minimum points of the curve of the given
function by the First Derivative Test:

1. f(x) = 3x4 – 4x3 – 12x2 + 5


Solution:
f'(x) = 12x3 – 12x2 – 24x
f'(x) = 12x(x2 – x – 2)
f'(x) = 12x(x + 1)(x – 2)

Set f’(x) = 0
0 = 12x(x + 1)(x – 2)
Critical points are
12x = 0; x=0
x + 1 = 0; x = −1
x – 2 = 0; x = 2

(1) (2) (3) (4) (5) (6)


Interval 12x x–2 x+1 = f’(x) f
x<-1 - - - = - decreasing on (-∞, -1)
-1<x<0 - - + = + increasing on (-1, 0)
0<x<2 + - + = - decreasing on (0,2)
x>2 + + + = + increasing on (2, ∞)

This chart helps us visualize the graph of the curve even without actually
sketching it. In column (1), critical points were used as intervals. In columns (2),
(3) and (4), signs represent the sign of the result when you substitute points on the
interval to the factor of the derivative of the function assigned in each column. For
example, assigning -2 to 12x for interval x<-1, will result to 12(-2) = -24, thus the
negative sign. Also, assigning -0.5 to x + 1 for interval -1 < x < 0, will result to (-0.5)
+ 1 = +0.5, thus the positive sign. In column (5) represents the resulting sign when
you multiply the signs of (2)x(3)x(4). For example, in interval -1<x<0, multiplying
(-)(-)(+) = (+) sign. Also, in the interval 0<x<2, multiplying (+)(-)(+) = (-) sign. In
column (6), is the interpretation if increasing or decreasing in that particular interval.

From the chart we see that


(a) f’(x) changes from negative to positive at -1, so f(-1) = 0 is a local minimum
value.
(b) f’(x), changes from negative to positive at 2, so f(2) = -27 is also a local
minimum value.
104
(c) f’(x) changes from positive to negative at 0, so f(0) = 5 is a local maximum
value.

2. g(x) = x + 2 sin x (0 ≤ x ≤ 2𝜋)

Solution:
First, let us get the first derivative of the function g(x)
g(x) = x + 2 sin x
g’(x) = 1 + 2 cos x
Setting g’(x) = 0
0 = 1 + 2 cos x
cos x = - ½
x = arc cos (-1/2)
Critical Points are
x = 2 𝜋/3
x = 4 𝜋/3
Interval g'(x) = 1 + 2 cos x g
0 < x < 2 𝜋/3 + Increasing on (0, 2𝜋/3)
2 𝜋/3 < x < 4 𝜋/3 - Decreasing on (2𝜋/3, 4𝜋/3)
4 𝜋/3 < x < 2 𝜋 + Increasing on (4𝜋/3, 2𝜋)

Because g’(x) changes from positive to negative at 2𝜋/3, the First Derivative Test
tells us that there is a local maximum at 2𝜋/3 and the local maximum value is
g(2𝜋/3) = 2𝜋/3 + 2 sin (2𝜋/3) = 2𝜋/3 + 2 (√3/2) = 2𝜋/3 + √3 ≈ 3.83

Also, g’(x) changes from negative to positive at 4𝜋/3 and the local minimum value
is
g(4𝜋/3) = 4𝜋/3 + 2 sin (4𝜋/3) = 4𝜋/3 + 2 (-√3/2) = 2𝜋/3 - √3 ≈ 2.46

3. y = (x – 2)3(x + 1)4

Solution:
First, let us get the first derivative of the function
y = (x – 2)3(x + 1)4
y’ = (x – 2)3 [4(x + 1)3] + (x + 1)4[3(x – 2)2]
y’ = 4(x – 2)3 (x + 1)3 + 3(x + 1)4 (x – 2)2
y’ = (x – 2)2 (x + 1)3 [4(x – 2) + 3(x + 1)]
y’ = (x – 2)2 (x + 1)3 (4x – 8 + 3x + 3)
y’ = (x – 2)2 (x + 1)3 (7x – 5)

Setting g’(x) = 0 to get the critical points


0 = (x – 2)2 (x + 1)3 (7x – 5)
Critical points are
(x – 2)2 = 0; x = 2
(x + 1)3 = 0; x = - 1
105
7x – 5 = 0; x = 5/7

Interval (x – 2)2 x–2 x+1 = y' y


x<-1 + - - = + increasing on (-∞, -1)
- 1 < x < 5/7 + - + = - decreasing on (-1, 5/7)
5/7 < x < 2 + - + = - decreasing on (5/7, 2)
x>2 + + + = + increasing on (2, ∞)

From the chart we see that


(a) y’ changes from positive to negative at -1, so point x = -1, y = 0, is a local
maximum value.
(b) y' changes from negative to positive at 2, so point x = 2, y = 0, is a local
minimum value.
(c) y’ does not change sign at 5/7, so point x = 5/7, y = -18.36, is neither a
maximum nor a minimum.

𝒙𝟐 +𝟑
4. y =
𝒙 −𝟏

Solution:
First, let us get the first derivative of the function
𝑥 2 +3
y=
𝑥 −1
(𝑥−1)(2𝑥) − (𝑥 2 +3)(1)
y’ =
(𝑥 −1)2
2𝑥 2 −2𝑥 − 𝑥 2 −3
y’ =
(𝑥 −1)2
𝑥 2 −2𝑥 − 3
y’ =
(𝑥 −1)2
(𝑥−3)(𝑥+1)
y’ =
(𝑥 −1)2

Setting g’(x) = 0 to get the critical points


(𝑥−3)(𝑥+1)
0=
(𝑥 −1)2
Critical points are
x – 3 = 0; x=3
x + 1 = 0; x = -1
x=1

Interval x–3 x+1 (x - 1)2 = y' y


x<-1 - - + = + increasing on (-∞, -1)
-1<x<1 - + + = - decreasing on (-1, 1)
1<x<3 - + + = - decreasing on (1, 3)
x>3 + + + = + increasing on (3, ∞)
106
From the chart we see that
(a) y’ changes from positive to negative at -1, so point x = -1, y = -3/2, is a local
maximum value.
(b) y' changes from negative to positive at 3, so point x = 3, y = 6, is a local
minimum value.
(c) y’ does not change sign at 1, so there is no minimum or maximum there.
(The Second Derivative Test could be used at 3 and -1 but not at 1 since y”
does not exist at this value.)

B. Confirm your answers in Examples A using Second Derivative Test:

1. f(x) = 3x4 – 4x3 – 12x2 + 5


Solution:
f'(x) = 12x3 – 12x2 – 24x
f”(x) = 36x2 – 24x – 24
f”(x) = 12(3x2 – 2x – 2)
Let us substitute the critical points when f’(x) = 0 to f”(x)
When x = −1
f”(-1) = 12[3(-1)2 – 2(-1) – 2] = 12(3+2-2) = 36
When x = 0
f”(0) = 12[3(0)2 – 2(0) – 2] = -24
When x = 2
f”(2) = 12[3(2)2 – 2(2) – 2] = 12(12+4-2) = 168

Applying the rules of the Second Derivative Test,


(a) f’’(x) > 0 at critical point x = -1, which means that f(-1) is indeed a local
minimum value.
(b) f’’(x) < 0 at critical point x = 0, which means that f(0) is indeed a local
maximum value.
(c) f’’(x) > 0 at critical point x = 2, which means that f(2) is also a local minimum
value.

2. g(x) = x + 2 sin x (0 ≤ x ≤ 2𝜋)

Solution:
g’(x) = 1 + 2 cos x
g’’(x) = - 2 sin x

Let us substitute the critical points when g’(x) = 0 to g”(x)


When x = 2 𝜋/3
g’’(2 𝜋/3) = - 2 sin (2 𝜋/3) = -0.577
When x = 4 𝜋/3
g’’(4 𝜋/3) = - 2 sin (4 𝜋/3) = 0.577
107
Applying the rules of the Second Derivative Test,
(a) g’’(x) < 0 at critical point x = 2 𝜋/3, which means that g(2 𝜋/3) is indeed a
local maximum value.
(b) g’’(x) > 0 at critical point x = 4 𝜋/3, which means that f(4 𝜋/3) is indeed a
local minimum value.

3. y = (x – 2)3(x + 1)4

Solution:
y’ = 4(x – 2)2 (x + 1)3 (7x – 5)
y’’ = 4(x – 2)2 (x + 1)3 (7) + 4(x – 2)2 (7x – 5) [3(x + 1)2]
+ (x + 1)3 (7x – 5) [8(x – 2)]
y’’ = 28(x – 2)2 (x + 1)3 + 12 (x – 2)2 (7x – 5) (x + 1)2
+ 8 (x + 1)3 (7x – 5) (x – 2)
y’’ = 4 (x – 2)(x + 1)2 [ 7(x – 2) (x + 1) + 3 (x – 2) (7x – 5)
+ 2 (x + 1) (7x – 5) ]
y’’ = 4 (x – 2)(x + 1)2 [ 7(x2 – x – 2) + 3 (7x2 – 19x + 10)
+ 2 (7x2 + 2x – 5) ]
y’’ = 4 (x – 2)(x + 1)2 ( 7x2 – 7x – 14 + 21x2 – 57x + 30
+ 14x2 + 4x – 10 )
y’’ = 4 (x – 2)(x + 1)2 ( 42x2 – 60x + 6 )
y’’ = 24 (x – 2)(x + 1)2 ( 7x2 – 10x + 1 )
Let us substitute the critical points when y’ = 0 to y”
when x = 2
y’’ = 24 (2 – 2)(2 + 1)2 [ 7(2)2 – 10(2) + 1 ] = 0
when x = -1
y’’ = 24 (-1 – 2)(-1 + 1)2 [ 7(-1)2 – 10(-1) + 1 ] = 0
when x = 5/7
y’’ = 24 (5/7 – 2)(5/7 + 1)2 [ 7(5/7)2 – 10(5/7) + 1 ] = 233.2

Applying the rules of the Second Derivative Test, y’’ = 0, which means that the test
fails.

NOTE: The Second Derivative Test is inconclusive when f”(c) = 0. In other


words, at such a point there might be a maximum, there might be a minimum,
or there might be neither. This test also fails when f”(c) does not exist. In
such cases the First Derivative Test must be used. In fact, even when both
tests apply, the First Derivative Test is often the easier one to use.

𝒙𝟐 +𝟑
4. y =
𝒙 −𝟏

Solution:
𝑥 2 −2𝑥 − 3
y’ =
(𝑥 −1)2
108

(𝑥−1)2 (2𝑥 − 2)−(𝑥 2 −2𝑥 − 3)(2)(𝑥−1)


y’’ =
(𝑥 −1)4
2(𝑥−1)2 (𝑥 − 1)−2(𝑥−1)(𝑥 2 −2𝑥 − 3)
y’’ =
(𝑥 −1)4
2(𝑥−1)[(𝑥 − 1)2 −(𝑥 2 −2𝑥 − 3)]
y’’ =
(𝑥 −1)4
2[𝑥 2 −2𝑥+1−𝑥 2 +2𝑥+ 3]
y’’ =
(𝑥 −1)3
2[4]
y’’ =
(𝑥 −1)3
8
y’’ =
(𝑥 −1)3
Let us substitute the critical points when y’ = 0 to y”
When x = 3
8 8
y’’ = 3 = =1
(3 −1) 8

When x = -1
8 8
y’’ = = = -1
(−1 −1)3 −8

Applying the rules of the Second Derivative Test,


(a) f’’(x) > 0 at critical point x = 3, which means that f(3) is indeed a local
minimum value.
(b) f’’(x) < 0 at critical point x = -1, which means that f(-1) is indeed a local
maximum value.
109
LESSON 7.3: CONCAVITY

Although the sign of the derivative of f reveals where the graph of f is


increasing or decreasing, it does not reveal the direction of the curvature.
The following figure suggests two ways to characterize the concavity of a
differentiable f on an open interval:
(a) f is concave up on an open interval if its tangent lines have increasing slopes
on that interval and is concave down if they have decreasing slopes.
(b) f is concave up on an open interval if its graph lies above the its tangent line
and concave down if it lies below its tangent lines.

Definition. If f is differentiable on an open interval, then f is said to be concave up


on the open interval if f’ is increasing on that interval, and f is said to be concave
down on the open interval if f’ is decreasing on that interval.

Since the slopes of the tangent lines to the graph of a differentiable function
f are the values of its derivative f’, it follows from Theorem 1 (applied to f’ rather
than f ) that f’ will be increasing on intervals where f’’ is positive and that f’ will be
decreasing on intervals where f’’ is negative. Thus we have the following theorem.

Concavity Test. Let f be twice differentiable on an open interval.


(a) If f’’(x) > 0 for every value of x in the open interval, then f is concave up on
that interval.
(b) If f’’(x) < 0 for every value of x in the open interval, then f is concave down
on that interval.
The following figure shows the graph of a function that is concave upward
(abbreviated CU) on the intervals (b, c), (d, e) and (e, p) and concave downward
(CD) on the intervals (a, b), (c, d), and (p, q).
110

Figure G

7.3.1 Points of Inflection

A point of inflection is a point at which the curve changes from concave


upward to concave downward, or vice versa (the points A, C and E in Figure H).
Also in Figure G, above, points of inflection are at points b, c, d and p. Notice that
if a curve has a tangent at a point of inflection, then the curve crosses its tangent
there.
In view of the Concavity Test, there is a point of inflection at any point where
the second derivative changes sign.

E
C
A

Figure H

A point at which y” vanishes or y” = 0, is a point of inflection, provided


y” changes sign at that point.

As noted in the previous section, a point where y’ = 0 is a maximum or a


minimum, provided y” ≠ 0. If y’ and y” both equal zero, the point is in general a
point of inflection with a horizontal tangent (point F in Figure A).
111
LESSON 7.4: SKETCHING OF POLYNOMIAL CURVES

Steps:

1. Find the points of intersection with the axes.


2. Determine the behavior of y for large values of x.
3. Locate the points where y’ = 0, and determine the maximum and minimum
values.
4. Locate the points where y” = 0 (points of inflection, in most cases), and
draw the tangent at each of those points.
5. If necessary, plot a few additional points.

ILLUSTRATIVE EXAMPLES

𝟏 𝟏
1. Let y = 𝟑x3 + 𝟐x2 – 6x + 8.
Find: (a) the critical numbers of the function;
(b) the points at which y has a relative or local maximum or minimum;
(c) the intervals on which the function is increasing or decreasing.

Solution:

a. Critical Points
1 1
y = 3x3 + 2x2 – 6x + 8
y’ = x2 + x − 6
y’ = (x + 3)(x − 2)

Solving for the critical values, set y’ = 0:


0 = (x + 3)(x − 2)
x1 + 3 = 0; x1 = −3
x2 – 2 = 0; x2 = 2
The critical values are when x1 = −3 and x2 = 2.
b. By using the second derivative test,
y’ = x2 + x − 6
y” = 2x + 1

Substituting the critical numbers in y”:


When x = -3
y" = 2(-3) + 1 = −5 ; y” < 0, therefore, local maximum
When x = 2
y" = 2(2) + 1 = 5 ; y” > 0, therefore, local minimum

Hence, by the second derivative test, the function has a relative maximum at
𝟒𝟑
point (−𝟑, − ).
𝟐
112
𝟐
Also, function y has a relative minimum at point (𝟐, 𝟑).

c. By using the first derivative test,


y’ = (x + 3)(x − 2)
Interval x+3 x−2 = y' y
x<-3 - - = + increasing on (-∞, -3)
-3<x<2 + - = - decreasing on (-3, 2)
x>2 + + = + increasing on (2, ∞)

Thus, y is increasing for x < −3 and x > 2, and decreasing for −3 < x < 2.

A sketch of part of the graph of the function is shown in the following figure.

2. Let f(x) = x4 + 2x3 − 3x2 − 4x + 4.

Find:
(a) the critical numbers of f;
(b) the points at which f has a relative extremum;
(c) the intervals on which f is increasing or decreasing.

Solution:

(a) Getting the first derivative,


f(x) = x4 + 2x3 − 3x2 − 4x + 4
f ′(x) = 4x3 + 6x2 − 6x − 4
By synthetic division,
f ′(x) = (x – 1) (4x2 + 10x + 4)
= (x – 1) [2(2x2 + 5x + 2)]
Thus, f ′(x) = 2(x − 1)(2x + 1)(x + 2),

Critical numbers are


x – 1 = 0; x=1
2x + 1 = 0; x = − 1/2
x + 2 = 0; x=−2

(b) Using Second Derivative Test,


f ′(x) = 4x3 + 6x2 − 6x − 4
f″(x) = 12x2 + 12x− 6
113
f″(x) = 6(2x2 + 2x− 1).

When x = 1
f″(1) = 6[2(1)2 + 2(1)− 1] = 6(3) = 18
When x = -1/2
f″(-1/2) = 6[2(-1/2)2 + 2(-1/2)− 1] = 6(1/2 – 1 – 1) = - 9
When x = -2
f″(-2) = 6[2(-2)2 + 2(-2)− 1] = 6(8 – 4 – 1) = 18

Therefore,
(i) at x = 1, f ″(1) = 18 > 0, there is a local minimum;
(ii) at x = −1/2 , f ″(−1/2) =− 9 < 0, there is a local maximum;
(iii) at x = −2, f ″(−2) = 18 > 0, there is a local minimum.

(c) Using the first derivative test


f ′(x) = 2(x − 1)(2x + 1)(x + 2),
Interval 2(x − 1) 2x + 1 x + 2 = f'(x) f
x<-2 - - - = - decreasing on (-∞, -2)
- 2 < x < -1/2 - - + = + increasing on (-2, -1/2)
-1/2 < x < 1 - + + = - decreasing on (-1/2, 1)
x>1 + + + = + increasing on (1, ∞)

Hence, f is increasing when x > 1 or – 2 < x < −1/2, and decreasing when x
<−2 or −1/2 < x < 1. The graph is sketched in the following figure.

3. For curve y = x4 – 4x3, find out the concavity, points of inflection, and local
maxima and minima. Sketch the curve.

a. Local maxima and minima


Using the first derivative test
y = x4 – 4x3
y’ = 4x3 – 12x2 = 4x2(x – 3)
Set y’ = 0
0 = 4x2(x – 3)
Critical points are
4x2 = 0; x=0
x – 3 = 0; x=3
114

Interval 4x2 x-3 = y' y


x<0 + - = - decreasing on (-∞, 0)
0<x<3 + - = - decreasing on (0, 3)
x>3 + + = + increasing on (3, ∞)

(a) y’ does not changes sign at 0, so point x = 0, y = 0, is neither a maxima or


minima.
(b) y' changes from negative to positive at 3, so point x = 3, y = -27, is a local
minima.

b. Concavity
Using the concavity test
y’ = 4x3 – 12x2
y” = 12x2 – 24x = 12x(x – 2)
Interval y" = 12x(x – 2) Concavity
x<0 + upward
0<x<3 - downward
x>3 + upward

c. points of inflection
y” = 12x(x – 2)
Setting y” = 0
0 = 12x(x – 2)

12x = 0; x = 0, thus y = (0)4 – 4(0)3 = 0


x – 2 = 0; x = 2, thus y = (2)4 – 4(2)3 = -16

Points of Inflection are


PI1 (0, 0) and PI2 (2, -16)

d. sketching the curve y = x4 – 4x3


i. Find the points of intersection with the axes.
Let y = 0; 0 = x4 – 4x3 = x3(x – 4)
x3 = 0; x1 = 0
x – 4 = 0; x2 = 4
Intersection points are at (0, 0) and (4, 0)

ii. Determine the behavior of y for large values of x.


It was already concluded in the first derivative test that y is decreasing for
interval (−∞, 0) and increasing for interval (3, ∞).

iii. Let us now locate our identified local maximum and minimum values.

iv. Let us also trace the points of inflection (0, 0) and (4, 0)
115

4. Sketch the graph of the function f(x) = x2/3 (6 – x)1/3.

a. Let us first find the local maxima and minima


Using the first derivative test
f(x) = x2/3 (6 – x)1/3
1 2
f’(x) = x2/3 (3)(6 – x)-2/3 (-1) + (6 – x)1/3 3 x-1/3
−𝑥 2/3 2(6−𝑥)1/3
f’(x) = +
3(6−𝑥)2/3 3 𝑥 1/3
−𝑥 + 2(6−𝑥)
f’(x) = 1/3
3𝑥 (6−𝑥)2/3
−𝑥 + 12 − 2𝑥
f’(x) = 1/3
3𝑥 (6−𝑥)2/3
12 − 3𝑥
f’(x) = 1/3
3𝑥 (6−𝑥)2/3
3(4 − 𝑥)
f’(x) = 1/3
3𝑥 (6−𝑥)2/3
4−𝑥
f’(x) = 1/3
𝑥 (6−𝑥)2/3

Set f’(x) = 0
4−𝑥
0 = 1/3
𝑥 (6−𝑥)2/3
Critical points are
4 – x = 0; x=4
x=0
x=6
116

Interval 4–x 𝑥 1/3 (6 − 𝑥)2/3 = f'(x) f


x<0 + - + = - decreasing on (-∞, 0)
0<x<4 + + + = + increasing on (0, 4)
4<x<6 - + + = - decreasing on (4, 6)
x>6 - + + = - decreasing on (6, ∞)

(a) y' changes from negative to positive at 0, so point x = 0, f(0) = 0, is a local


minima.
(b) y’ changes from positive to negative at 4, so point x = 4, f(4) = 3.2, is a local
maxima.
(c) The sign of y’ does not change at 6, so there is neither a local maxima nor
minima at this point. (The Second Derivative Test could be used at 4 but
not at 0 or 6 since f” does not exist at either of these numbers.)

b. Concavity
Using the concavity test
4−𝑥
f’(x) = 1/3
𝑥 (6−𝑥)2/3
[𝑥 1/3 (6−𝑥)2/3 ](−1) −(4 − 𝑥)[𝑥 1/3 (2/3)(6−𝑥)−1/3 (−1)+(6−𝑥)2/3 (1/3)𝑥 −2/3 ]
f”(x) = 2
[𝑥 1/3 (6−𝑥)2/3 ]

−2𝑥1/3 (6−𝑥)2/3
−𝑥 1/3 (6−𝑥)2/3 −(4 − 𝑥)[ + ]
3(6−𝑥)1/3 3𝑥2/3
f”(x) =
𝑥 2/3 (6−𝑥)4/3
−2𝑥 + (6−𝑥)
−𝑥 1/3 (6−𝑥)2/3 –(4 – 𝑥)[ ]
3𝑥2/3 (6−𝑥)1/3
f”(x) =
𝑥 2/3 (6−𝑥)4/3
(4 – 𝑥)(6−3𝑥)
−𝑥 1/3 (6−𝑥)2/3 –[ ]
3𝑥2/3 (6−𝑥)1/3
f”(x) =
𝑥 2/3 (6−𝑥)4/3
−3𝑥(6−𝑥) − (4 – 𝑥)(6−3𝑥)
3𝑥2/3 (6−𝑥)1/3
f”(x) =
𝑥 2/3 (6−𝑥)4/3
−18𝑥+3𝑥 2 − 24 + 18𝑥 −3𝑥 2
f”(x) =
3𝑥 4/3 (6−𝑥)5/3
− 24
f”(x) = 4/3
3𝑥 (6−𝑥)5/3
−8
f”(x) = 4/3
𝑥 (6−𝑥)5/3

−8
Interval f”(x) = Concavity
𝑥 4/3 (6−𝑥)5/3
(-∞, 0) - downward
(0, 4) - downward
(4, 6) - downward
(6, ∞) + upward
117
c. points of inflection

Since setting f”(x) = 0 will not be applicable in this problem, let us just observe
the chart on concavity test above.

The point where there is a change in the concavity is at point (6, 0), but take
note that the curve has vertical tangents at (0, 0) and (6, 0) because |f’(x)| → ∞
as x→ 0 and as x → 6.

d. sketching the curve f(x) = x2/3 (6 – x)1/3


i. Find the points of intersection with the axes.
Let f(x) = 0; 0 = x2/3 (6 – x)1/3
x2/3 = 0; x1 = 0
(6 – x) = 0; x2 = 6
1/3

Intersection points are at (0, 0) and (6, 0)

ii. Determine the behavior of y for large values of x.


It was already concluded in the first derivative test that y is decreasing
on intervals (-∞, 0) and (6, ∞).

iii. Let us now locate our identified local maximum and minimum values, (4, 3.2)
and (0, 0), respectively.

iv. Let us also trace the point of inflection (6, 0) and the vertical tangents at (0,
0) and (6, 0).
118
STUDENT ACTIVITY 7.1

A. Find the equations of the tangent/s and normal/s to the following equations at
the given point:

1. x2 + y2 = 25 at (3, 4)
2. x2 – 6x + 2y – 8 = 0 at x = 3
3. 2xy + 5x – 3y = 0 at a point where y = ½
4. (x + 2y)2 = x + 10 at (-1, 2)
5. x2 + y2 – 12x + 4y – 5 = 0 at a point where y = 1

B. Find the tangent/s as directed:

1. to the ellipse 4x2 + 9y2 = 40 with slope of mt = 2/9.


2. to the curve y = x4 – 4x3 + 2x2 – 4x + 15, perpendicular to the line
x + 8y = 7.
3. to the parabola (x + 2y)2 + 2x – y – 3 = 0, parallel to the line x – 4y = 2.
4. to the ellipse x2 + 4y2 = 8 and parallel to the line x + 2y = 6.
5. to the curve y = x4 – 14x3 + 17x + 40, perpendicular to the line x = 7y + 4.

C. For the following functions, find out the concavity, points of inflection, local maxima
and minima and sketch the curve.
1. y = x (4 – x2)1/2
2. y = x3 + (3/x)
3. y = (x2 – 10x)4
4. y = (x + 4) / x1/2
5. y = x / (x2 + 1)

SUMMARY

Tangent Line Equation. y – y1= mt (x – x1)


Normal Line Equation. y – y1= (- 1/mt) (x – x1)
Critical Points. The points at which y’ = 0.

First Derivative Test


At a point where y’=0,
(a) if y’ changes from positive to negative (as x increases), y is a maximum;
(b) if y’ changes from negative to positive, y is a minimum;
(c) if y’ does not change sign, y is neither a maximum nor a minimum.

Second Derivative Test


At a point where y’ = 0
(a) if y’’ < 0, the value of the function ‘y’ is a maximum;
(b) if y’’ > 0, ‘y’ is a minimum; and
(c) if y’’ = 0, the test fails

Concavity Test. Let f be twice differentiable on an open interval.


119
(a) If f’’(x) > 0 for every value of x in the open interval, then f is concave up on that interval.
(b) If f’’(x) < 0 for every value of x in the open interval, then f is concave down on that interval.

Point of Inflection. A point at which y” vanishes or y” = 0, provided y” changes sign at that


point.

Sketching of Polynomial Curves


1. Find the points of intersection with the axes.
2. Determine the behavior of y for large values of x.
3. Locate the points where y’ = 0, and determine the maximum and minimum values.
4. Locate the points where y” = 0 (points of inflection, in most cases), and draw the tangent at
each of those points.
5. If necessary, plot a few additional points.

REFERENCES

Love, Clyde E., Differential and Integral Calculus, The Macmillan Company
Stewart, James, Single Variable Calculus, Brooks/Cole CENGAGE Learning, 7th
Edition, 2012
Varela, Benjamin D., Workbook in Differential Calculus, 2009
120
OVERVIEW

MODULE In this module we will discuss all about polynomial


curves. The topics are tangents and normals, critical
points, maxima and minima, concavity, points of
inflection and sketching polynomial curves.

LEARNING OUTCOMES

At the end of this module, you should be


able to:
16. develop the equation of tangents and normals to plane curves;
17. locate the critical points and maxima and minima of a curve;
18. determine the concavity and points of inflection of the curve; and
19. sketch the polynomial curve.

LESSON 8.1: APPLICATIONS OF MAXIMA AND MINIMA

◼ In engineering and technology, there is a need to maximize or minimize


quantities. However, to be able to determine the maximum or minimum value/s,
quantities should be expressed as a differentiable function. Quantities to be
maximized or minimized are usually modified by superlative adjectives, such as
greatest, most, widest, smallest, nearest, longest, shortest, least, etc.

◼ In solving problems involving maxima or minima, verbal statements should be


converted into the language of mathematics by introducing formulas,
functions or equations. Since there are a lot of types of applications, there are
no hard rules on finding solutions. However, the following guidelines may be
helpful:
a. Read the problem carefully several times and think about the given facts,
together with the unknown quantities that are to be found.
b. If possible, sketch a picture or diagram and label it appropriately,
introducing variables for unknown quantities. Words such as “what,”
“find,” “how much,” “how far,” or “when” are hints of the unknown
quantities.
c. Make a list of known facts together with any relationship involving the
variables. A relationship may often be described by means of an equation
of some type.
d. After analyzing the list in preceding guideline, determine which variable is
to be maximized or minimized and express this variable as a function
of “one” of the other variables. Differentiate.
121
e. Find the critical numbers of the function obtained in the preceding
guideline and setting the derivative into zero (0) and test each of them for
maxima or minima.
f. Check your answers.

ILLUSTRATIVE EXAMPLES

Solve the following maxima and minima problems :

1. What number exceeds its square by the maximum amount?

Let: x = be the number


x2 = the square of this number
y = difference of the number and its square (because of the word
“exceeds”)

Mathematical Sentence(s): y = x – x2
What will be maximized? “y” since the problem requires the number which will
gain maximum difference if its square is subtracted
from it.
Solution:
y=x–x 2

y’ = 1 – 2x
To get critical points, set y’ = 0
0 = 1 – 2x
2x = 1
x=½

Let us check: (you can do this on your own and need not reflect in your
solutions)
Substitute our value of x to the original equation.
y = (1/2) – (1/2)2 = 1/4 or 0.25

Although this is not required in your solution, we can also try to substitute
values greater or less than the critical value of x.
y = (3/4) – (3/4)2 = 3/16 or 0.1875 < 0.25
y = (1/3) – (1/3)2 = 2/9 or 0.2222 < 0.25
This confirms the correctness of our solution. Less and more than 1/2 value of x
will lead to difference lesser than 0.25.

Therefore, x = ½ is the number that will gain maximum difference if its


square is subtracted from it.
122
2. Find two real numbers whose difference is 40 and whose product is minimum.

Let: x = be the bigger number


y = the lesser number
P = product
Mathematical Sentence(s): x – y = 40 (hint: difference)
xy = P (hint: product)
What will be minimized? “P” since the problem requires the numbers which will
gain minimum product maintaining their difference
to 40.
Solution:
x – y = 40
y = x – 40 eq(1) We will use this equation to get the relation of x and y
xy = P eq(2)

Substitute eq (1) to eq (2)


x (x – 40) = P
x2 – 40x = P

Differentiate
𝑑𝑃
= 2x - 40
𝑑𝑥
𝑑𝑃
To get critical points, set =0
𝑑𝑥
0 = 2x - 40
2x = 40
x = 20

Substitute back the value of x to eq(1) to get the value of y


y = 20 – 40
y = -20

Let us check: (you can do this on your own and need not reflect in your
solutions)
Substitute our values of x and y to equation (2) to get the product.
xy = P
(20)(-20) = - 400

Although this is not required in your solution, we can also try to substitute values
greater or less than the critical value of x.
(25)(-15) = - 375 > - 400
(10)(-30) = - 300 > - 400
This confirms the correctness of our solution. Less and more than 20 value of x will
lead to products greater than -400.
Therefore, 20 and -20 are the numbers that will gain minimum product
maintaining their difference to 40.
123
3. A rectangular field of fixed area is to be enclosed and divided into three
lots by parallels to one of the sides. What should be the relative
dimensions of the field to make the amount of fencing a minimum?

Illustration:
x

Let: x = be the width


y = be the length
A = lot area, constant value
F = fence

Mathematical Sentence(s): xy = A (hint: area of rectangular lot)


2x + 4y = F (hint: sides to be fenced)
What will be minimized? “F” since the problem requires the relative dimensions
of the field to make the amount of fencing a
minimum.

Solution:
xy = A
A
y= eq(1) We will use this equation to get the relation of x
x
and y
2x + 4y = F eq(2)

Substitute eq (1) to eq (2)


2x + 4y = F
A
2x + 4( ) = F
x
4A
2x + =F
x

Differentiate
𝑑𝐹 4A
=2-
𝑑𝑥 x2

𝑑𝐹
Set =0
𝑑𝑥
4A
0=2-
x2
124
2x2 − 4A
0=
x2
2
0 = 2x − 4A
2x2 = 4A
x2 = 2A
x = √2A
𝑥2
A= eq(3)
2

Substitute back the value of x to eq(1) to get the value of y


A
y=
√2A
√A
y=
√2
𝐀
y=√ eq(4)
𝟐

Now, let us solve for the relationship of the length and the width of the area by
substituting eq(3) to eq(4)
(𝑥 2 /2)
y=√
2

𝑥2
y=√
4
𝒙
y=
𝟐

Therefore, to make the amount of fencing of the field a minimum, the


length should be half of the width.

4. A rectangular lot is bounded at the back by a river. No fence is needed


along the river and there is to be a 24-ft opening in front. If the fence along
the front costs P 150 per ft, along the sides P 100 per ft, find the
dimensions of the largest lot which can be fenced for P 30,000.

Illustration:
x

24 ft

Let: x = be the length


125
y = be the width
A = lot area

Mathematical Sentence(s): xy = A (hint: area of rectangular lot)


30,000=150(x-24)+100(2y) (hint: cost of fence)

What will be maximized? “A” since the problem requires the dimensions of the
largest lot which can be fenced for P 30,000.
Solution:
xy = A eq(1)
30,000=150(x-24)+100(2y) eq(2) We will use this equation to get the
relation of x and y
Let us simplify eq (2) and express x in terms of y or vice versa.
30,000 = 150(x-24)+100(2y)
30,000 = 150x – 3600 + 200y
30,000 = 150x – 3600 + 200y
33,600 – 150x = 200y
33,600 – 150𝑥 = 200𝑦
200
168 – 0.75x = y eq(3)

Substitute eq(3) to eq(1)


x(168 – 0.75x) = A
168x – 0.75x2 = A

Differentiate
𝑑𝐴
= 168 – 1.5x
𝑑𝑥
𝑑𝐴
To get critical points, set =0
𝑑𝑥
0 = 168 – 1.5x
1.5x = 168
168
x=
1.5
x = 𝟏𝟏𝟐

Substitute back the value of x to eq(3) to get the value of y


y = 168 – 0.75(112)
y = 168 - 84
y = 84

Let us check: (you can do this on your own and need not reflect in your
solutions)
Substitute our values of x and y to equation (2) to get the length of fence in
terms of Area.
A = xy = (112)(84) = 9,408 sq. ft.
126
Although this is not required in your solution, we can also try to substitute values
greater or less than the critical value of x.
(115)(81.75) = 9401.25 < 9,408
(100)(75) = 7500 < 9,408
This confirms the correctness of our solution. Less and more than 112 value of x
will lead to products lesser than 9,408 sq.ft.

Therefore, 112 and 84 are the dimensions of the largest lot which can be
fenced for P 30,000.

5. An open box with a rectangular base is to be made from a rectangular


piece of cardboard 16 in wide and 21 in long by cutting out equal squares
from each corner and then bending up the sides. Find the side of the
corner square which will result in a box of maximum volume.

Illustration:
21
x x
x x

16 16 – 2x

x x
x x
21 – 2x

21 - 2x

x 16 – 2x

Let: x = be the length of the sides of the squares cut from the corners of
the cardboard
V = volume of the box

Mathematical Sentence(s):
V = (21 – 2x) (16 – 2x) x (hint: V = length x width x height)

What will be maximized? “V” since the problem requires the length of the side
of the corner square which will result in a box of
maximum volume.
127
Solution:
V = (21 – 2x) (16 – 2x) x eq(1)

Let us simplify eq (1) to get the value of x.


V = (21 – 2x) (16 – 2x) x
𝑑𝑉
= (21 – 2x)(16 – 2x)(1) + (21 – 2x)(x)(-2) + (16 – 2x)(x)(-2)
𝑑𝑥
𝑑𝑉
= (336 – 42x – 32x + 4x2) + (-42x + 4x2) + (-32x + 4x2)
𝑑𝑥
𝑑𝑉
= 12x2 – 148x + 336
𝑑𝑥

𝑑𝑉
To get critical points, set =0
𝑑𝑥
0 = 12x2 – 148x + 336
0 = 12𝑥 2 – 148𝑥 + 336
4
0 = 3x2 – 37x + 84
0 = (3x – 28)(x – 3)

Let us now get the values of x


0 = 3x – 28
3x = 28; x = 28/3
0=x–3
x=3

Since there are two values of x, let us substitute back these values to eq(1) for
us to select the value that will give the maximum volume.
When x = 28/3
V = [21 – 2(28/3)] [16 – 2(28/3)] (28/3)
V = -58.07 cu. in.
When x = 3
V = [21 – 2(3)] [16 – 2(3)] (3)
V = 450 cu. in. (maximum volume)

Therefore, 3 inches is the length of the side of the corner square which
will result in a box of maximum volume.
128
6. Find the maximum volume of a right circular cylinder that can be
inscribed in a cone of altitude 12 cm and a base radius of 4 cm, if the axes
of the cylinder and cone coincide each other.

Illustration:

12-y

12 cm x
12

x 4
4 cm
4 cm

Let: x = be the radius of the cylinder


y = height of the cylinder
V = volume of the cylinder

Mathematical Sentence(s):
V = 𝝅x2y (hint: V = 𝝅 x (radius)2 x height)
𝟏𝟐−𝒚 𝟏𝟐
= (hint: Use the ratio of similar right
𝒙 𝟒
triangles as shown above)

What will be maximized? “V” since the problem requires the maximum volume
of a right circular cylinder that can be inscribed in a
cone with the given dimensions.
Solution:
12−𝑦 12
= eq(1) we will use this equation to find the
𝑥 4
relationship of x and y

V = 𝜋x2y eq(2)

Let us simplify eq (1)


12−𝑦 12
=
𝑥 4
height/base of smaller 𝟏𝟐−𝒚 𝟏𝟐 height/base of bigger
right triangle = right triangle
𝒙 𝟒
4(12 – y) = 12x
48 – 4y = 12x
4y = 48 – 12x
y = 12 – 3x eq(3)
Now, let us substitute eq(3) to eq(2)
129

V = 𝜋 x2 (12 – 3x)
V = 3𝜋 x2 (4 – x)

𝑑𝑉
Find the
𝑑𝑥
𝑑𝑉
= 3𝜋 [x2(-1) + (4 – x)(2x)]
𝑑𝑥
𝑑𝑉
= 3𝜋 [-x2 + 8x – 2x2]
𝑑𝑥
𝑑𝑉
= 3𝜋 [8x – 3x2]
𝑑𝑥

𝑑𝑉
To get critical points, set =0
𝑑𝑥
0 = 3𝜋 [8x – 3x2]
0 = 3𝜋(8𝑥−3𝑥 2 )
3𝜋
0 = x (8 – 3x)

Let us now get the values of x


0=x
x=0
0 = 8 – 3x
3x = 8; x = 8/3 cm

Obviously, we cannot choose 0 as value of x since it will lead to zero volume.


Therefore, x = 8/3 and let us substitute back this value to eq(3) for us to solve the
value of y.
When x = 8/3
y = 12 – 3(8/3)
y = 4 cm
Therefore, the value of maximum volume is

V = 𝜋(8/3)2 (4)
𝟐𝟓𝟔𝝅
V= cu. cm. ≈ 89.36 cu. cm.
𝟗
130
7. A North-South highway intersects an East-West highway at a point P. An
automobile crosses P at 10 AM, traveling east at a constant speed of 20
mph. At that same instant, another automobile is two miles north of P,
traveling south at 50 mph. Find the time at which they are closest each
other and the minimum distance between them.

Illustration:

y1 = 50t
2 mi z
z y = 2 – 50t
y
x = 20 t
P
x = 20 t

Let: x = the distance travelled by the blue automobile away from point P
with a velocity of 20 mph ()
y = the distance travelled by the red automobile from point 2 miles
North of point P with a velocity of 50 mph towards South ()
z = distance between the 2 automobiles
t = time travelled by the two automobiles after 10:00 AM.

Mathematical Sentence(s):
z = √𝒙𝟐 + 𝒚𝟐 (hint: Pythagorean Theorem)
𝑦
y = 2 – 50t (hint: from vy = 𝑡1, making y1 = vyt; therefore,
y = 2 – y1)
𝑥
x = 20t (hint: from vx = 𝑡 ; therefore, x = vxt)

What will be maximized? “d” and “t” since the problem requires the time at
which the automobiles are closest to each other and
the minimum distance between them.
Solution:
z = √𝑥 2 + 𝑦 2 eq(1)
y = 2 – 50t eq(2)
x = 20t eq(3)

Let us substitute equations (2) and (3) in eq(1)


z = √(20𝑡)2 + (2 − 50𝑡)2
z = √400𝑡 2 + 4 − 200𝑡 + 2500𝑡 2
z = √2900𝑡 2 − 200𝑡 + 4 eq(4)
131
𝑑𝑧
Find the
𝑑𝑡
𝑑𝑧 5800𝑡 − 200
=
𝑑𝑡 2√2900𝑡 2 −200𝑡+4
𝑑𝑧 2900𝑡 − 100
=
𝑑𝑡 √2900𝑡 2 −200𝑡+4

𝑑𝑧
To get critical points, set =0
𝑑𝑡
2900𝑡 − 100
0=
√2900𝑡 2 −200𝑡+4
2900t – 100 = 0
1
t= hr
29
1 60 min 60 sec
t= hr ( ) = 2 min + 0.069 min ( 1 min ) = 2 min & 4.14 sec
29 1 hr
time = 10:02:4.14 AM

Let us find the value of z by substituting the value of t to eq(4)


z = √2900𝑡 2 − 200𝑡 + 4
1 2 1
z = √2900 (29) − 200 (29) + 4
z = 0.743 miles

Therefore, the time at which automobiles are closest each other is at 10:02:4.14
AM and the minimum distance between them is 0.743 miles.

8. A fence, 8 ft tall, stands on a level ground and runs parallel to a tall


building. If the fence is 1 ft from the building, find the shortest ladder that
will extend from the ground over the fence into the wall of the building.

Illustration:

y
L L 1
8 ft 8

x
x 1 ft

Let: x = the distance of the bottom of the ladder from the fence
y = the distance of the top of the ladder from the top of the fence
L = the length of the ladder

Mathematical Sentence(s):
L = √(𝒙 + 𝟏)𝟐 + (𝒚 + 𝟖)𝟐 (hint: Pythagorean Theorem)
132
𝒚 𝟖
= (hint: Use the ratio of similar right
𝟏 𝒙
triangles as shown above)

What will be maximized? “L” since the problem requires the shortest length of
ladder that will extend from the ground over the
fence into the wall of the building.
Solution:
𝑦 8 8
= ;y= eq(1) we will use this equation to find
1 𝑥 𝑥
the relationship of x and y

L = √(𝑥 + 1)2 + (𝑦 + 8)2 eq(2)

Let us substitute eq (1) to eq(2)

8 2
L = √(𝑥 + 1)2 + ( + 8)
𝑥

8 + 8𝑥 2
L = √𝑥 2 + 2𝑥 + 1 + ( )
𝑥

64 +128𝑥+64𝑥 2
L = √𝑥 2 + 2𝑥 + 1 + 𝑥2

64 128
L = √𝑥 2 + 2𝑥 + 1 + + + 64
𝑥2 𝑥

64 128
L = √𝑥 2 + 2𝑥 + 65 + +
𝑥2 𝑥

𝑑𝐿
Find the
𝑑𝑥
(2)(64) 128
𝑑𝐿 2𝑥+2− − 2
𝑥3 𝑥
=
𝑑𝑥 2 64 128
2√𝑥 +2𝑥+65+ 2 +
𝑥 𝑥
128 128
𝑑𝐿 2𝑥+2− 3 − 2
𝑥 𝑥
=
𝑑𝑥 2√𝑥 2 +2𝑥+65+ 2 +
64 128
𝑥 𝑥
64 64
𝑑𝐿 2(𝑥+1− 3 − 2 )
𝑥 𝑥
=
𝑑𝑥 2√𝑥 2 +2𝑥+65+ 2 +
64 128
𝑥 𝑥
64 64
𝑑𝐿 𝑥+1− 3 − 2
𝑥 𝑥
=
𝑑𝑥 √𝑥 2 +2𝑥+65+ 2 +
64 128
𝑥 𝑥
𝑥4 +𝑥3 −64−64𝑥
𝑑𝐿 𝑥3
=
𝑑𝑥 64 128
√𝑥 2 +2𝑥+65+ 2 +
𝑥 𝑥
𝑑𝐿 𝑥 4 +𝑥 3 −64𝑥−64
=
𝑑𝑥 𝑥 3 √𝑥 2 +2𝑥+65+ 2 +
64 128
𝑥 𝑥
𝑑𝐿
To get critical points, set =0
𝑑𝑥
133
𝑥 4 +𝑥 3 −64𝑥−64
0= 64 128
𝑥 3 √𝑥 2 +2𝑥+65+ 2 +
𝑥 𝑥
4 3
0 = 𝑥 + 𝑥 − 64𝑥 − 64
0 = 𝑥 3 (𝑥 + 1) − 64(𝑥 + 1)
0 = (𝑥 3 − 64)(𝑥 + 1)

Let us now get the values of x


0 = 𝑥 3 − 64
x3 = 64; x = 4 ft
0=x+1
x = -1

Obviously, we cannot choose -1 as value of x since it would mean that it is not


leaning on the fence. Therefore, x = 4 ft and let us substitute back this value to
eq(1) and eq(2) for us to solve the values of y and L, respectively.
8 8
y= ; y= ; y = 2 ft.
𝑥 4

Therefore, shortest length of ladder that will extend from the ground over the
fence into the wall of the building

L = √(𝑥 + 1)2 + (𝑦 + 8)2


L = √(4 + 1)2 + (2 + 8)2
L = √(5)2 + (10)2 = √125
L = 𝟓√𝟓 ft ≈ 11.18 ft.

9. Find the dimensions of the rectangle of maximum area that can be


inscribed in a semicircle of radius 2 meters, if two vertices lie on the
diameter.

Illustration:

y
y 2m y 2

𝑥
= √(2)2 − 𝑦 2
r=2m 2

Let: x = the length of the rectangle


y = the height of the rectangle
A = the area of the rectangle
134
Mathematical Sentence(s):
A = xy (hint: Area of the rectangle)
𝒙
= √(𝟐)𝟐 − 𝒚𝟐 (hint: Pythagorean Theorem)
𝟐

What will be maximized? “A” since the problem requires the dimensions of the
rectangle of maximum area that can be inscribed in
a semicircle with the given radius.
Solution:
𝑥
= √(2)2 − 𝑦 2 ; x = 2√4 − 𝑦 2 eq(1) we will use this equation to find
2
the relationship of x and y

A = xy eq(2)

Let us substitute eq (1) to eq(2)

A = (2√4 − 𝑦 2 ) y

𝑑𝐴
Find the
𝑑𝑦
𝑑𝐴 −2𝑦
= 2√4 − 𝑦 2 (1) + 2y
𝑑𝑦 2√4−𝑦2
𝑑𝐴 4𝑦2
= 2√4 − 𝑦 2 –
𝑑𝑦 2√4−𝑦2
𝑑𝐴 2𝑦2
= 2√4 − 𝑦 2 –
𝑑𝑦 √4−𝑦2
𝑑𝐴 2(4−𝑦2 ) − 2𝑦2
=
𝑑𝑦 √4−𝑦2
𝑑𝐴 8 −2𝑦2 − 2𝑦2
=
𝑑𝑦 √4−𝑦2
𝑑𝐴 8 −4𝑦2
=
𝑑𝑦 √4−𝑦2
𝑑𝐴 4(2 −𝑦2 )
=
𝑑𝑦 √4−𝑦2

𝑑𝐴
To get critical points, set =0
𝑑𝑦
4(2 −𝑦2 )
0=
√4−𝑦2
0 = 2 − 𝑦2

Let us now get the values of y


0 = 2 − 𝑦2
y2 = 2; y = ±√𝟐 m
135

Obviously, we cannot choose -√2 as value of y since it would mean that the
rectangle is not within the semicircle. Therefore, y = +√𝟐 m and let us substitute
back this value to eq(1) and eq(2) for us to solve the values of x and A, respectively.
2
x = 2√4 − (√2) ; x = 2√4 − 2 ; x = 𝟐√𝟐 m

Therefore, the maximum area that can be inscribed in a semicircle with the
radius of 2 meters,

A = xy
A = (2√2)(√2)
A = (2)(2)
A = 𝟒 m2

10. A window has the shape of a rectangle surmounted by a semi-circle. If the


perimeter of the window is 15 ft, find the dimensions which will allow the
maximum amount of light to enter.

Illustration:

r = x/2

Let: x = the width of the rectangular shape and the diameter of the semi-
circle surmounting it composing the window
y = the height of the rectangle
A = the total area of the window opening

Mathematical Sentence(s):
𝒙
15 = (x + 2y) + 𝝅 (𝟐) (hint: Perimeter of the rectangular portion
less the upper width and the circumference of a semi-circle = 𝜋r)
𝝅 𝒙 𝟐
A = xy + 𝟐 (𝟐) (hint: Area of the rectangle and the
𝝅𝒓𝟐
semicircle = )
𝟐
136
What will be maximized? “A” since the problem requires the dimensions which
will allow the maximum amount of light to enter, and
this means the Area of the window to be maximum.
Solution:
𝑥
15 = (x + 2y) + 𝜋 (2) eq(1) we will use this equation to find
the relationship of x and y

𝜋 𝑥 2
A = xy + 2 (2) eq(2)

Let us first simplify eq (1) to express y in terms of x

𝑥
15 = x + 2y + 𝜋 (2)
𝜋𝑥
2y = 15 – x –
2
30 − 2𝑥 − 𝜋𝑥
2y =
2
30 − 2𝑥 − 𝜋𝑥
y= eq(3)
4

Substitute eq(3) to eq(2)


30 − 2𝑥 – 𝜋𝑥 𝜋 𝑥 2
A = x( ) + 2 (2)
4
30𝑥 − 2𝑥 2 − 𝜋𝑥 2 𝜋𝑥 2
A= +
4 8
60𝑥 − 4𝑥 − 2𝜋𝑥 + 𝜋𝑥 2
2 2
A=
8
60𝑥 − 4𝑥 2 − 𝜋𝑥 2
A=
8

𝑑𝐴
Find the
𝑑𝑥
𝑑𝐴 60 − 8𝑥 − 2𝜋𝑥
=
𝑑𝑥 8

𝑑𝐴
To get critical points, set =0
𝑑𝑥
60 − 8𝑥 − 2𝜋𝑥
0=
8
0 = 60 − 8𝑥 − 2𝜋𝑥
8x + 2𝜋𝑥 = 60
2x(4 + 𝜋) = 60
60 𝟑𝟎
x= ; x= ≈ 4.20 ft
2 (4+ 𝜋) (𝟒 + 𝝅)

Let us now get the values of x to eq(3)


30 − 2(4.2) − 𝜋(4.2)
y= ; y = 2.1 ft
4
137

Therefore, area of the window that can allow maximum amount of light to enter

𝜋 4.2 2
A = (4.2)(2.1) +
2
(2)
A = 8.82 + 2.205𝜋
A = 15.747 ft2

STUDENT ACTIVITY 8.1

Solve the following maxima and minima problems:

1. The sum of two positive numbers is 2. Find the smallest value possible for the
sum of the cube of one number and the square of the other.

2. Find two positive real numbers whose sum is 40 and whose product is
maximum.

3. Find two numbers whose sum is 32, if the product of one by the cube of the
other is to be maximum.

4. A circular cylindrical container, open at the top and having a capacity of 24 cu.
In., is to be manufactured. If the cost of the material for the bottom of the
container is three times that of the cost of material for the curved part and if
there is no waste of material, find the dimensions which will minimize the cost.

5. A man in a rowboat 2 miles from the nearest point on a straight shoreline wishes
to reach a point 6 km further down the shore. If he can row at a rate of 3 mph
and run at the rate of 5 mph, how should he proceed in order to arrive at his
destination in the shortest possible time ?

6. Find the dimensions of the rectangle of maximum area that can be inscribed in
an equilateral triangle of side 1 meter, if two vertices of the rectangle lie on one
of the sides of the triangle.

7. Find the dimensions of the rectangle of maximum area having two vertices on
the x-axis and 2 vertices above the x-axis on the graph of y = 4 – x2.

8. A window has the shape of a rectangle surmounted by an equilateral triangle.


If the perimeter of the window is 12 ft, find the dimensions of the rectangle which
will produce the largest area for the window.

9. A steel storage tank for propane gas is to be constructed in the shape of a right
circular cylinder with a hemisphere at both ends. If the desired capacity is 100
cu. ft., what dimensions will require the least amount of steel ?
138

10. A silo consists of a right circular cylinder surmounted by a right circular cone
cover. What is the maximum volume of this silo that can be constructed using
a 100 m2 of plane GI sheet ?

LESSON 8.2: TIME RATES

In everyday living, many quantities changes or varies with time. As such


they are known as time rates.

In solving problems involving time rates, quantities are always


differentiated with respect to time.

Just like maxima and minima problems, there is no particular equation that
can be used in solving time rate problems. The following procedures will greatly
help you in dealing with such problems:

a. Read the problem carefully several times and think about the given facts,
together with the unknown quantities that are to be found.
b. Sketch a picture or diagram and label it appropriately as described in
the problem, introducing variables for unknown quantities. Words such
as “what,” “find,” “how much,” “how far,” or “when” are hints of the
unknown quantities.
c. Make a list of known facts together with any relationship involving the
variables. Time rates of other objects are usually given. A value may also
be given, like the specific time, height or length a time rate is being asked.
d. After analyzing the list in preceding guideline, finalize the equation.
Express a variable in terms of another variable and make sure that there
should be a maximum of two variables present in the equation.
e. Differentiate all variables with respect to time.
f. Substitute the given conditions of time, length, height, etc. to find the time
rate in that particular instant.

ILLUSTRATIVE EXAMPLES

Solve the following problems involving time rates.

1. A ladder, 20 ft long, leans against a vertical building. If the bottom of the


ladder slides away from the building horizontally at a rate of 3 ft/sec, how
fast is the ladder sliding down the building when the top of the ladder is
8 ft from the ground ?
139
Illustration:

𝑑𝑦
𝑑𝑡

y
𝑑𝑥 𝑓𝑡
= -3 𝑠𝑒𝑐
𝑑𝑡
x

Let: x = the horizontal distance of the bottom of the ladder from the base
of the building
𝑑𝑥
= the rate bottom of the ladder slides away from the building, -3
𝑑𝑡
ft/sec
y = the vertical distance of the top of the ladder from the base of the
building
𝑑𝑦
= the rate top of the ladder is sliding down the building, what is
𝑑𝑡
required in the problem

Mathematical Sentence(s):
(20)2 = x2 + y2 (hint: Pythagorean Theorem)

Condition: The rate the ladder is sliding down the building when
the top of the ladder is 8 ft from the ground, or when
y = 8 ft
Solution:
(20)2 = x2 + y2
400 = x2 + y2 eq(1)

Let us now differentiate with respect to time


𝑑𝑥 𝑑𝑦
0 = 2x + 2y
𝑑𝑡 𝑑𝑡
𝑑𝑥 𝑑𝑦
2x = - 2y
𝑑𝑡 𝑑𝑡
𝑑𝑥 𝑑𝑦
x =-y eq(2)
𝑑𝑡 𝑑𝑡

Next, we substitute the condition y = 8 ft to eq(1) to find the corresponding value of


x.
400 = x2 + (8)2
x2 = 400 – 64
x = √336
x = 4√21

𝑑𝑥
We substitute the values of , x and y to eq(2)
𝑑𝑡
𝑑𝑥 𝑑𝑦
x =-y
𝑑𝑡 𝑑𝑡
𝑑𝑦
(4√21) (-3) = - (8) 𝑑𝑡
140
𝑑𝑦 −12√21
=
𝑑𝑡 −8
𝒅𝒚 𝟑√𝟐𝟏
= ≈ 6.874 ft/sec
𝒅𝒕 𝟐

2. A man on a dock is pulling in a boat by means of a rope attached to the


vertical bow of the boat 1 ft above water level and passing through a
simple pulley located on the dock 8 ft above water level. If he pulls in the
rope at a rate of 2 ft/sec, how fast is the boat approaching the dock when
the bow of the boat is 25 ft from a point on the water directly below the
pulley?

Illustration:

7 ft
8 ft
𝑑𝑥
𝑑𝑡
x 1 ft

Let: x = the horizontal distance of the bottom of the ladder from the base
of the building
𝑑𝑥
= the rate bottom of the ladder slides away from the building, what
𝑑𝑡
is required in the problem
L = the length of the rope attached to the boat and the pulley on the
dock
𝑑𝐿
= the rate the man is pulling the rope, 2 ft/sec
𝑑𝑡

Mathematical Sentence(s):
x2 + 72 = L2 (hint: Pythagorean Theorem)

Condition: The rate the boat is approaching the dock when the
bow of the boat is 25 ft from a point on the water
directly below the pulley, or when x = 25 ft
Solution:
x2 + 7 2 = L 2
x2 + 49 = L2 eq(1)

Let us now differentiate with respect to time


𝑑𝑥 𝑑𝐿
2x = 2L
𝑑𝑡 𝑑𝑡
𝑑𝑥 𝑑𝐿
x =L eq(2)
𝑑𝑡 𝑑𝑡
141
Next, we substitute the condition x = 25 ft to eq(1) to find the corresponding value
of L.
(25)2 + 49 = L2
L2 = 625 + 49
L = √674
L = 25.962 ft
𝑑𝐿
We substitute the values of , x and L to eq(2)
𝑑𝑡
𝑑𝑥 𝑑𝐿
x =L
𝑑𝑡 𝑑𝑡
𝑑𝑥
(25) 𝑑𝑡 = (25.962) (2)
𝑑𝑥 51.924
=
𝑑𝑡 25
𝒅𝒙
= 2.077 ft/sec
𝒅𝒕

3. An airplane, flying at a constant speed of 360 mph and climbing at an


angle of 45 degrees, passes over a point P on the ground at an altitude of
10,560 ft. Find the rate at which its distance from P is changing one
minute later.

Illustration:

45°
90° θ=135° 𝑑𝑧

z z 𝑑𝑡
10,560 ft 10,560 ft
𝑑𝑧
𝑑𝑡 = 2 miles
45°
P P

Convert:
1 𝑚𝑖𝑙𝑒
10,560 ft x = 2 miles
5280 𝑓𝑡

Let: L = the distance of the plane from the altitude above point P
𝑑𝐿
= the rate the plane is moving upward at an angle of 45° against
𝑑𝑡
the horizontal, 360 mph
z = the distance of the plane from point P on the ground
𝑑𝑧
= the rate at which the distance of the plane from P is changing
𝑑𝑡

Mathematical Sentence(s):
z = √𝐋𝟐 + (𝟐)𝟐 − 𝟐𝐋(𝟐)𝐜𝐨𝐬 𝟏𝟑𝟓° (hint: Cosine Law)
L = 360t (hint: velocity = distance/time;
distance = velocity x time, thus L = 360t)
142

Condition: The rate at which the distance of the plane from P is


changing one minute later or when t = 1 min
Solution:
z = √L2 + (2)2 − 2L(2)cos 135°
z = √(360t)2 + 4 − 4(360t)cos 135°
z = √129,600t 2 + 1,018.234t + 4 eq(1)

Let us now differentiate with respect to time


𝑑𝑧 2(129,600𝑡)+1,018.234
=
𝑑𝑡 2√129,600t2 +1,018.234t+4
𝑑𝑧 259,200𝑡+1,018.234
= eq(2)
𝑑𝑡 2√129,600t2 +1,018.234t+4

Next, we substitute the condition t = 1 min = 1/60 hr to eq(2)


1
𝑑𝑧 259,200( )+1,018.234
60
=
𝑑𝑡 1 2 1
2√129,600( ) +1,018.234( )+4
60 60

𝑑𝑧 4,320+1,018.234
=
𝑑𝑡 2√36 +16.971+4
𝑑𝑧 5,338.234
=
𝑑𝑡 23.096
𝒅𝒛
= 231.134 mph
𝒅𝒕
143
4. Water is flowing into an inverted conical reservoir, 20 ft deep and 10 ft
across the top, at the rate of 20 ft3 per minute. Find how fast the water
surface is rising when the water is 6 ft deep.

Illustration:
10 ft
10
5
dV
= 20 ft3/min
dt
dh r
dt
20 ft 20 h

Let: r = the radius of the water surface inside the inverted conical reservoir
𝑑𝑉
= the rate of flow of water into the inverted conical reservoir, 20
𝑑𝑡
ft3/min
h = the height of water level inside the inverted conical reservoir
𝑑ℎ
= the rate the water surface is rising, the unknown
𝑑𝑡

Mathematical Sentence(s):
𝒓 𝟓
= (hint: ratio and proportion of similar right triangle)
𝒉 𝟐𝟎
V = 𝝅 r2 h (hint: Volume of a cone)

Condition: The rate the water surface is rising when the water is
6 ft deep, or when h = 6 ft
Solution:
By ratio and proportion of similar right triangle as shown on the right figure above,
we can express the radius in terms of height
𝑟 5
=
ℎ 20

𝑟 = eq(1)
4

Substitute eq(1) to the volume equation


V = 𝜋 r2 h
ℎ 2
V=𝜋( ) h
4
ℎ2
V = 𝜋 ( )h
16
𝜋ℎ3
V= eq(2)
16
144

Let us now differentiate with respect to time


𝑑𝑉 3𝜋 ℎ2 𝑑ℎ
=
𝑑𝑡 16 𝑑𝑡
𝑑𝑉 3𝜋 ℎ2 𝑑ℎ
= eq(3)
𝑑𝑡 16 𝑑𝑡

𝑑𝑉
Next, we substitute the condition h = 6 ft and = 20 ft3/min to eq(3) to find the
𝑑𝑡
𝑑ℎ
corresponding value of .
𝑑𝑡
𝑑𝑉 3𝜋 ℎ2 𝑑ℎ
=
𝑑𝑡 16 𝑑𝑡
3𝜋 (6)2 𝑑ℎ
20 =
16 𝑑𝑡
𝑑ℎ 20(16)
=
𝑑𝑡 3𝜋 (6)2
𝒅𝒉
= 0.059 ft/min
𝒅𝒕

5. A trapezoidal trough is 10 ft long, 4 ft wide at the top, 2 ft wide at the bottom,


and 2 ft deep. If the water flows in at 10 ft3/min, find how fast the water surface
is rising, when the water is 7.5 inches deep.

Illustration:

dV
= 10 ft3/min 4
dt
1

4 ft
x 2 x

2
h
10 ft
2 ft

2 ft 2
Let: h = the water level inside the trough
𝑑ℎ
= the rate the water surface is rising, the unknown
𝑑𝑡
2 + 2x = width of the water surface
𝑑𝑉
= the rate the water flows in the trough, 10 ft3/min
𝑑𝑡

Mathematical Sentence(s):
𝒙 𝟏
= (hint: ratio and proportion of similar right triangle)
𝒉 𝟐
𝟐+𝟐𝒙 𝑏𝑢 +𝑏𝑑
V= h (10) (hint: Area of trapezoid (A = h) multiplied by length
𝟐 2
of trough)
145
Condition: The rate the water surface is rising when the water is
7.5 inches deep, or when h = 7.5 in.
Solution:
By ratio and proportion of similar right triangle created by the slanting side of the
trapezoid against the vertical, as shown on the right figure above, we can express
“x” in terms of height
𝑥 1
=
ℎ 2

x= eq(1)
2

Substitute eq(1) to the volume equation


2+2𝑥
V= h (10)
2
2(1 + 𝑥)
V= h (10)
2
V = 10 h (1 + x)

V = 10 h (1 + 2)
2+ℎ
V = 10 h ( 2 )
V = 5 h (2 + h)
V = 10h + 5h2 eq(2)

Let us now differentiate with respect to time


𝑑𝑉 𝑑ℎ
= (10 + 10h)
𝑑𝑡 𝑑𝑡
𝑑𝑉 𝑑ℎ
= 10(1 + h) eq(3)
𝑑𝑡 𝑑𝑡

7.5 𝑑𝑉
Next, we substitute the condition h = 7.5 in = ft = 0.625 ft. and = 10 ft3/min to
12 𝑑𝑡
𝑑ℎ
eq(3) to find the corresponding value of .
𝑑𝑡
𝑑𝑉 𝑑ℎ
= 10(1 + h)
𝑑𝑡 𝑑𝑡
𝑑ℎ
10 = 10(1 + 0.625)
𝑑𝑡
𝑑ℎ 10
=
𝑑𝑡 16.25
𝒅𝒉 𝟖
= or 0.615 ft/min
𝒅𝒕 𝟏𝟑
146
STUDENT ACTIVITY 8.2

Solve the following time rates problems.

1. A boy flying a kite pays out string at a rate of 2 ft/sec as the kite moves
horizontally at an altitude of 100 ft. Assuming there is no sag in the string, find
the rate at which the kite is moving when 125 ft of string have been paid out.
2. A softball diamond has the shape of a square wide sides 60 ft long. If a player
is running from second base to third at a speed of 24 ft/sec, at what rate is his
distance from home plate changing when she is 20 ft from the third?
3. The ends of a water trough 8 ft long are equilateral triangles whose sides are 2
ft long. If water is being pumped into the trough at a rate of 5 ft 3/min, find the
rate at which the water level is rising when the depth is 8 in.
4. A girl starts at a point A and runs east at the rate of 10 ft/sec. One minute later,
another girl starts at A and runs north at a rate of 8 ft/sec. At what rate is the
distance between them changing 1 minute after the second girl starts ?
5. A ship sails east 20 miles and then turns N30oW. If the ship’s speed is 10 mph,
find how fast it will be leaving the starting point 6 hrs after the start.

SUMMARY
Maxima or Minima
a. Read the problem carefully.
b. Sketch a picture or diagram and label it appropriately.
c. Make a list of known facts together with any relationship involving the variables.
d. Determine which variable is to be maximized or minimized and express this variable as a
function of “one” of the other variables.
e. Differentiate.
f. Find the critical numbers of the function obtained in the preceding guideline and setting the
derivative into zero (0) and test each of them for maxima or minima.
g. Check your answers.

Time Rates
a. Read the problem carefully.
b. Sketch a picture or diagram and label it appropriately as described in the problem.
c. Make a list of known facts together with any relationship involving the variables. Time rates
of other objects are usually given.
d. Finalize the equation. Express a variable in terms of another variable and make sure that
there should be a maximum of two variables present in the equation.
e. Differentiate all variables with respect to time.
f. Substitute the given conditions to find the time rate in a particular instant.

REFERENCES

Love, Clyde E., Differential and Integral Calculus, The Macmillan Company
Stewart, James, Single Variable Calculus, Brooks/Cole CENGAGE Learning, 7th
Edition, 2012
Varela, Benjamin D., Workbook in Differential Calculus, 2009

You might also like